Вы находитесь на странице: 1из 215

Kombinatorika jegyzet s feladatgyjtemny

Kirly Balzs, Tth Lszl


Pcsi Tudomnyegyetem
2011
2
Lektor: Ktai Imre egyetemi tanr, az MTA rendes tagja
Tartalomjegyzk
Elsz 5
I. Jegyzet 7
I.1. Permutcik, varicik, kombincik 9
I.1.1. Permutcik . . . . . . . . . . . . . . . . . . . . . . . . . . . . . . . . . . . . . 9
I.1.2. Ismtlses permutcik . . . . . . . . . . . . . . . . . . . . . . . . . . . . . . . 11
I.1.3. Varicik . . . . . . . . . . . . . . . . . . . . . . . . . . . . . . . . . . . . . . . 12
I.1.4. Ismtlses varicik . . . . . . . . . . . . . . . . . . . . . . . . . . . . . . . . . 13
I.1.5. Kombincik . . . . . . . . . . . . . . . . . . . . . . . . . . . . . . . . . . . . . 14
I.1.6. Ismtlses kombincik . . . . . . . . . . . . . . . . . . . . . . . . . . . . . . . 16
I.2. A binomilis s a polinomilis ttel 19
I.2.1. A binomilis ttel . . . . . . . . . . . . . . . . . . . . . . . . . . . . . . . . . . 19
I.2.2. A polinomilis ttel . . . . . . . . . . . . . . . . . . . . . . . . . . . . . . . . . 22
I.2.3. A binomilis egytthatk tulajdonsgai . . . . . . . . . . . . . . . . . . . . . . 22
I.3. Szitakpletek 27
I.4. sszeszmllsi feladatok 31
I.4.1. sszeszmllsi feladatok . . . . . . . . . . . . . . . . . . . . . . . . . . . . . . 31
I.4.2. Egsz szmok partcii . . . . . . . . . . . . . . . . . . . . . . . . . . . . . . . . 34
I.5. Kombinatorika a geometriban 37
I.6. Fibonacci-szmok 41
I.7. Catalan-szmok 45
I.8. Stirling-szmok 51
I.8.1. Msodfaj Stirling-szmok . . . . . . . . . . . . . . . . . . . . . . . . . . . . . 51
I.8.2. Elsfaj Stirling-szmok . . . . . . . . . . . . . . . . . . . . . . . . . . . . . . . 54
I.9. Grfelmleti fogalmak 59
I.9.1. A grfok szemlletes bevezetse . . . . . . . . . . . . . . . . . . . . . . . . . . . 59
I.9.2. Egyszer grfok . . . . . . . . . . . . . . . . . . . . . . . . . . . . . . . . . . . 62
I.9.3. Fagrfok . . . . . . . . . . . . . . . . . . . . . . . . . . . . . . . . . . . . . . . 67
I.9.4. Fesztfk, Kruskal-algoritmus . . . . . . . . . . . . . . . . . . . . . . . . . . . 72
I.9.5. Multigrfok, grfok bejrsa . . . . . . . . . . . . . . . . . . . . . . . . . . . . 75
3
4 TARTALOMJEGYZK
II. Feladatgyjtemny 77
II.1. Permutcik, varicik, kombincik 79
II.1.1. Kidolgozott pldk . . . . . . . . . . . . . . . . . . . . . . . . . . . . . . . . . . 79
II.1.2. Tovbbi gyakorl feladatok . . . . . . . . . . . . . . . . . . . . . . . . . . . . . 100
II.2. A binomilis s a polinomilis ttel 105
II.2.1. Kidolgozott pldk . . . . . . . . . . . . . . . . . . . . . . . . . . . . . . . . . . 105
II.2.2. Tovbbi gyakorl feladatok . . . . . . . . . . . . . . . . . . . . . . . . . . . . . 108
II.3. Szitakpletek 111
II.3.1. Kidolgozott pldk . . . . . . . . . . . . . . . . . . . . . . . . . . . . . . . . . . 111
II.3.2. Tovbbi gyakorl feladatok . . . . . . . . . . . . . . . . . . . . . . . . . . . . . 115
II.4. sszeszmllsi feladatok 117
II.4.1. Kidolgozott pldk . . . . . . . . . . . . . . . . . . . . . . . . . . . . . . . . . . 117
II.4.2. Tovbbi gyakorl feladatok . . . . . . . . . . . . . . . . . . . . . . . . . . . . . 120
II.5. Kombinatorika a geometriban 123
II.5.1. Kidolgozott pldk . . . . . . . . . . . . . . . . . . . . . . . . . . . . . . . . . . 123
II.5.2. Tovbbi gyakorl feladatok . . . . . . . . . . . . . . . . . . . . . . . . . . . . . 125
II.6. Fibonacci-szmok 127
II.6.1. Kidolgozott pldk . . . . . . . . . . . . . . . . . . . . . . . . . . . . . . . . . . 127
II.6.2. Tovbbi gyakorl feladatok . . . . . . . . . . . . . . . . . . . . . . . . . . . . . 134
II.7. Catalan-szmok 135
II.7.1. Kidolgozott pldk . . . . . . . . . . . . . . . . . . . . . . . . . . . . . . . . . . 135
II.7.2. Tovbbi gyakorl feladatok . . . . . . . . . . . . . . . . . . . . . . . . . . . . . 140
II.8. Stirling-szmok 143
II.8.1. Kidolgozott pldk . . . . . . . . . . . . . . . . . . . . . . . . . . . . . . . . . . 143
II.8.2. Tovbbi gyakorl feladatok . . . . . . . . . . . . . . . . . . . . . . . . . . . . . 146
II.9. Grfelmleti fogalmak 147
II.9.1. Kidolgozott pldk . . . . . . . . . . . . . . . . . . . . . . . . . . . . . . . . . . 147
II.9.2. Tovbbi gyakorl feladatok . . . . . . . . . . . . . . . . . . . . . . . . . . . . . 171
III. Megoldsok, tmutatsok, eredmnyek 173
III.1. Permutcik, varicik, kombincik 175
III.2. A binomilis s a polinomilis ttel 185
III.3. Szitakpletek 189
III.4. sszeszmllsi feladatok 191
III.5. Kombinatorika a geometriban 195
TARTALOMJEGYZK 5
III.6. Fibonacci-szmok 199
III.7. Catalan-szmok 205
III.8. Stirling-szmok 207
III.9. Grfelmleti fogalmak 209
6 TARTALOMJEGYZK
Elsz
Ezt a jegyzetet s feladatgyjtemnyt azoknak az eladsoknak, illetve gyakorlatoknak az anyagai
alapjn rtuk, amelyeket az elmlt vekben a PTE TTK Matematika BSc szakos hallgatknak a
Kombinatorika cm trgy keretben tartottunk a nappali s levelez tagozaton.
A jegyzet s a pldatri rsz felleli az emltett szak Kombinatorika trgynak tematikjban
szerepl anyag szinte teljes egszt.
Ez a tananyag jl hasznlhat tovbb a Matematika BSc szakon az Elemi matematika trgyhoz,
tovbb a Programtervez Informatikus s Fizika BSc szakokon a Diszkrt matematika s ezzel
rokon trgyakhoz.
Arra trekedtnk, hogy a legegyszerbb bizonytsokat, megoldsokat, magyarzatokat adjuk.
Ugyanakkor sok esetben tbb bizonytst, illetve megoldst is adtunk ugyanarra a problmra.
Az elmleti rszben is tallhatk megoldott s kitztt feladatok, olyanok, amelyek kiegsztik a
bemutatott anyagrszeket. Remljk, hogy mindezek hozzsegtenek az anyag alaposabb s jobb
megrtshez.
A pldatri rszben gyakorl s nehezebb feladatok is szerepelnek, ezek rszletes megoldsokkal,
illetve tmutsokkal s eredmnyekkel vannak elltva.
Kszlt a Trsadalmi Megjuls Operatv Program TMOP - 4.1.2-08/1/A kdszm plyza-
tnak keretben L
A
T
E
Xdokumentumkszt rendszer felhasznlsval, bngszhet PDF formtum-
ban.
Ksznetnket fejezzk ki a lektornak, Ktai Imre egyetemi tanrnak, az MTA rendes tagjnak,
akinek szrevteleit s hasznos tancsait felhasznltuk a tananyag vgs vltozatnak kidolgoz-
sban.
A szerzk
7
8 TARTALOMJEGYZK
Els rsz
Jegyzet
9
I.1. fejezet
Permutcik, varicik, kombincik
I.1.1. Permutcik
I.1.1.1. Feladat. Hnyfle sorrendje van az 1, 2, 3 szmoknak?
Megolds. Hatfle sorrend van, ezek a kvetkezk:
123 132 213 231 312 321
I.1.1.2. Feladat. Hnyfle sorrendje van az a, b, c, d betknek?
Megolds. A sorrendek szma 24, ezek:
abcd abdc acbd acdb adbc adcb
bacd badc bcad bcda bdac bdca
cabd cadb cbad cbda cdab cdba
dabc dacb dbac dbca dcab dcba
I.1.1.3. Denci. Tekintsnk vges sok klnbz elemet. Ezek klnbz sorrendjeit az elemek
permutciinak nevezzk. A permutcik kpzst (felrst) az elemek permutlsnak nevezzk.
Ha adott n klnbz elem, akkor jellje P
n
ezek sszes permutciinak szmt.
Az I.1.1.1 Feladatban P
3
= 6, az I.1.1.2 Feladatban pedig P
4
= 24. Krds: Mennyi P
n
?
Emlkeztetnk a kvetkez fogalomra: A k 1 termszetes szm faktorilisa k! =123 k.
gy pl. 1! = 1, 2! = 2, 3! = 6, 4! = 24, . . . . Rgtn addik, hogy (*) k! = (k1)! k, ahol k 2. Ha
(*)-ban k = 1, akkor kapjuk, hogy 1 = 0!, ez indokolja, hogy megllapods szerint 0! = 1 legyen.
I.1.1.4. Ttel. Ha n1, akkor n klnbz elem sszes permutciinak szma n!, azaz P
n
=n! .
Bizonyts. Az els helyre az n elem kzl brmelyiket rhatjuk, ez n lehetsg, a msodik helyre
a megmaradt n1 elem brmelyike kerlhet, ez n1 lehetsg. Az els kt elemet gy n(n1)-
flekppen vlaszthatjuk meg. Tovbb, a harmadik elem a megmaradt n2 elem brmelyike lehet,
ez jabb n2 lehetsg, ..., az utols, n-edik elem megvlasztsra n(n1) = 1 lehetsgnk
van. Kapjuk, hogy P
n
=n(n1)(n2) 3 2 1 = n!.
Mskpp: n szerinti indukcival. Ha n = 1, akkor P
1
= 1, ami igaz. Tegyk fel, hogy P
n1
=
=(n1)!. Ha most n klnbz elem permutciit kpezzk, akkor az els helyre brmelyik elem
11
12 I.1. FEJEZET. PERMUTCIK, VARICIK, KOMBINCIK
kerlhet, a fennmarad n1 elemet pedig P
n1
=(n1)!-flekppen permutlhatjuk. gy minden
permutcit megkapunk s pontosan egyszer, teht
P
n
=P
n1
+P
n1
+... +P
n1
. .
nszer
= nP
n1
=n(n1)! = n! ,
amit igazolnunk kellett.
Teht P
1
= 1! = 1, P
2
= 2! = 2, P
3
= 3! = 6, P
4
= 4! = 24, P
5
= 5! = 120, P
6
= 6! = 720,... .
A tovbbiakban emlkeztetnk az injektv, szrjektv s bijektv fggvnyek fogalmra s
nhny tulajdonsgra.
I.1.1.5. Denci. Legyenek A s B tetszleges nemres halmazok s legyen f : A B egy
fggvny (lekpezs). Azt mondjuk, hogy
f injektv, ha A klnbz elemeinek klnbz kpelemek felelnek meg, azaz, ha brmely
x
1
, x
2
A, x
1
,=x
2
esetn f(x
1
) ,=f(x
2
). Ez egyenrtk a kvetkez lltssal : Brmely x
1
, x
2
A
esetn, ha f(x
1
) = f(x
2
), akkor x
1
=x
2
;
f szrjektv, ha B-nek minden eleme kpelem, azaz, ha brmely y B esetn ltezik x A
gy, hogy f(x) = y. Ez a felttel gy is rhat: f(A) :=f(x) : x A = B;
f bijektv, ha injektv s szrjektv, azaz, ha minden y B-re ltezik egy s csak egy x A
gy, hogy f(x) = y.
I.1.1.6. Feladat. Adjunk pldt olyan vges A s B halmazokra s olyan f : A B fggvnyre,
amely
i) injektv, de nem szrjektv,
ii) szrjektv, de nem injektv,
iii) nem injektv s nem szrjektv.
I.1.1.7. Feladat. Legyenek A s B egyenl szmossg vges halmazok s legyen f : A B egy
fggvny. (Specilisan, legyen A=B egy vges halmaz s legyen f : AA egy fggvny.) Igazoljuk,
hogy a kvetkez lltsok egyenrtkek:
i) f injektv,
ii) f szrjektv,
iii) f bijektv.
Az a
1
, a
2
, ..., a
n
klnbz elemek permutciit gy is denilhatjuk, mint az adott a
1
, a
2
, ..., a
n
elemekbl alkotott (a
i
1
, a
i
2
, ..., a
i
n
) olyan rendezett elem n-eseket (n komponens vektorokat),
amelyekben a
i
1
, a
i
2
, ..., a
i
n
pronknt klnbzek. Ennl pontosabb denci a kvetkez:
I.1.1.8. Denci. Permutciknak nevezzk egy vges halmaz nmagra val bijekciit (bijektv
lekpezseit). Rszletesebben: ha A egy vges, n elem halmaz (n 1), akkor A permutcii az
f : A A bijektv fggvnyek. Ha A = 1,2, ...n, akkor teht A permutcii az f : 1,2, ..., n
1,2, ..., n bijektv fggvnyek. Ezeket n-edfok permutciknak nevezzk s gy jelljk:
f =
_
1 2 . . . n
f(1) f(2) . . . f(n)
_
.
I.1.2. ISMTLSES PERMUTCIK 13
I.1.2. Ismtlses permutcik
I.1.2.1. Feladat. Hnyfle klnbz sorrendje van a MATEMATIKA sz betinek?
Megolds. Klnbztessk meg a kt M bett, a hrom A bett s a kt T bett, pl. gy,
hogy ms-ms sznnel jelljk ket: MATEMAT IKA.
Akkor 10 klnbz elem permutciirl van sz s ezek szma P
10
= 10!. De a hrom A bet
egyms kztti permutlsa, ezek szma 3! =6, valjban nem vltoztat a sorrenden (ismt azonos
sznnel rjuk ket). Hasonlan az A s T betkre vonatkozan. A lehetsges sorrendek szma:
10!
2! 3! 2!
= 151 200.
I.1.2.2. Denci. Az olyan elemek klnbz sorrendjeit, amelyek kztt egyenlek is vannak
ismtlses permutciknak nevezzk. Ha n elem kzl k
1
elem egymssal egyenl (azonos),
tovbbi k
2
elem egymssal egyenl s az elbbiektl klnbz,..., tovbbi k
r
elem egymssal egyenl
s az elbbiektl klnbz, ahol k
1
+k
2
+... +k
r
= n, akkor ezek klnbz sorrendjeit az n elem
(k
1
, k
2
, ..., k
r
) tpus ismtlses permutciinak nevezzk s ezek szmt gy jelljk: P
(k
1
,k
2
,...,k
r
)
n
.
Az I.1.2.1 Feladatban P
(2,3,2,1,1,1)
10
= 151 200. Krds: Mennyi P
(k
1
,k
2
,...,k
r
)
n
?
I.1.2.3. Ttel. Ha n 1, k
1
, k
2
, ..., k
r
1, ahol r 1, k
1
+k
2
+... +k
r
= n, akkor
P
(k
1
,k
2
,...,k
r
)
n
=
n!
k
1
! k
2
! k
r
!
.
Bizonyts. Tekintsnk egy tetszleges, rgztett ismtlses permutcit. Ha az ebben szerepl
k
1
szm egymssal egyenl elemet permutljuk, akkor nem kapunk j ismtlses permutcit.
Ugyanakkor, megklnbztetve ezeket az elemeket (pl. gy, hogy ms-ms sznnel jelljk ket),
ezeket k
1
!-flekppen permutlhatjuk. gy a rgztett ismtlses permutcibl k
1
! szm olyan
ismtlses permutcit kapunk, amelyre az n elem kzl az els k
1
elem klnbz, a tovbbi
k
2
elem egymssal egyenl s az elbbiektl klnbz,..., tovbbi k
r
elem egymssal egyenl s
az elbbiektl klnbz. Most megklnbztetve a k
2
szm azonos elemet, ezeket k
2
!-flekppen
permutlhatjuk. gy a rgztett ismtlses permutcibl k
1
! k
2
! szm olyan ismtlses permutcit
kapunk, amelyre az n elem kzl az els k
1
elem klnbz, a kvetkez k
2
elem egymstl s az
elbbiektl klnbz, a soron kvetkez k
3
elem egymssal egyenl s az elbbiektl klnbz,...,
tovbbi k
r
elem egymssal egyenl s az elbbiektl klnbz. Ugyangy folytatva vgl n klnbz
elemk
1
! k
2
! k
r
! klnbz permutcijhoz jutunk. gy a P
(k
1
,k
2
,...,k
r
)
n
szm ismtlses permutcibl
k
1
! k
2
! k
r
! P
(k
1
,k
2
,...,k
r
)
n
szm ismtls nlkli permutcihoz jutunk. Ezek a permutcik mind
klnbzek s minden permutcit megkapunk, ezrt P
n
=k
1
! k
2
! k
r
! P
(k
1
,k
2
,...,k
r
)
n
, azaz P
(k
1
,k
2
,...,k
r
)
n
=
=n! /(k
1
! k
2
! k
r
!).
A P
(k
1
,k
2
,...,k
r
)
n
szmokat polinomilis szmoknak vagy polinomilis egytthatknak is nevezzk,
mert ezek a polinomilis ttelben szerepl egytthatk, lsd ksbb. Ms jells:
_
n
k
1
,k
2
,...,k
r
_
=
=
_
k
1
+k
2
+...+k
r
k
1
,k
2
,...,k
r
_
. Figyeljk meg, hogy P
(1,1,...,1)
n
=P
n
, P
(k,1,1,...,1)
n
=
n!
k!
=
P
n
P
k
.
I.1.2.4. Feladat. Hnyfle klnbz sorrendje van n olyan elemnek, amelyek kzl k szm
egyenl s a tbbi nk is egyenl, de az elbbiektl klnbz?
Megolds. P
(k,nk)
n
=
n!
k! (nk)!
.
14 I.1. FEJEZET. PERMUTCIK, VARICIK, KOMBINCIK
I.1.3. Varicik
I.1.3.1. Feladat. Az 1, 2, 3, 4 szmok kzl vlasszunk ki kettt s rjuk fel ezeket az sszes
lehetsges sorrendben. Mennyi a lehetsgek szma?
Megolds. A kvetkezket kapjuk:
12 13 14 21 23 24 31 32 34 41 42 43
A lehetsgek szma 12.
I.1.3.2. Denci. Legyen adott n klnbz elem (n 1). Vlasszunk ki kzlk k elemet, ahol
1 k n s rjuk fel ezeket az sszes lehetsges sorrendben. Ezeket a sorrendeket az n elem
k-adosztly variciinak nevezzk. Jellje V
k
n
az n elem k-adosztly variciinak a szmt.
A I.1.3.1 Feladatban V
2
4
= 12. Krds: Mennyi V
k
n
?
I.1.3.3. Ttel. Ha 1 k n, akkor V
k
n
= n(n1)(n2) (nk+1) .
Bizonyts. A varicik kpzst tekinthetjk gy, hogy adott n elem (pl. az 1,2, ..., n szmok) s
adott k hely (cella), ahov a kivlasztott elemeket az sszes lehetsges sorrendben berjuk.
Ezek utn az I.1.1.4 Ttel els bizonytshoz hasonlan: Az els helyre (cellba) az n elem
kzl brmelyiket rhatjuk, ez n lehetsg, a msodik helyre a megmaradt n1 elem brmelyike
kerlhet, ez n1 lehetsg, tovbb, a harmadik elem a megmaradt n2 elem brmelyike lehet, ez
jabb n2 lehetsg, ... . Most a k-adik cellnl meg kell llnunk, az ide kerl elem megvlasztsra
n(k1) = nk+1 lehetsgnk van. Kapjuk, hogy V
k
n
=n(n1)(n2) (nk+1).
A fenti kplet jobb oldaln a tnyezk szma k. Ez a kplet gy is rhat:
V
k
n
=
n(n1)(n2) (nk+1)(nk)(nk1) 2 1
(nk)(nk1) 2 1
=
n!
(nk)!
,
teht V
k
n
=
n!
(nk)!
. Ha k = 0, akkor innen V
0
n
=
n!
n!
= 1, ami megfelel annak, hogy n elembl 0
szm elemet egyflekppen vlaszthatunk ki s permutlhatunk: gy, hogy egy elemet se vesznk.
Figyeljk meg, hogy V
1
n
=n, V
n
n
=P
n
=n!. Ha k >n, akkor nem lehet varicikat kpezni, ezrt
k >n esetn clszer hasznlni, hogy V
k
n
= 0.
A varicik pontosabb dencija a kvetkez:
I.1.3.4. Denci. Variciknak nevezzk egy vges halmaznak egy msik vges halmazba val
injektv lekpezseit. Rszletesebben: ha A egy k elem halmaz, B pedig egy n elem halmaz (n, k
1), akkor az f : AB injektv fggvnyeket n elem k-adosztly (ismtls nlkli) variciinak
nevezzk.
Ha k>n, akkor nincs ilyen injektv fggvny, ha pedig kn, akkor az ilyen injektv fggvnyek
szma V
k
n
.
Szoksos a kvetkez jells: ha x vals szm s k 1 termszetes szm, akkor
[x]
k
=x(x1)(x2) (xk+1).
gy 1 k n esetn V
k
n
= [n]
k
=n(n1)(n2) (nk+1).
I.1.4. ISMTLSES VARICIK 15
I.1.4. Ismtlses varicik
I.1.4.1. Feladat. Az 1, 2, 3, 4 szmok kzl vlasszunk ki kettt gy, hogy ugyanazt az elemet
ktszer is vehetjk s rjuk fel ezeket az sszes lehetsges sorrendben. Mennyi a lehetsgek szma?
Megolds. A kvetkezket kapjuk:
11 12 13 14 21 22 23 24 31 32 33 34 41 42 43 44
A lehetsgek szma 16.
I.1.4.2. Denci. Legyen adott n klnbz elem (n 1). Vlasszunk ki kzlk k elemet, ahol
k 1 gy, hogy ugyanazt az elemet tbbszr is vehetjk s rjuk fel ezeket az sszes lehetsges
sorrendben. Ezeket a sorrendeket az n elem k-adosztly ismtlses variciinak nevezzk.
Jellje V
k
n
az n elem k-adosztly ismtlses variciinak a szmt.
Az I.1.4.1 Feladatban V
2
4
= 16. Krds: Mennyi V
k
n
?
I.1.4.3. Ttel. Ha n, k 1, akkor V
k
n
=n
k
.
Bizonyts. Ugyangy, mint az I.1.3.3 Ttel bizonytsban, de most a k szm cella mindegyikbe
brmelyik elemet rhatjuk az n kzl. Kapjuk, hogy V
k
n
=n n n
. .
kszor
=n
k
.
Az ismtlses varicik a kvetkezkppen is denilhatk:
I.1.4.4. Denci. Ismtlses variciknak nevezzk egy vges halmaznak egy msik vges halmazba
val lekpezseit. Rszletesebben: ha A egy k elem halmaz, B pedig egy n elem halmaz (n, k1),
akkor az f : A B fggvnyeket n elem k-adosztly ismtlses variciinak nevezzk.
Ha n, k 1, akkor az f : A B fggvnyek szma n
k
.
Figyelem! Az ismtlses jelznek ms jelentse van a permutciknl s ms a variciknl.
Az ismtlses permutciknl ez arra vonatkozik, hogy a permutland elemek kztt ismtldek
(azonosak) vannak. Itt pontosan meg van adva, hogy az ismtld elemek a permutcikban
hnyszor fordulhatnak el.
Az ismtlses varicik esetn n klnbz elembl kell a varicikat kpezni gy, hogy mindegyik
elemet akrhnyszor felhasznlhatjuk.
Figyeljk meg azt is, hogy a permutcik a variciknak egy specilis esete (ha k = n),
ugyanakkor az ismtlses permutcik nem specilis esetei az ismtlses variciknak.
Az ismtlses varici fogalma ltalnosthat:
I.1.4.5. Feladat. Adott n doboz, bennk rendre k
1
, k
2
, . . . , k
n
darab pronknt klnbz trgy.
Mindegyik dobozbl kivlasztunk egy trgyat. Hnyfle kivlaszts lehetsges a rgztett sorrend
mellett ?
Megolds. A lehetsgek szma k
1
k
2
k
n
. Az els dobozbl ugyanis k
1
-fle trgyat vlaszthatunk.
Brmelyik trgyat is vlasztottuk, a msodik dobozbl val hzsnl a vlasztst k
2
-flekppen
folytathatjuk, gy az els kt trgy kivlasztsra k
1
k
2
lehetsgnk van. Ez a harmadik dobozbl
val hzssal k
3
-flekppen folytathat, s gy tovbb.
Ezt a szmtsi mdszert, amely a lehetsgek szma = rszlehetsgek szmainak szorzata
elven alapszik s amelyet a fentiekben mr tbbszr hasznltunk, szorzsi szablynak nevezzk.
16 I.1. FEJEZET. PERMUTCIK, VARICIK, KOMBINCIK
I.1.4.6. Feladat. Hny pozitv osztja van az 48 600 = 2
3
3
5
5
2
szmnak?
Megolds. 4 6 3 = 72. Ugyanis az adott szm brmely pozitv osztja 2
a
3
b
5
c
alak, ahol
0 a 3, 0 b 5, 0 c 2. Az a kitev megvlasztsra teht 4 lehetsg van, b-re 6, c-re 3.
ltalnosts: Adott az n=p
a
1
1
p
a
2
2
p
a
r
k
szm, ahol p
1
, p
2
, . . . , p
r
pronknt klnbz prmszmok.
Akkor n pozitv osztinak szma (n) = (a
1
+1)(a
2
+1) (a
r
+1).
Kombinatorikai feladatokban ms esetekben a lehetsgek szmt nem szorzssal, hanem
sszeadssal kapjuk a kvetkez sszeadsi szably szerint: sszes lehetsgek szma = az
egymst kizr eseteknek megfelel lehetsgek szmainak sszege. Gyakran egytt kell alkalmaznunk
a szorzsi szablyt s az sszeadsi szablyt.
I.1.4.7. Feladat. Tekintsk azokat a dominkat, amelyek mindkt feln a pontok szma 0-tl
8-ig terjed. Ezeket a pontok szmnak megfelelen gy azonosthatjuk: xy, ahol 0 x y 8.
a) Hny ilyen domin van?
b) Hnyflekppen lehet a 45 ilyen domin kzl kettt kivlasztani gy, hogy a kt domint
egyms mell lehessen tenni (azaz valamelyik felkn a pontok szma azonos)?
Megolds. a) Ha x = 0, akkor y rtkei 0,1,2, . . . ,8 lehetnek, ez 9 lehetsg. Ha x = 1, akkor
y rtkei 1,2, . . . ,8 lehetnek, ez 8 lehetsg, s gy tovbb, ha x = 8, akkor csak y = 8 lehet, ez 1
lehetsg. Az sszeadsi szably szerint a domink szma 9+8+. . . +1 = 45.
b) Vlasszunk egy domint. Ez lehet 1. eset: dupla domin, azaz 00, 11, 22, . . . ,88, ezek szma
9, vagy 2. eset: olyan domin, amelyre x <y, ezek szma 36.
Az 1. esetben a msodik domint 8-flekppen vlaszthatjuk, pl. 22 esetn vehetjk a 02, 12,
23, 24, 25, 26, 27, 28 jelzseket.
A 2. esetben 8+8 = 16-fle prt vlaszthatunk, pl. 27 lehetsges prjai : 02, 07, 12, 17, 22, 23,
24, 25, 26, 28, 37, 47, 57, 67, 77, 78.
A szorzsi szably szerint az 1. esetben 9 8 =72, a 2. esetben pedig 36 16 =576 a lehetsgek
szma. Az sszeadsi szably szerint az sszes lehetsgek szma 72+576 = 648.
Az elbbiekben klnbsget tettnk aszerint, hogy a kt domint milyen sorrendben hzzuk.
Minden sszeill domin-pr ktszer is szerepel, pl. 35 s 52 majd 52 s 35. Ha ezeket nem tekintjk
klnbzknek, akkor a lehetsgek szma az elbbi fele, azaz 324.
I.1.5. Kombincik
I.1.5.1. Feladat. Az 1, 2, 3, 4 szmok kzl vlasszunk ki kettt (kt klnbzt) s rjuk fel ezeket
gy, hogy nem vagyunk tekintettel a kivlasztott elemek sorrendjre. Mennyi a lehetsgek szma?
Megolds. Hat lehetsg van, ezek a kvetkezk:
12 13 14 23 24 34
I.1.5.2. Denci. Legyen adott n klnbz elem (n 1). Vlasszunk ki kzlk k elemet, ahol
1 k n s rjuk fel ezeket gy, hogy nem vagyunk tekintettel a kivlasztott elemek sorrendjre.
Ezeket az elemsorozatokat az n elem k-adosztly kombinciinak nevezzk. Jellje C
k
n
az n
elem k-adosztly kombinciinak a szmt.
Ha az 1,2, ..., n elemek k-adosztly kombinciit tekintjk, akkor a szmokat (ltalban)
nagysgrendi sorrendben rjuk.
Az I.1.5.1 Feladatban C
2
4
= 6. Krds: Mennyi C
k
n
?
I.1.5. KOMBINCIK 17
I.1.5.3. Ttel. Ha 1 k n, akkor C
k
n
=
V
k
n
P
k
, C
k
n
=
n(n1)(n2) (nk+1)
k!
.
Ha 0 k n, akkor C
k
n
=
n!
k! (nk)!
.
Bizonyts. Tekintsnk egy tetszleges, rgztett kombincit. Ha az ebben szerepl k elemet
permutljuk, akkor nem kapunk j kombincit. Ugyanakkor ezek n elemk-adosztly variciinak
tekinthetk. ly mdon a rgztett k-adosztly kombincibl k! szm k-adosztly varicit
kapunk, s gy a C
k
n
szm kombincibl k! C
k
n
szm varicihoz jutunk. Ezek a varicik mind
klnbzek s minden varicit megkapunk, ezrt V
k
n
=k! C
k
n
, azaz C
k
n
=V
k
n
/k! =V
k
n
/P
k
. A tbbi
kplet a V
k
n
-ra vonatkoz elbbi kpletekbl addik.
Ha k = 0, akkor innen C
0
n
=
n!
n!
= 1, ami megfelel annak, hogy n elembl 0 szm elemet
egyflekppen vlaszthatunk ki : gy, hogy egy elemet se vesznk.
I.1.5.4. Ttel. (Szimmetria-tulajdonsg) Ha 0 k n, akkor C
k
n
=C
nk
n
.
Bizonyts. Azonnali az I.1.5.3 Ttel utols kplete szerint. Mskpp: Az n elembl k elemet
kivlasztani ugyanazt jelenti, mint a tbbi nk elemet nem kivlasztani. Ezt annyiflekppen
lehet, ahnyflekppen az nk elemet ki tudjuk vlasztani.
Figyeljk meg, hogy minden n 1-re C
0
n
=C
n
n
= 1, C
1
n
=C
n1
n
=n, C
2
n
=C
n2
n
=
n(n1)
2
, C
3
n
=
= C
n3
n
=
n(n1)(n2)
6
,... . Ha k > n, akkor nem lehet kombincikat kpezni, ezrt k > n esetn
clszer hasznlni, hogy C
k
n
= 0.
A n elem k-adosztly kombincii szmnak ms jellse
_
n
k
_
, olvasd n alatt k. Teht
C
k
n
=
_
n
k
_
=
n!
k! (nk)!
, 0 k n ,
ezeket a szmokat binomilis szmoknak vagy binomilis egytthatknak is nevezzk, lsd ksbb
a binomilis ttelt.
Itt
_
n
k
_
=P
(k,nk)
n
, lsd I.1.2.4. Ez az egyenlsg kzvetlenl is belthat. Tekintsnk n elemet,
amelyek k-adosztly kombinciit kpezzk. rjunk mindegyik elem al 1-et vagy 0-t aszerint,
hogy kivlasztottuk a kombinci kpzsekor vagy sem. Pl. ha n=5, az elemek a, b, c, d, e s k =3,
akkor az a, c, d s a, d, e kombincik esetn legyen: 10110, ill. 10011. gy minden k-adosztly
kombincinak megfelel egy k szm 1-esbl s nk szm 0-bl ll ismtlses permutci, s
klnbz k-adosztly kombinciknak klnbz ilyen ismtlses permutcik felelnek meg.
A kvetkez denci is adhat:
I.1.5.5. Denci. Egy n elem halmaz k elem rszhalmazait n elem k-adosztly kombinciinak
nevezzk.
Egy n elem halmaz k elem rszhalmazainak a szma teht
_
n
k
_
. gy a k=0 elem rszhalmazok
szma
_
n
0
_
=1, ez az res halmaz (), a k =1 elem rszhalmazok szma
_
n
1
_
=n, ..., a k =n elem
rszhalmazok szma
_
n
n
_
= 1, ez az adott halmaz.
I.1.5.6. Ttel. Legyen n 1.
1) Egy n elem halmaz sszes rszhalmazainak a szma 2
n
.
2)
_
n
0
_
+
_
n
1
_
+
_
n
2
_
+... +
_
n
n
_
= 2
n
.
18 I.1. FEJEZET. PERMUTCIK, VARICIK, KOMBINCIK
Bizonyts. 1) A rszhalmazokat gy kapjuk, hogy az adott halmaz bizonyos elemeit kivlasztjuk
a rszhalmazba, a tbbit pedig nem. gy mind az n elemre kt lehetsg van: vagy kivlasztjuk,
vagy sem. gy a lehetsgek szma, s ezzel egytt a rszhalmazok szma 2 2 2
. .
nszer
= 2
n
.
2) Az 1) pont azonnali kvetkezmnye.
I.1.5.7. Feladat. Legyen A = 1,2, ..., k, B = 1,2, ..., n. Hny f : A B szigoran nvekv
fggvny ltezik?
Megolds. Legyen f(1) =a
1
B, f(2) =a
2
B,..., f(k) =a
k
B. Felttel : a
1
<a
2
<... <a
k
.
Ez csak akkor lehetsges, ha k n s ekkor a lehetsgek szma, teht az f : A B szigoran
nvekv fggvnyek szma ppen C
k
n
(a denci szerint).
Ennek alapjn az 1,2, ..., n elemek k-adosztly kombincii gy is denilhatk, mint az f :
: A B szigoran nvekv fggvnyek.
I.1.5.8. Feladat. Igazoljuk, hogy minden k 1-re k egymsutni egsz szm szorzata oszthat
k!-sal.
Megolds. Feltehetjk, hogy az adott szmok mind pozitvak, legyenek ezek (fordtott sorrendben)
n, n1, ..., nk+1, ahol nk. Akkor szorzatuk n(n1) (nk+1) =k! C
k
n
. Itt C
k
n
egsz szm
s kvetkezik, hogy n(n1) (nk+1) oszthat k!-sal.
Ennek kvetkezmnyeknt addik, hogy kt egymsutni egsz szm szorzata oszthat 2-vel,
hrom egymsutni egsz szm szorzata oszthat 6-tal, stb.
I.1.6. Ismtlses kombincik
I.1.6.1. Feladat. Az 1, 2, 3, 4 szmok kzl vlasszunk ki kettt gy, hogy ugyanazt az elemet
ktszer is vehetjk, de nem vagyunk tekintettel a kivlasztott elemek sorrendjre. Mennyi a lehetsgek
szma?
Megolds. A kvetkezket kapjuk:
11 12 13 14 22 23 24 33 34 44
A lehetsgek szma 10.
I.1.6.2. Denci. Legyen adott n klnbz elem (n 1). Vlasszunk ki kzlk k elemet,
ahol k 1 gy, hogy ugyanazt az elemet tbbszr is vehetjk s rjuk fel ezeket gy, hogy nem
vagyunk tekintettel a kivlasztott elemek sorrendjre. Ezeket az elemsorozatokat az n elem k-
adosztly ismtlses kombinciinak nevezzk. Jellje C
k
n
az n elem k-adosztly ismtlses
kombinciinak a szmt.
Ha az 1,2, ..., n elemek k-adosztly ismtlses kombinciit tekintjk, akkor a szmokat (ltalban)
nagysgrendi sorrendben rjuk gy, hogy ismtldsek is lehetnek.
Az I.1.6.1 Feladatban C
2
4
= 10. Krds: Mennyi C
k
n
?
I.1.6.3. Ttel. Ha n, k 1, akkor C
k
n
=C
k
n+k1
, C
k
n
=
n(n+1)(n+2) (n+k1)
k!
.
Ha n 1, k 0, akkor C
k
n
=
(n+k1)!
k! (n1)!
.
I.1.6. ISMTLSES KOMBINCIK 19
Bizonyts. Megmutatjuk, hogy bijektv lekpezs ltesthet n elemk-adosztly ismtlses kombincii
s n+k 1 elem k-adosztly (ismtls nlkli) kombincii kztt. Innen kvetkezni fog, hogy
C
k
n
= C
k
n+k1
.
Tekintsnk az 1,2, ..., n elemek egy tetszleges, rgztett a
i
1
, a
i
2
, ..., a
i
k
ismtlses kombincijt,
ahol 1 a
i
1
a
i
2
... a
i
k
n. Adjuk hozz az elemekhez rendre a 0,1,2, ..., k1 szmokat, azaz
legyen
a
i
1
, a
i
2
+1, ..., a
i
k
+(k1).
Ez az 1,2, ..., n+k1 elemeknek egy ismtls nlkli kombincija, mert itt 1 a
i
1
<a
i
2
+1 <
<... <a
i
k
+(k1) n+k1. Minden ilyen ismtls nlkli kombincit megkapunk s pontosan
egyszer. Fordtva, ha b
i
1
, b
i
2
, ..., b
i
k
az 1,2, ..., n+k 1 elemek egy ismtls nlkli kombincija,
akkor b
i
1
, b
i
2
1, ..., b
i
k
(k1) az 1,2, ..., n elemek egy ismtlses kombincija lesz.
A tbbi kplet a C
k
n
szmokra vonatkoz korbbi kpletekbl addik.
Ha k = 0, akkor innen C
0
n
=
n!
n!
= 1, ami megfelel annak, hogy n elembl 0 szm elemet
egyflekppen vlaszthatunk ki : gy, hogy egy elemet se vesznk.
Az n elem k-adosztly ismtlses kombincii szmnak ms jellse
n
k
). Teht
C
k
n
=
_
n
k
_
=
n(n1)(n2) (nk+1)
k!
,
C
k
n
=
_
n
k
_
=
n(n+1)(n+2) (n+k1)
k!
,
ahol a nevezk egyenlek, a szmllkban pedig mindkt esetben k egymsutni szm szorzata ll
n-tl kezdve lefel, illetve n-tl kezdve felfel.
I.1.6.4. Feladat. Hny olyan domin van, amelynek mindkt feln a pontok szma 0-tl 8-ig
terjed, lsd I.1.4.7 Feladat.
Megolds. A dominkat a pontok szmnak megfelelen xy-nal jelljk, ahol 0 x y 8.
A lehetsgek szma denci szerint C
2
9
=
910
2
= 45.
I.1.6.5. Feladat. Legyen A = 1,2, ..., k, B = 1,2, ..., n. Hny f : A B nvekv fggvny
ltezik?
Megolds. Legyen f(1) =a
1
B, f(2) =a
2
B,..., f(k) =a
k
B. Felttel : a
1
a
2
... a
k
.
A lehetsgek szma, teht az f : AB nvekv fggvnyek szma minden n, k 1 esetn ppen
C
k
n
(a denci szerint).
Ennek alapjn az 1,2, ..., n elemek k-adosztly ismtlses kombincii gy is denilhatk,
mint az f : A B nvekv fggvnyek.
Szoksos a kvetkez jells is: ha x vals szm s k 1 termszetes szm, akkor
[x]
k
= x(x+1)(x+2) (x+k1).
gy
C
k
n
=
[n]
k
k!
=
n(n+1)(n+2) (n+k1)
k!
.
20 I.1. FEJEZET. PERMUTCIK, VARICIK, KOMBINCIK
I.2. fejezet
A binomilis s a polinomilis ttel
I.2.1. A binomilis ttel
Az (a+b)
2
= a
2
+2ab +b
2
, (a+b)
3
= a
3
+3a
2
b +3ab
2
+b
3
kpletek ltalnostsaknt igazoljuk a
kvetkez ttelt.
I.2.1.1. Ttel. Ha a, b tetszleges komplex szmok s n 1 egsz szm, akkor
(a+b)
n
=
n

k=0
_
n
k
_
a
nk
b
k
.
Bizonyts. Itt (a +b)
n
= (a+b)(a+b) (a+b)
. .
nszer
. A szorzsok elvgzse rdekben az n zrjel
mindegyikbl vagy az a-t vagy a b-t kell vlasztani, ezeket ssze kell szorozni, majd a kapott
szorzatokat ssze kell adni. gy egy olyan sszeget kapunk, amelynek minden tagja a
nk
b
k
alak,
ahol 0 k n. Ez a tag annyiszor szerepel, ahnyszor az n szm b kzl k szm b-t vlasztunk
s ez ppen C
k
n
=
_
n
k
_
.
Kirva a tagokat (a+b)
n
kvetkez kifejtst kapjuk:
(a+b)
n
=
_
n
0
_
a
n
+
_
n
1
_
a
n1
b+
_
n
2
_
a
n2
b
2
+... +
_
n
n
_
b
n
.
Figyeljk meg, hogy a kifejtsben n+1 tag van, az a kitevi n-tl 0-ig cskkennek, a b kitevi
pedig 0-tl n-ig nvekednek. Az egytthatk a binomilis egytthatk (a binom grg eredet
sz, jelentse kt tag, ez az a+b kttag sszegre vonatkozik).
I.2.1.2. Ttel. Ha n 1, akkor
n

k=0
_
n
k
_
= 2
n
,
n

k=0
(1)
k
_
n
k
_
= 0 .
Bizonyts. A binomilis ttelben legyen a = b = 1, ill. a = 1, b =1.
A binomilis egytthatk sszegre vonatkoz sszefggst mr lttuk az I.1.5.6 Ttelben. A
msodik, a binomilis egytthatk vltakoz eljel sszegre vonatkoz kplet gy is rhat:
_
n
0
_
+
_
n
2
_
+
_
n
4
_
+... =
_
n
1
_
+
_
n
3
_
+
_
n
5
_
+... = 2
n1
,
21
22 I.2. FEJEZET. A BINOMILIS S A POLINOMILIS TTEL
teht rgztett n fels index mellett a pros als index binomilis egytthatk sszege egyenl
a pratlan als index binomilis egytthatk sszegvel, s egyenl 2
n1
-gyel, mert az sszes
binomilis egytthat sszege 2
n
.
A binomilis egytthatk egy msik fontos tulajdonsga a kvetkez:
I.2.1.3. Ttel. (Addicis kplet) Ha 1 k n, akkor
_
n
k
_
=
_
n1
k
_
+
_
n1
k1
_
.
Bizonyts. Az a
1
, a
2
, . . . , a
n
, halmazbl hnyflekppen vlaszthatunk ki k elemet? Egyrszt
_
n
k
_
-flekppen. Msrszt, rgztsnk egy elemet, pl. az a
n
-et. A kivlasztott elemek kztt a
n
vagy szerepel vagy sem. Ha szerepel, akkor az a
1
, a
2
, . . . , a
n1
halmazbl vlasztanunk kell mg
k 1 szm elemet, ez
_
n1
k1
_
-flekppen trtnhet. Ha nem szerepel, akkor az sszes elemet az
a
1
, a
2
, . . . , a
n1
halmazbl kell vlasztanunk. Ez
_
n1
k
_
-flekppen lehetsges. sszesen teht
_
n1
k1
_
+
_
n1
k
_
a lehetsgek szma.
Ez egy tipikus kombinatorikus bizonyts, ellenttben az I.2.1.2 Ttel elbbi bizonytsval,
amely algebrai bizonyts, ott nincs semmi szerepe a
_
n
k
_
binomilis egytthatk jelentsnek,
csak az algebrai tulajdonsgaikat hasznltuk ki. Termszetesen minden (hibt nem tartalmaz)
bizonyts helyes s j, de gyakran a kombinatorikus bizonytsok szebbek, jobban rvilgtanak
a tulajdonsg lnyegre. Ugyanakkor sokszor nehz ilyeneket tallni.
Az I.2.1.3 Ttel algebrai bizonytsa:
_
n1
k
_
+
_
n1
k1
_
=
(n1)!
k! (nk1)!
+
(n1)!
(k1)! (nk)!
=
(n1)! (nk+k)
k! (nk)!
=
=
(n1)! n
k! (nk)!
=
n!
k! (nk)!
=
_
n
k
_
.
Ez egy rekurzv kplet, amelynek segtsgvel kiszmthatk a binomilis egytthatk. Az I.2.1
tblzat a binomilis egytthatkat tartalmazza. A rekurzv kplet szerint itt minden bels szm
egyenl az elz sorban a szm felett ll s az attl balra ll kt szm sszegvel.
n
_
n
0
_ _
n
1
_ _
n
2
_ _
n
3
_ _
n
4
_ _
n
5
_ _
n
6
_
0 1
1 1 1
2 1 2 1
3 1 3 3 1
4 1 4 6 4 1
5 1 5 10 10 5 1
6 1 6 15 20 15 6 1
I.2.1. tblzat. Binomilis egytthatk
A binomilis egytthatk a I.2.2 tblzat szerint is megadhatk, amit Pascal-hromszgnek
neveznk. Ebben az elrendezsben minden bels szm egyenl a szm felett ll kt szm sszegvel.
Ennek alapjn a tblzat knnyen kiegszthet tovbbi sorokkal.
Mivel
_
n
k
_
=
_
n
nk
_
, a Pascal-hromszg soraiban a szlektl egyenl tvolsgra ll szmok
egyenlek.
I.2.1.4. Feladat. Adjuk meg a Pascal-hromszg kvetkez hrom sort.
I.2.1. A BINOMILIS TTEL 23
1
1 1
1 2 1
1 3 3 1
1 4 6 4 1
1 5 10 10 5 1
1 6 15 20 15 6 1
. . . . . . . . . . . . . . .
I.2.2. tblzat. Pascal-hromszg
I.2.1.5. Feladat. Adjuk meg (a+b)
7
, (a+b)
8
, (x
2
+1)
6
, (

x+2
3

y)
6
kifejtseit.
I.2.1.6. Feladat. Igazoljuk, hogy
n

k=1
k
_
n
k
_
=n 2
n1
, ahol n 1.
Megolds.
n

k=1
k
_
n
k
_
=
n

k=1
k
n!
k! (nk)!
= n
n

k=1
(n1)!
(k1)! (nk)!
= n
n

k=1
_
n1
k1
_
= n 2
n1
, a
binomilis egytthatk sszegre vonatkoz kplet szerint.
Mskpp: Legyen S(n)=
n

k=1
k
_
n
k
_
=1
_
n
1
_
+2
_
n
2
_
+...+(n1)
_
n
n1
_
+n
_
n
n
_
. A binomilis
egytthatk szimmetria-tulajdonsga miatt S(n)=n
_
n
0
_
+(n1)
_
n
1
_
+...+2
_
n
n2
_
+1
_
n
n1
_
.
sszeadva: 2S(n) = n(
_
n
0
_
+
_
n
1
_
+... +
_
n
n1
_
+
_
n
n
_
) = n 2
n
, ahonnan S(n) = n 2
n1
.
A binomilis kpletnek rvnyes a kvetkez ltalnostsa, amelyet ltalnostott binomilis
kpletnek neveznk:
(1+x)

= 1+x+
(1)
2
x
2
+... +
(1) (k+1)
k!
x
k
+... =

k=0
_

k
_
x
k
,
ahol , xR, [x[ <1 s
_

k
_
=
(1) (k+1)
k!
, k =0,1,2, ..., az ltalnostott binomilis
egytthatk. Ezt a kpletet itt nem bizonytjuk.
I.2.1.7. Feladat. Igazoljuk, hogy az ltalnostott binomilis egytthatkra is igaz a
_

k
_
=
_
1
k
_
+
_
1
k1
_
addicis kplet.
tmutats. Kzvetlen szmolssal.
I.2.1.8. Feladat. Adjuk meg az ltalnostott binomilis kpletet =1 esetn.
Megolds. Ha =1, akkor
_
1
k
_
= (1)
k
s azt kapjuk, hogy:
(1+x)
1
= 1x+x
2
x
3
+x
4
... =

k=0
(1)
k
x
k
.
24 I.2. FEJEZET. A BINOMILIS S A POLINOMILIS TTEL
I.2.2. A polinomilis ttel
A binomilis ttel egy ltalnostsa az (a
1
+a
2
+...+a
r
)
n
kifejtsre vonatkoz polinomilis ttel.
I.2.2.1. Ttel. Ha r 1, n 1 egsz szmok s a
1
, a
2
, ..., a
r
tetszleges komplex szmok, akkor
(a
1
+a
2
+... +a
r
)
n
=

k
1
,k
2
,...,k
r
0
k
1
+k
2
+...+k
r
=n
n!
k
1
! k
2
! k
r
!
a
k
1
1
a
k
2
2
a
k
r
r
,
ahol az sszeg a k
1
, k
2
, ..., k
r
0 szmok sszes olyan megvlasztsra vonatkozik, a sorrend gye-
lembevtelvel, amelyekre k
1
+k
2
+... +k
r
= n.
Bizonyts. Az (a
1
+a
2
+... +a
r
)
n
kifejezst egy n-tnyezs szorzatknt tekintve s minden tagot
minden taggal szorozva azt kapjuk, hogy
(a
1
+a
2
+... +a
r
)
n
=

1i
1
,i
2
,...,i
n
r
a
i
1
a
i
2
a
i
n
,
ahol i
1
, i
2
, ..., i
n
az 1,2, ..., r elemek n-edosztly ismtlses varicii. Itt minden tag a
k
1
1
a
k
2
2
a
k
r
r
alakra hozhat, ahol k
1
, k
2
, ..., k
r
0 s k
1
+k
2
+...+k
r
=n. Krds: Adott k
1
, k
2
, ..., k
r
esetn hny
ilyen tag van?
Az a
i
1
a
i
2
a
i
n
szorzatbl (a tnyezk felcserlsvel) akkor kapunk ilyen a
k
1
1
a
k
2
2
a
k
r
r
tagot,
ha az i
1
, i
2
, ..., i
n
kztt pontosan k
1
db 1-es, k
2
db 2-es,..., k
r
db r-es van. gy i
1
, i
2
, ..., i
n
az
1,2, ..., r ismtlses permutcii s szmuk P
(k
1
,k
2
,...,k
r
)
n
=
n!
k
1
!k
2
!k
r
!
.
Az egytthatk a polinomilis egytthatk (a polinom grg eredet sz, jelentse tbb
tag, ez az a
1
+a
2
+... +a
r
tbbtag sszegre vonatkozik).
I.2.2.2. Feladat. Adjuk meg (a+b+c)
3
, (a+b+c)
4
, (1+x+x
2
)
3
, (x+y +z +t)
3
kifejtseit.
I.2.2.3. Feladat. Igazoljuk, hogy a polinomilis egytthatk sszege:

k
1
,k
2
,...,k
r
0
k
1
+k
2
+...+k
r
=n
n!
k
1
! k
2
! k
r
!
=r
n
.
Megolds. A polinomilis ttelben legyen a
1
= a
2
=... =a
r
= 1.
I.2.3. A binomilis egytthatk tulajdonsgai
Vizsgljuk a binomilis egytthatkat. Ezek tovbbi tulajdonsgait rgztik a kvetkez ttelek.
I.2.3.1. Ttel.
1) (Elnyelsi tulajdonsg) Ha 1 k n, akkor
_
n
k
_
=
n
k
_
n1
k1
_
.
2) (Trinomilis alak) Ha 1 mk n, akkor
_
n
k
__
k
m
_
=
_
n
m
__
nm
km
_
.
I.2.3. A BINOMILIS EGYTTHATK TULAJDONSGAI 25
Bizonyts. 1)
_
n
k
_
=
n!
k!(nk)!
=
n
k

(n1)!
(k1)!(nk)!
=
n
k
_
n1
k1
_
.
2) Algebrai ton az
_
n
k
_
=
n!
k!(nk)!
kplet alapjn. Vgezzk el !
Kombinatorikus eljrs: Adott n f (szemly), akikbl egy k tag bizottsgot kell vlasztani,
majd a k fs bizottsg tagjai kzl egy m fs albizottsgot kell ltrehozni. Ez
_
n
k
__
k
m
_
-flekppen
tehet meg. Ugyanezt mskpp sszeszmolva: Elszr az n fbl kivlasztjuk az mtag albizottsgot,
majd a fennmarad nm szemly kzl kivlasztjuk azt a k m ft, akik a bizottsgnak az
albizottsgon kvli rszt kpezik. A lehetsgek szma:
_
n
m
__
nm
km
_
.
I.2.3.2. Ttel.
1) (Fels sszegzs) Ha 1 k n, akkor
_
k1
k1
_
+
_
k
k1
_
+
_
k+1
k1
_
+... +
_
n1
k1
_
=
_
n
k
_
.
2) (Prhuzamos sszegzs) Ha n, m0, akkor
_
m
0
_
+
_
m+1
1
_
+
_
m+2
2
_
+... +
_
m+n
n
_
=
_
m+n+1
n
_
.
Bizonyts. 1) Adjuk ssze az addicis kpletbl szrmaz kvetkez egyenlsgeket:
_
n
k
_
=
_
n1
k
_
+
_
n1
k1
_
_
n1
k
_
=
_
n2
k
_
+
_
n2
k1
_
_
n2
k
_
=
_
n3
k
_
+
_
n3
k1
_
......................................
_
k+1
k
_
=
_
k
k
_
+
_
k
k1
_
sszevons utn a bal oldalon csak az
_
n
k
_
els tag marad, a jobb oldal els oszlopban pedig
csak az 1 =
_
k
k
_
=
_
k1
k1
_
utols tag.
Mskpp, kombinatorikus ton: Az 1,2,3, ..., n halmaznak
_
n
k
_
szm k-elem rszhalmaza
van. Csoportostsuk ezeket a rszhalmazokat a legkisebb elemk szerint:
az 1 a legkisebb elem
_
n1
k1
_
szm rszhalmazban, mert az 1-hez a 2,3, ..., n szmok kzl
kell mg k1 szmot vlasztani,
a 2 a legkisebb elem
_
n2
k1
_
szm rszhalmazban, mert a 2-hz a 3,4, ..., n szmok kzl kell
mg k1 szmot vlasztani,
.......................
az nk+1 a legkisebb elem
_
n(nk+1)
k1
_
=
_
k1
k1
_
=1 szm rszhalmazban, itt az nk+1-et
kvet szmok az nk+2, ..., n lesznek.
2)
_
m
0
_
+
_
m+1
1
_
+
_
m+2
2
_
+... +
_
m+n
n
_
=
n

k=0
_
m+k
k
_
(1)
=
n

k=0
_
m+k
m
_
=
=
_
m
m
_
+
_
m+1
m
_
+... +
_
m+n
m
_
(2)
=
_
m+n+1
m+1
_
(3)
=
_
m+n+1
n
_
,
26 I.2. FEJEZET. A BINOMILIS S A POLINOMILIS TTEL
ahol a kvetkezket hasznltuk: (1) - szimmetria-tulajdonsg, (2) - fels sszegzs (a jelen ttel
1) pontja), (3) - szimmetria-tulajdonsg.
I.2.3.3. Feladat. Igazoljuk, hogy az 1,2,3, ..., n halmaz sszes k-elem rszhalmazai legkisebb
elemeinek a szmtani kzparnyosa
n+1
k+1
.
I.2.3.4. Ttel.
1) (Vandermonde-azonossg) Ha 0 r s r m, r n, akkor
_
m
0
__
n
r
_
+
_
m
1
__
n
r 1
_
+
_
m
2
__
n
r 2
_
+... +
_
m
r
__
n
0
_
=
_
m+n
r
_
.
2) Ha n 0, akkor
_
n
0
_
2
+
_
n
1
_
2
+
_
n
2
_
2
+... +
_
n
n
_
2
=
_
2n
n
_
.
Bizonyts. 1) Kombinatorikus eljrs: Legyen A egy m elem halmaz, B pedig egy n elem
halmaz gy, hogy AB = . Akkor AB szmossga m+n. Hny r elem rszhalmaza van
AB-nek? Egyrszt
_
m+n
r
_
. Msrszt, minden r elem rszhalmazt megkapunk gy, hogy sszes
lehetsges mdon vesszk A-nak egy k elem rszhalmazt, B-nek egy r k elem rszhalmazt
s kpezzk ezek unijt, ahol 0 k r. A lehetsgek szma ppen a 1) kpletben a bal oldali
sszeg.
2) A Vandermonde-azonossgban legyen m=n, r =n s hasznljuk a binomilis egytthatk
szimmetria-tulajdonsgt.
I.2.3.5. Feladat. Legyen 0 mn. Igazoljuk, hogy
n

k=m
_
k
m
__
n
k
_
=
_
n
m
_
2
nm
.
Megolds.
n

k=m
_
k
m
__
n
k
_
=
n

k=m
_
n
k
__
k
m
_
(1)
=
n

k=m
_
n
m
__
nm
km
_
=
_
n
m
_
n

k=m
_
nm
km
_
(2)
=
=
_
n
m
_
nm

j=0
_
nm
j
_
(3)
=
_
n
m
_
2
nm
,
ahol a kvetkezket hasznltuk: (1) - trinomilis alak, (2) - sszegzsi index csere: j =km , (3)
- a binomilis egytthatk sszegre vonatkoz kplet.
Foglaljuk ssze a binomilis egytthatk legfontosabb tulajdonsgait:
(1) Faktorilis kifejts (I.1.5.3 Ttel):
_
n
k
_
=
n!
k! (nk)!
, 0 k n.
(2) Szimmetria-tulajdonsg (I.1.5.4 Ttel):
_
n
k
_
=
_
n
nk
_
, 0 k n.
(3) Elnyelsi tulajdonsg (I.2.3.1 Ttel):
_
n
k
_
=
n
k
_
n1
k1
_
, 1 k n.
(4) Addicis kplet (I.2.1.3 Ttel):
_
n
k
_
=
_
n1
k
_
+
_
n1
k1
_
, 1 k n.
I.2.3. A BINOMILIS EGYTTHATK TULAJDONSGAI 27
(5) Trinomilis alak (I.2.3.1 Ttel):
_
n
k
__
k
m
_
=
_
n
m
__
nm
km
_
, 1 mk n.
(6) Binomilis ttel (I.2.1.1 Ttel): (a+b)
n
=
_
n
0
_
a
n
+
_
n
1
_
a
n1
b+
_
n
2
_
a
n2
b
2
+... +
_
n
n
_
b
n
,
n 1.
(7) Fels sszegzs (I.2.3.2 Ttel):
_
k1
k1
_
+
_
k
k1
_
+
_
k+1
k1
_
+... +
_
n1
k1
_
=
_
n
k
_
,
1 k n.
(8) Prhuzamos sszegzs (I.2.3.2 Ttel):
_
m
0
_
+
_
m+1
1
_
+
_
m+2
2
_
+... +
_
m+n
n
_
=
_
m+n+1
n
_
, n, m0.
(9) Vandermonde-azonossg (I.2.3.4 Ttel):
_
m
0
__
n
r
_
+
_
m
1
__
n
r 1
_
+
_
m
2
__
n
r 2
_
+... +
_
m
r
__
n
0
_
=
_
m+n
r
_
,
0 r, r m, r n.
(10) Binomilis egytthatk ngyzetsszege (I.2.3.4 Ttel):
_
n
0
_
2
+
_
n
1
_
2
+
_
n
2
_
2
+... +
_
n
n
_
2
=
_
2n
n
_
, n 0.
28 I.2. FEJEZET. A BINOMILIS S A POLINOMILIS TTEL
I.3. fejezet
Szitakpletek
Jellje [X[ az X vges halmaz elemeinek a szmt. Ha A s B vges halmazok, akkor
[AB[ =[A[ +[B[ [AB[.
Valban, a jobb oldalon [A[+[B[ felrsval ktszer szmoltuk a kzs elemeket, egyszer A-nl,
egyszer B-nl, ezrt le kell vonni a kzs elemek szmt, azaz [AB[-t. Hasonlkppen gondolhat
vgig, hogy ha A, B s C vges halmazok, akkor
[ABC[ =[A[ +[B[ +[C[ [AB[ [AC[ [BC[ +[ABC[.
Itt a hrom halmaz kzs elemeit tekintve, ezek szmt [A[ +[B[ +[C[ felrsval hromszor
hozzadtuk, majd hromszor levontuk ([AB[[AC[[BC[), az [ABC[ felrsval pedig
egyszer ismt hozzadtuk.
ltalnostva ezeket a kpleteket igazoljuk, hogy
I.3.1. Ttel. (Szitakplet, a tartalmazs s kizrs elve) Ha A
1
, A
2
, ..., A
r
vges halmazok, akkor
[A
1
A
2
... A
r
[ =
n

i=1
[A
i
[

1i<jr
[A
i
A
j
[ +

1i<j<kr
[A
i
A
j
A
k
[ ...+
+(1)
r1
[A
1
A
2
... A
r
[.
Bizonyts. Lssuk be, hogy az A
1
A
2
... A
r
halmaz minden elemt a jobb oldalon pontosan
egyszer szmoltuk. Tegyk fel, hogy egy tetszleges x elem az A
1
, A
2
, ..., A
r
halmazok kzl p
szmnak eleme (1 p r), a tbbinek nem. Feltehet (tjellve a halmazokat), hogy x A
1
,
x A
2
,..., x A
p
s x / A
p+1
,..., x / A
r
. Akkor x-et a kplet jobb oldaln m-szer szmoltuk, ahol
m=p
_
p
2
_
+
_
p
3
_
... +(1)
p1
_
p
p
_
,
de itt m= 1 az I.2.1.2 Ttel msodik kplete alapjn.
Megjegyezzk, hogy a szitakplet r-szerinti indukcival is igazolhat. Ez a kplet megadja az
A
1
, A
2
, ..., A
r
halmazok unijnak szmossgt, ha ismerjk az A
1
, A
2
, ..., A
r
halmazok elemszmt
s az sszes A
i
A
j
, A
i
A
j
A
k
, ... metszet elemszmt.
Legyenek A
1
, A
2
, ..., A
r
E. Krds: Mennyi az E-re vonatkoz E (A
1
A
2
... A
r
) =
=A
1
A
2
... A
r
kiegszt halmaz szmossga?
29
30 I.3. FEJEZET. SZITAKPLETEK
I.3.2. Ttel. (Szitakplet, msodik alak) Ha A
1
, A
2
, ..., A
r
E vges halmazok, akkor
[A
1
A
2
... A
r
[ =[E[
n

i=1
[A
i
[ +

1i<jr
[A
i
A
j
[

1i<j<kr
[A
i
A
j
A
k
[ +...+
+(1)
r
[A
1
A
2
... A
r
[.
Bizonyts. [A
1
A
2
... A
r
[ =[E[ [A
1
A
2
... A
r
[ s alkalmazzuk az elz Ttelt.
I.3.3. Feladat. Legyen n > 1, n = p
a
1
1
p
a
2
2
p
a
r
r
kanonikus alak termszetes szm. Jellje (n)
azoknak az a szmoknak a szmt, amelyekre 1 a n s (a, n) =1 (a s n relatv prmek), ez az
Euler-fle szmelmleti fggvny. Adjunk kpletet (n)-re!
Megolds. Legyen E = 1,2, ..., n, A
i
= a N : 1 a n, p
i
[ a, 1 i r. Akkor (n) =
= [A
1
A
2
... A
r
[. Itt, ha i < j < k, akkor A
i
A
j
= a : 1 a n, p
i
p
j
[ a, A
i
A
j
A
k
= a :
: 1 a n, p
i
p
j
p
k
[ a,..., s kapjuk, hogy [A
i
[ =
n
p
i
, [A
i
A
j
[ =
n
p
i
p
j
, [A
i
A
j
A
k
[ =
n
p
i
p
j
p
k
, ... .
Kvetkezik, hogy
(n) = n
_
1
r

i=1
1
p
i
+

1i<jr
1
p
i
p
j
... +(1)
k

1i
1
<...<i
k
r
1
p
i
1
p
i
k
+... +(1)
r
1
p
1
p
r
_
=
=n
_
1
1
p
1
_

_
1
1
p
r
_
.
I.3.4. Ttel. (Specilis szitakplet) Legyenek A
1
, A
2
, ..., A
r
E vges halmazok. Ha minden k
esetn (1kr) az A
1
, A
2
, ..., A
r
halmazok kzl brmely k klnbz metszete azonos elemszm,
akkor
[A
1
A
2
... A
r
[ =[E[
_
r
1
_
[A
1
[ +
_
r
2
_
[A
1
A
2
[
_
r
3
_
[A
1
A
2
A
3
[ +...+
+(1)
r
_
r
r
_
[A
1
A
2
... A
r
[.
Bizonyts. Azonnali a szitakplet msodik alakjbl.
I.3.5. Feladat. Legyen A egy k elem halmaz, B pedig egy n elem halmaz (n, k 1). Hny
f : A B szrjektv fggvny ltezik?
Megolds. Ha k<n, akkor nem ltezik f :AB szrjektv fggvny. Legyen kn. Feltehetjk,
hogy A=1,2, ..., k, B =1,2, ..., n. Legyen E az sszes f : AB fggvny halmaza, A
i
pedig
az olyan f : A B fggvnyeknek a halmaza, amelyekre i nem kpelem, ahol 1 i n. Akkor
az f : A B szrjektv fggvny halmaza A
1
A
2
... A
n
. Ha 1 i
1
< i
2
< ... < i
r
n, akkor
A
i
1
A
i
2
... A
i
r
azoknak az f : A B fggvnyeknek a halmaza, amelyekre i
1
, i
2
, ..., i
r
egyike
sem kpelem. Az ilyen fggvnyek szma (nr)
k
, lsd I.1.4.4. Az I.3.4 Ttel szerint kapjuk, hogy
[A
1
A
2
... A
n
[ =n
k
+
n

r=1
(1)
r
_
n
r
_
(nr)
k
=
n1

r=0
(1)
r
_
n
r
_
(nr)
k
,
ahol elegend r = 0-tl r =n1-ig venni a tagokat, mert r = n-re az utols tag 0.
Az f : 1,2, ..., k 1,2, ..., n szrjektv fggvnyek szma teht
s
k,n
=n
k

_
n
1
_
(n1)
k
+
_
n
2
_
(n2)
k
... +(1)
n2
_
n
n2
_
2
k
+(1)
n1
_
n
n1
_
.
31
Ha k < n, akkor nincs szrjektv fggvny, ezrt s
k,n
= 0 s kvetkezik, hogy a fenti sszeg
rtke nulla.
Ha n = k, akkor s
n,n
az f : 1,2, ..., n 1,2, ..., n bijektv fggvnyek szma, ami n!, s a
kvetkez egyenlsget kapjuk:
n! = n
n

_
n
1
_
(n1)
n
+
_
n
2
_
(n2)
n
... +(1)
n1
_
n
n1
_
.
Az s
k,n
-re adott kplet gy is rhat: s
k,n
=
n

j=1
(1)
nj
_
n
j
_
j
k
= (1)
n
n

j=1
(1)
j
_
n
j
_
j
k
, ahol a
j =nr helyettestst vgeztk s hasznltuk, hogy
_
n
nj
_
=
_
n
j
_
.
I.3.6. Feladat. Legyen egy n-edfok permutci. Azt mondjuk, hogy i a xpontja, ha (i) =i
(i a helyn marad). Hny olyan n-edfok permutci van, amelynek nincs xpontja (egy szm se
marad a helyn)?
Megolds. Legyen A
i
azoknak a permutciknak a halmaza, amelyeknek i xpontja, azaz
(i) = i, ahol 1 i n. Akkor a xpont nlkli permutcik szma: D
n
=[A
1
A
2
... A
n
[. Ha
1 i
1
<i
2
<... <i
r
n tetszleges szmok, akkor A
i
1
A
i
2
... A
i
r
azoknak a permutciknak
a halmaza, amelyeknek i
1
, i
2
, ...i
r
xpontjai, azaz (i
1
) = i
1
, (i
2
) = i
2
,..., (i
r
) = i
r
, s gy [A
i
1

A
i
2
... A
i
r
[ = (nr)!.
A specilis szitakplet szerint kapjuk, hogy
D
n
=n!
_
n
1
_
(n1)! +
_
n
2
_
(n2)!
_
n
3
_
(n3)! +... +(1)
n
_
n
n
_
0! =
=n!
_
1
1
1!
+
1
2!
... +(1)
n
1
n!
_
.
Megjegyezzk, hogy itt L = lim
n
D
n
n!
=
1
e
, ahol e 2,718 a termszetes logaritmus alapszma.
Az L=
1
e
0,367 rtk annak a valsznsgnek tekinthet, hogy egy vletlenszeren kivlasztott
permutci xpont nlkli legyen.
A fenti kpletbl azonnali a kvetkez rekurzi: D
n
= nD
n1
+(1)
n
, ahol n 1 s D
0
= 1
(megllapods szerint). Innen meghatrozhatk D
n
egymst kvet rtkei : D
1
=0, D
2
=1, D
3
=2,
D
4
= 9, D
5
= 44, D
6
= 265,... .
I.3.7. Feladat. Hny olyan n-edfok permutci van, amelynek pontosan r xpontja van (0
r n)?
Megolds. A vlaszt a D
n,r
=
_
n
r
_
D
nr
kplet adja, ugyanis az r xpont
_
n
r
_
-flekppen
vlaszthat meg, a tbbi nr elem pedig egy olyan (nr)-edfok permutcit hatroz meg,
amely xpont nlkli s ezek szma D
nr
. Alkalmazzuk ezek utn a szorzsi szablyt.
Megjegyzs. Csoportostsuk az n-edfok permutcikat aszerint hogy hny xpontjuk van. Az
sszes n-edfok permutci szma n! s a kvetkez kpletet kapjuk:
n! =
n

r=0
D
n,r
=
n

r=0
_
n
r
_
D
nr
.
32 I.3. FEJEZET. SZITAKPLETEK
I.4. fejezet
sszeszmllsi feladatok
I.4.1. sszeszmllsi feladatok
I.4.1.1. Feladat. Adott n klnbz trgy s r doboz (n, r1). Helyezzk a trgyakat a dobozokba
gy, hogy az 1. dobozba k
1
trgy kerljn, a 2. dobozba k
2
trgy,..., az r-edik dobozba k
r
trgy, ahol
k
1
+k
2
+...+k
r
=n. Hnyflekppen lehetsges ez? (Az egyes dobozokban szmt a trgyak sorrendje.)
Megolds. Az 1. dobozba
_
n
k
1
_
-flekppen vlaszthat meg a k
1
trgy (a sorrend nem szmt).
Ezutn a 2. dobozba a megmaradt nk
1
kzl brmelyiket vlaszthatjuk
_
nk
1
k
2
_
-flekppen. A
3. dobozba a megmaradt nk
1
k
2
kzl brmelyiket vlaszthatjuk
_
nk
1
k
2
k
3
_
-flekppen, stb. A
lehetsgek szma gy
_
n
k
1
__
nk
1
k
2
__
nk
1
k
2
k
3
_

_
nk
1
k
2
... k
r1
k
r
_
=
=
n!
k
1
! (nk
1
)!

(nk
1
)!
k
2
! (nk
1
k
2
)!

(nk
1
k
2
)!
k
3
! (nk
1
k
2
k
3
)!

(nk
1
k
2
... k
r1
)!
k
r
! 0!
=
=
n!
k
1
! k
2
! k
r
!
,
ami nem ms, mint n elem (k
1
, k
2
, ..., k
r
) tpus ismtlses permutciinak P
(k
1
,k
2
,...,k
r
)
n
szma.
Ez kzvetlenl is belthat: Szmozzuk meg az n trgyat s a sorszmokat rjuk le egyms
mell. rjunk 1-eseket azon sorszmok al, amelyek az 1. dobozba kerltek (k
1
db 1-es lesz),
rjunk 2-eseket azon sorszmok al, amelyek a 2. dobozba kerltek (k
2
db 2-es), ..., rjunk r-
eseket azoknak a trgyaknak a sorszmai al, amelyek az r-edik dobozba kerltek (k
r
db r-es).
gy egy (k
1
, k
2
, ..., k
r
) tpus ismtlses permutcit kapunk. Fordtva, minden ilyen ismtlses
permutcinak megfelel egy dobozokba oszts.
Pl. n = 10, k
1
= 2, k
2
= 4, k
3
= 3, k
4
= 1 esetn:
1 2 3 4 5 6 7 8 9 10
2 1 2 3 1 2 3 2 3 4
vagy
1 2 3 4 5 6 7 8 9 10
1 4 2 3 2 2 2 3 3 1
, stb.
I.4.1.2. Feladat. Hnyflekppen lehet n egyforma trgyat k szmozott dobozba helyezni gy,
hogy nem kell felttlen minden dobozba trgyakat tenni (n, k 1)? (Az egyes dobozokban itt nem
szmt a trgyak sorrendje.)
Megolds. Az n = 5, k = 3 esetben tekintsnk 5 egyforma pontot (labdt): .
33
34 I.4. FEJEZET. SSZESZMLLSI FELADATOK
Az elhelyezsi lehetsgeket adjuk meg [ elvlasztjelekkel, pl. [ [ azt jelenti, hogy az 1.
dobozba 1 trgy kerl, a 2. dobozba 3 trgy, a 3. dobozba 1 trgy kerl, ez az (1,3,1) rendezett
szmhrmassal is jellemezhet. Ms plda: [[ azt jelenti, hogy az 1. dobozba 2 trgy kerl,
a 2. dobozba nem kerl trgy, a 3. dobozba 3 trgy kerl, azaz (2,0,3).
Kt elvlasztjel is kerlhet egyms mell, s [ jel llhat a legelejn, vagy a legvgn. Kt [
jelre van szksg, amelyek elvlasztjk a pontokat. A pontokkal egytt ez 5+2 = 7 jel, amelyeket
tetszlegesen permutlhatunk. A megoldsok szma gy P
(5,2)
7
=
7!
5!2!
= 21.
ltalnosan k 1 db [ jelre van szksg, az n db ponttal egytt ez n+k 1 jel, amelyeket
tetszlegesen permutlhatunk. A megoldsok szma
P
(n,k1)
n+k1
=
(n+k1)!
n! (k1)!
=
_
n+k1
n
_
=
_
n+k1
k1
_
.
Mskpp: Annyi lehetsg van, ahnyflekppen az n+k 1 pozci kzl kivlaszthat az
a k 1, ahova az elvlasztjelek kerlnek (illetve az az n hely, ahov a labdk kerlnek), s ez
kombincik dencija szerint
_
n+k1
k1
_
=
_
n+k1
n
_
.
I.4.1.3. Feladat. Legyenek n, k1. Hny olyan megoldsa van az x
1
+x
2
+...+x
k
=n egyenletnek,
ahol x
1
, x
2
, ..., x
k
0 egsz szmok s tekintettel vagyunk a sorrendre is?
Megolds. Pl. n = 5, k = 3 esetn az x
1
+x
2
+x
3
= 5 egyenlet ilyen megoldsai (x
1
, x
2
, x
3
) =
=(0,0,5), (0,5,0), (5,0,0), (0,1,4), (0,4,1), (1,0,4), (1,4,0), (4,0,1), (4,1,0), (0,2,3), (0,3,2), (2,0,3), (2,3,0),
(3,0,2), (3,2,0), (1,1,3), (1,3,1), (3,1,1), (1,2,2), (2,1,2), (2,2,1), a megoldsok szma 21.
Lthat, hogy ez ugyanaz a problma, mint az I.4.1.2 Feladatbeli. Ha (x
1
, x
2
, ..., x
k
) jelli a
megoldsokat, akkor ppen x
1
, x
2
, ..., x
k
0 lesznek az egyes dobozokba kerl trgyak darabszmai.
A vlasz teht: az (x
1
, x
2
, ..., x
k
) megoldsok szma
_
n+k1
n
_
.
I.4.1.4. Feladat. Hnyflekppen lehet n egyforma trgyat k szmozott dobozba helyezni gy,
hogy minden dobozba kerljn legalbb egy trgy (n k 1)?
Megolds. Az n=5, k =3 esetnl maradva tekintsnk 5 egyforma pontot (labdt): . A
megoldsokat adjuk meg most is [ elvlasztjelekkel (lsd I.4.1.2 Feladat), pl. ha az 1. dobozba 1
trgy kerl, a 2. dobozba is 1 trgy, a 3. dobozba 3 trgy kerl, azaz (1,1,3), akkor ennek megfelel :
[ [ , (2,2,1)-nek pedig megfelel [ [ . Itt kt elvlasztjel nem kerlhet egyms mell,
s [ jel nem llhat a legelejn s a legvgn.
Annyi megolds van, ahnyflekppen a pontok kztti 4 helyre a 2 elvlasztjel beilleszthet,
teht a 4 hely kzl kell kettt kivlasztani a sorrendre val tekintet nlkl, ez a szm C
2
4
=
_
4
2
_
=6.
ltalnosan az n pont kztti n1 helyre kell a k1 elvlasztjelet beilleszteni, teht az n1
hely kzl kell k1-et kivlasztani a sorrendre val tekintet nlkl, ez a szm C
k1
n1
=
_
n1
k1
_
.
Mskpp: Visszavezetjk az I.4.1.2 Feladatra. Tegynk elszr minden dobozba egy-egy trgyat.
Akkor a megmaradt nk trgyat kell mg a k dobozba tenni gy, hogy nem kell felttlen minden
dobozba tovbbi trgyakat tenni. Az I.4.1.2 Feladat eredmnyt alkalmazva (n helyett nk-ra)
kapjuk, hogy a lehetsgek szma
_
nk+k1
k1
_
=
_
n1
k1
_
feltve, hogy n k.
Megjegyzs. Ha minden dobozba legalbb r trgynak kell kerlnie, akkor a lehetsgek szma
_
nk(r1)1
k1
_
. Valban, elbb minden dobozba tegynk r trgyat, majd osszuk szt a megmaradt
nkr trgyat. Az I.4.1.2 Feladat szerint a lehetsgek szma
_
nkr+k1
k1
_
=
_
nk(r1)1
k1
_
. Ha r =1,
akkor az I.4.1.4 Feladat felttelei teljeslnek.
I.4.1.5. Feladat. Legyenek n, k1. Hny olyan megoldsa van az x
1
+x
2
+...+x
k
=n egyenletnek,
ahol x
1
, x
2
, ..., x
k
1 egsz szmok s tekintettel vagyunk a sorrendre is?
I.4.1. SSZESZMLLSI FELADATOK 35
Megolds. Pl. n = 5, k = 3 esetn az x
1
+x
2
+x
3
= 5 egyenlet ilyen megoldsai (x
1
, x
2
, x
3
) =
= (1,1,3), (1,3,1), (3,1,1), (1,2,2), (2,1,2), (2,2,1), a megoldsok szma 6.
Lthat, hogy ez ugyanaz a problma, mint az I.4.1.4 Feladatbeli. Ha (x
1
, x
2
, ..., x
k
) jelli a
megoldsokat, akkor ppen x
1
, x
2
, ..., x
k
1 lesznek az egyes dobozokba kerl trgyak darabszmai.
gy a vlasz: az (x
1
, x
2
, ..., x
k
) megoldsok szma
_
n1
k1
_
, ahol n k. Ha n < k, akkor nincs
megolds (a megoldsok szma nulla).
Mskpp: Az egyenlet gy rhat: (x
1
1)+(x
2
1)+...+(x
k
1) =nk, ahol x
i
10 minden
i-re. Az I.4.1.3 Feladat szerint (n helyett nk-t rva) a megoldsok szma
_
n1
k1
_
.
Megjegyzs. Jellje az I.4.1.5 Feladatbeli egyenlet megoldsainak szmt c
k
(n), ahol c
k
(n) =
=
_
n1
k1
_
, itt k rgztett. Az egyenlet sszes x
1
, x
2
, ..., x
k
1 egsz szm megoldsainak a szma,
tekintettel a sorrendre, ahol k vltoz:
c
n
=
n

k=1
c
k
(n) =
n

k=1
_
n1
k1
_
= 2
n1
.
I.4.1.6. Feladat. Hnyflekppen lehet n klnbz trgyat k szmozott dobozba helyezni a
sorrend gyelembevtelvel (n, k 1)?
Megolds. Az I.4.1.2 Feladathoz kpest az a vltozs, hogy most nem n egyforma trgyat,
hanem n klnbz trgyat kell a k dobozba tenni.
A trgyakat jellje
1

2
...
n
, ezek most klnbzk. Az elhelyezsi lehetsgeket k1 elvlasztjellel
tudjuk megadni, pl. n=5, k=3 esetn
2

3
[
1
[
4
, vagy
4

2
[[
1

3
. Ezt az n+k1 jelet tetszlegesen
permutlhatjuk. A megoldsok szma gy P
(1,1,...,1,k1)
n+k1
=
(n+k1)!
(k1)!
=k(k+1)(k+2) (k+n1).
Ha n = 5, k = 3, akkor a lehetsgek szma 3 4 5 6 7 = 2520.
Mskpp: Az elhelyezst kt lpsben vgezzk. 1. lps: Sorbatesszk a trgyakat, erre P
n
=n!
lehetsg van. 2. lps: A trgyakat egyformknak tekintve a dobozokba tesszk, a lehetsgek
szma a I.4.1.2 Feladat szerint
_
n+k1
k1
_
. A szorzsi szably szerint a elhelyezsek szma n!
_
n+k1
k1
_
=
=
(n+k1)!
(k1)!
.
Megjegyzs. Az eredmny gy is megadhat: [k]
n
=k(k+1)(k+2) (k+n1).
I.4.1.7. Feladat. Adott n A-tpus klnbz trgy s k B-tpus klnbz trgy (n, k 1).
Hnyfle sorrendje lehetsges ezeknek gy, hogy kt B-tpus trgy nem kerlhet egyms mell?
Megolds. Elszr helyezzk el egyms mell az n db A-tpus trgyat. Az elhelyezsek
szma P
n
=n!. Ezutn a k db B-tpus trgy szmra n+1 hely van, ugyanis az A-tpusak kz
lehet tenni ket (a legelejre s a legvgre is lehet tenni). Az n+1 helybl kell k szm helyet
kivlasztani gy, hogy a sorrend is szmt. A lehetsgek szma V
k
n+1
, feltve, hogy k n+1. A
vlasz: P
n
V
k
n+1
=
n!(n+1)!
(nk+1)!
, ha k n+1. Ha k >n+1, akkor a ez a szm 0.
I.4.1.8. Feladat. Adott n A-tpus egyforma trgy s k B-tpus egyforma trgy (n, k 1).
Hnyfle sorrendje lehetsges ezeknek gy, hogy kt B-tpus trgy nem kerlhet egyms mell?
Megolds. Pl. hnyflekppen lehet sorba rendezni n db nullt (A-tpus) s k db egyest (B-
tpus) gy, hogy kt egyes ne kerljn egyms mell?
1. md. Hasonlan, mint az I.4.1.7 Feladat megoldsa, de most a sorrend nem szmt, mivel
egyforma elemekrl (trgyakrl) van sz. Elszr helyezzk el az n nullt. Ekkor a k db egyes
szmra n+1 hely van, a nullk kz lehet tenni ket (a legelejre s a legvgre is lehet tenni).
Pl. n = 5, k = 3 esetn: 10010100, 01001001, 00010101, stb. Az n+1 helybl kell k szm helyet
kivlasztani gy, hogy a sorrend nem szmt. A lehetsgek szma C
k
n+1
=
_
n+1
k
_
, feltve, hogy
k n+1. Ha n = 5, k = 3, akkor ez
_
6
3
_
= 20. Ha k >n+1, akkor a lehetsgek szma 0.
36 I.4. FEJEZET. SSZESZMLLSI FELADATOK
2. md. Elszr a k egyes kz berunk k1 nullt, gy egy ilyen sorozatot kapunk: 101010...10,
majd a tbbi nk +1 nullt tetszlegesen beillesztjk gy, hogy a legelejre s a legvgre is
tehetnk. Teht nk+1 nullt k+1 helyre tesznk ismtlssel : C
nk+1
k+1
=C
nk+1
k+1+nk+11
=C
nk+1
n+1
=
=
_
n+1
nk+1
_
=
_
n+1
k
_
. Pl. n = 5, k = 3-ra elbb a k = 3 egyes kz a k 1 = 2 nullt berva 10101,
majd mg nk+1 = 3 nullt beillesztve: 10010010, 01001010, 01001001, stb.
I.4.1.9. Feladat. a) A knyvespolcon 15 klnbz knyv van. Hnyflekppen lehet kzlk
kivlasztani 7 knyvet gy, hogy ezek kztt ne legyenek egyms mellett llk?
b) Hnyflekppen lehet n knyv kzl k knyvet kivlasztani gy, hogy ezek kztt ne legyenek
egyms mellett llk?
Megolds. a) Alkalmazzuk az I.4.1.8 Feladat eredmnyt. A kivlasztott knyvhz 1-est
rendelnk, a tbbihez 0-t, ahol kt 1-es nem lehet egyms mellett. Teht k = 7 db 1-est s n = 8
db 0-t kell elhelyeznnk az adott mdon. A keresett rtk:
_
n+1
k
_
=
_
9
7
_
=
_
9
2
_
= 36.
b) A keresett rtk
_
nk+1
k
_
. Ez akkor nem nulla, ha k nk+1, azaz 2k1 n, teht ekkor
lehet n knyv kzl k knyvet az adott mdon kivlasztani.
I.4.2. Egsz szmok partcii
Ebben a szakaszban azt vizsgljuk, hogy egy n 1 egsz szm hnyflekppen rhat fel pozitv
egszek sszegeknt, ill. k db pozitv egsz sszegeknt, ahol k rgztett, gy, hogy nem vagyunk
tekintettel a tagok sorrendjre.
Pldul : 1 = 1, 2 = 2 = 1+1, 3 = 3 = 2+1 = 1+1+1,
4 = 4 = 3+1 = 2+2 = 2+1+1 = 1+1+1+1,
5 = 5 = 4+1 = 3+2 = 3+1+1 = 2+2+1 = 2+1+1+1 = 1+1+1+1+1.
I.4.2.1. Denci. Az n 1 egsz szm partcijnak neveznk minden n =
1
+
2
+... +
k
ellltst, ahol
1

2
...
k
1 egszek, jells: = (
1
,
2
, ...,
k
). Azt mondjuk, hogy ez
egy k rszre (tagra) val partci. Legyen p(n) az n szm sszes partciinak szma s p
k
(n) az n
olyan partciinak szma, amelyekben a tagok szma k.
A fentiek szerint p(1) =1, p(2) =2, p(3) =3, p(4) =5, p(5) =7. Tovbb pl. p
1
(5) =1, p
2
(5) =2,
p
3
(5) = 2, p
4
(5) = 1, p
5
(5) = 1. Clszer a (*) p
0
(0) = 1, p
k
(0) = 0, k 1 megllapods.
Rgtn addik, hogy minden n-re p
1
(n) = 1 (ahol n =n), p
n
(n) = 1 (ahol n = 1+1+... +1) s
p(n) = p
1
(n)+p
2
(n)+... +p
n
(n).
Az I.4.1.5 Feladat megoldsban lttuk, hogy ha tekintettel vagyunk a sorrendre is, akkor az
n=x
1
+x
2
+...+x
k
, x
1
, ..., x
k
1 ellltsok szma
_
n1
k1
_
. Kvetkezik, hogy p
k
(n)
1
k!
_
n1
k1
_
. Nem
ltezik explicit kplet p(n)-re.
A p
k
(n) fggvnyre vonatkozik a kvetkez rekurzi:
I.4.2.2. Ttel. Ha n k 2, akkor p
k
(n) = p
k1
(n1)+p
k
(nk) .
Bizonyts. Legyen egy k rszre val partci. Vizsgljuk p
k
(n)-et aszerint, hogy az utols
k
rsz 1 vagy 1-nl nagyobb.
Ha
k
=1, akkor n-nek ugyanannyi partcija van k rszre, mint amennyi partcija van n1-
nek k1 rszre. Ha
k
> 1, akkor minden rsz nagyobb, mint 1, gy minden rszbl 1-et kivonva
az nk egy k rszre val partcijt kapjuk.
I.4.2. EGSZ SZMOK PARTCII 37
n p(n) p
1
(n) p
2
(n) p
3
(n) p
4
(n) p
5
(n) p
6
(n) p
7
(n) p
8
(n)
1 1 1
2 2 1 1
3 3 1 1 1
4 5 1 2 1 1
5 7 1 2 2 1 1
6 11 1 3 3 2 1 1
7 15 1 3 4 3 2 1 1
8 22 1 4 5 5 3 2 1 1
I.4.1. tblzat. p(n) s p
k
(n) rtkei
Ha k =n, akkor p
n
(n) = p
n1
(n1)+p
n
(0) addik, azaz 1 = 1+p
n
(0), ami igaz (*) alapjn.
Az I.4.1 tblzat a p(n) s p
k
(n) rtkeket tartalmazza, ahol k n 8.
A partcik egy lehetsges reprezentlsa az n. Ferrers-diagramokkal trtnik. Pl. a 10 =
= 5+3+1+1 partcit gy adjuk meg, hogy 5 pont kerl az els sorba, 3 pont a msodik sorba,
1 pont a harmadikba s 1 pont a negyedikbe:

Ennek a partcinak a konjugltja a 10 =4+2+2+1+1, amelynek Ferrers-diagramjt gy


kapjuk, hogy az elbbi diagramot tkrzzk a ftlra (az y =x egyenesre). Az (5,3,1,1) partci
konjugltja teht a (4,2,2,1,1), amelynek diagramja a kvetkez:


A diagramok segtsgvel belthat:


I.4.2.3. Ttel. Az n szm olyan partciinak szma, amelyekben k a legnagyobb rsz, egyenl az
n szm k rszre val partciinak szmval.
38 I.4. FEJEZET. SSZESZMLLSI FELADATOK
I.5. fejezet
Kombinatorika a geometriban
Azt mondjuk, hogy n egyenes ltalnos helyzet a skban, ha brmely kett metszi egymst, de
brmely hromnak nincs kzs pontja. Hasonlan, n sk ltalnos helyzet a trben, ha brmely
hromnak van kzs pontja, brmely ngynek viszont nincs.
I.5.1. Feladat. Hny rszre osztja a skot n ltalnos helyzet egyenes?
Megolds. Jellje E
n
ezt a szmot. Itt E
1
= 2, E
2
= 4, E
3
= 7. Rajzoljunk meg n egyenest,
amelyek a skot E
n
rszre osztjk. Az (n+1)-edik egyenest meghzva, ez az elbbi n egyenes
mindegyikt metszi s az n metszspont az (n+1)-edik egyenest (n+1)-rszre osztja. Minden
rsz egy skbeli tartomnyt kettvg, teht gy a skrszek szma (n+1)-gyel nvekszik. Kapjuk
a kvetkez rekurzit:
E
n+1
=E
n
+(n+1), n 1.
Pldul, ha n = 3, akkor adottak az e
1
, e
2
, e
3
egyenesek s rajzoljunk meg jabb e
4
egyenest,
amely hrom (klnbz) pontban metszi az elz egyeneseket, lsd az I.5.1 brt. Ezek a metszspontok
e
4
-et ngy rszre osztjk. Mindegyik rsz egy skbeli tartomnyt kettvg, teht a skrszek szma
4-gyel nvekszik: E
4
= E
3
+4, E
4
= 7+4 = 11.
I.5.1. bra. Egyenesek a skban
sszeadve az E
2
= E
1
+2, E
3
= E
2
+3, E
4
= E
3
+4, ..., E
n1
= E
n2
+(n1), E
n
= E
n1
+n
39
40 I.5. FEJEZET. KOMBINATORIKA A GEOMETRIBAN
egyenleteket kapjuk, hogy E
n
=E
1
+(2+3+... +(n1)+n), ahonnan E
n
= 1+
n(n+1)
2
,
E
n
=
n
2
+n+2
2
, n 1.
Megjegyzs. E(n) gy is rhat: E(n) = 1+
_
n+1
2
_
, vagy E(n) =
_
n
2
_
+
_
n
1
_
+
_
n
0
_
.
I.5.2. Feladat. Hny rszre osztja a teret n ltalnos helyzet sk?
Megolds. Jellje S
n
a keresett szmot. Itt S
1
=2, S
2
=4. Tekintsnk n skot, amelyek a teret
S
n
rszre osztjk. Egy (n+1)-edik skot az elbbi skok E
n
rszre osztanak a metszsvonalaikkal,
ahol E
n
az I.5.1 Feladatbeli szm. Az (n +1)-edik sk E
n
szm rsze minden eddigi trrszt
kettoszt, gy a trrszek szmt E
n
-nel nveli. Kvetkezik, hogy
S
n+1
=S
n
+E
n
, n 1.
Felrva ezt n =1,2,3, ..., n1-re s sszeadva a kapott egyenleteket kapjuk, hogy S
n
=2+E
1
+
+E
2
+... +E
n1
. Hasznlva, hogy E
n
= 1+
_
n+1
2
_
, innen S
n
= 2+(n1)+
_
2
2
_
+
_
3
2
_
+... +
_
n
2
_
,
S
n
=n+1+
_
n+1
3
_
a binomilis egytthatk fels sszegzsi tulajdonsga szerint (I.2.3.2 Ttel),
teht
S
n
=
n
3
+5n+6
6
, n 1.
Megjegyezzk, hogy az addicis kpletet hasznlva S
n
gy is rhat: S
n
=
_
n
3
_
+
_
n
2
_
+
_
n
1
_
+
+
_
n
0
_
.
Rgtn addik, hogy n klnbz pont az egyenest P
n
=n+1=
_
n
1
_
+
_
n
0
_
rszre osztja. Vegyk
szre, hogy a rekurzv kpletek szerint az (S
n
) sorozat klnbsgsorozata az (E
n
) szmsorozat, (E
n
)
klnbsgsorozata pedig a (P
n
) szmsorozat, azaz S
n+1
S
n
=E
n
, E
n+1
E
n
= P
n
minden n-re.
Innen az egymst kvet P
n
, E
n
, S
n
rtkek knnyen meghatrozhatk, lsd az I.5.1 tblzatot,
ahol az E
n
s az S
n
sorban a msodik szmtl kezdden minden szm az eltte ll N szm s
az N fltt ll szmok sszege.
n 1 2 3 4 5 6
P
n
2 3 4 5 6 7
E
n
2 4 7 11 16 22
S
n
2 4 8 15 26 42
I.5.1. tblzat. A P
n
, E
n
s S
n
szmok tblzata
Azt mondjuk, hogy n kr ltalnos helyzet a skban, ha brmely kettnek kt kzs pontja
van, brmely hromnak viszont nincs kzs pontja.
41
I.5.3. Feladat. Hny rszre osztja a skot n ltalnos helyzet kr?
Megolds. Jellje K
n
a keresett szmot. Itt K
1
=2, K
2
=4, K
3
=8. Hrom ltalnos helyzet
kr a skot 8 rszre osztja, lsd az I.5.2 brt. Hny rszre osztja a skot ngy kr?
Igaz-e vajon, hogy K
n
= 2
n
minden n-re?
I.5.2. bra. Krk a skban
Tekintsnk n krt s mg egy (n+1)-edik krt. Az (n+1)-edik kr az az els n kr mindegyikt
kt pontban metszi, gy ezek a metszspontok az (n+1)-edik krt 2n rszre (krvre) osztjk.
Minden ilyen krv minden sktartomnyt kettvg, ezrt
K
n+1
=K
n
+2n, n 2.
Felrva ezt n=2,3, .., n1-re s sszeadva kapjuk, hogy K
n
=2+2(1+2+...+n1) =2+n(n
1) = n
2
n+2, teht
K
n
= n
2
n+2, n 1.
gy K
4
= 14, K
5
= 22, K
6
= 32, stb., s kvetkezik, hogy K
n
< 2
n
, ha n 4.
Megjegyzs. Innen kvetkezik, hogy ngy halmaz esetn Venn-diagramot nem lehet krlapokkal
megrajzolni. Ugyanis ngy halmaz esetn a skot, illetve egy tglalapot, 16 rszre kell osztani,
mert egy elem az adott ngy halmaz mindegyikhez vagy hozztartozik vagy sem, ez V
2
4
= 16
lehetsg. Krk helyett ms zrt grbkkel (pl. ellipszisekkel) vagy tglalapokkal megadhat a
Venn-diagram.
42 I.5. FEJEZET. KOMBINATORIKA A GEOMETRIBAN
I.6. fejezet
Fibonacci-szmok
A Fibonacci-szmokat gy deniljuk: F
0
= 0, F
1
= 1, F
n
=F
n1
+F
n2
, ahol n 2. A rekurzis
kplet alapjn F
2
-tl kezdve minden tag egyenl az elz kt tag sszegvel s kapjuk, hogy F
0
=0,
F
1
= 1, F
2
= 1, F
3
= 2, F
4
= 3, F
5
= 5, F
6
= 8, F
7
= 13, F
8
= 21, F
9
= 34, F
10
= 55, ... .
Ez a szmsorozat a kvetkez problmbl szrmazik, amely szerepel Fibonacci (Leonardo
Pisano) Liber Abaci cm 1202-ben rt munkjban:
Hny pr nylra szaporodik egy v alatt a kezdeti pr, ha tudjuk, hogy a nyulak kt hnap
alatt vlnak szaporodkpess, s ezutn minden nyl-pr minden hnapban egy j prnak ad
letet s mindegyikk letben marad?
A feladat megoldsban a nyl-prok szmnak idbeli alakulst kell kvetni. Az els hnapban
1 nyl-pr van, s ugyanannyi lesz a msodikban is, a prok szma a harmadik hnapban vltozik
1-rl 2-re. A kvetkez hnapban a szlk jabb prnak adnak letet, gy a prok szma 3-ra n.
Az tdik hnapban mr az j pr is szaporodkpes, gy az j prok szma kettvel n, s az
sszes prok szma 5 lesz. A kvetkez hnapban mr mindkt ifjabb generci hoz ltre utdokat,
s a prok szma hrommal nvekedve 8-ra vltozik, stb.
Jellje a
n
a nylprok szmt az n-edik hnapban. Itt a
n
=a
n1
+a
n2
, n 2, mert az n-edik
hnapban annyi nylpr van, mint az elz, az (n1)-edik hnapban (a
n1
rgi nylpr), amihez
hozz kell adni az (n2)-edik hnapban ltez nylprok szmt, mert ezek egy-egy j prral
szaporodtak (a
n2
j nylpr).
Teht, a kezdeti rtkeket is gyelembe vve, a nyl-prok szma az n-edik hnapban ppen
F
n
.
I.6.1. Feladat. Hnyflekppen mehetnk fel egy n lpcsfokbl ll lpcsn, ha egyszerre csak
egy vagy kt lpcsfokot lphetnk?
Megolds. Legyen x
n
a lehetsgek szma. Itt x
1
=1, x
2
=2, x
3
=3, x
4
=5, mert pl. n =3-ra
ezek a lehetsgek: 1+1+1, 1+2 vagy 2+1 lpcsfok. Sejts: x
n
= F
n+1
minden n 1-re.
Valban, ha n 3, akkor az utols lps szerint kt lehetsg van:
1. ha az utols lps 1 lpcsfok, akkor az (n1)-edik fokra x
n1
-flekppen lphetnk s 1
foknyi lpssel felrnk az n-edik fokra,
2. ha az utols lps 2 lpcsfok, akkor az (n2)-edik fokra x
n2
-flekppen lphetnk s 2
foknyi lpssel felrnk az n-edik fokra,
teht x
n
= x
n1
+x
n2
s innen indukcival azonnali, hogy x
n
= F
n+1
minden n 1-re.
I.6.2. Feladat. Egy n emeletes hz emeleteit hnyflekppen festhetjk le zld s srga sznekkel
gy, hogy ne legyen kt szomszdos srga emelet ?
I.6.3. Feladat. Hny olyan rszhalmaza van az 1,2, ..., n halmaznak, amelyek elemei kztt
nincs kt egymsutni szm?
43
44 I.6. FEJEZET. FIBONACCI-SZMOK
Megolds. Jellje x
n
ezt a szmot. Itt x
1
=2 (mert j az s az 1), x
2
=3 (j: , 1, 2,
nem j: 1,2), x
3
= 5 (j: , 1, 2, 3, 1,3, nem j: 1,2, {2,3},{1,2,3}).
Legyen n 3 tetszleges s tekintsk az 1,2, ..., n halmaz olyan H rszhalmazait, amelyek
elemei kztt nincs kt egymsutni szm. Csoportostsuk ezeket gy:
I. Ha n H, akkor n1 / H s annyi H rszhalmaz van, amennyi az x
n2
rtk.
II. Ha n / H, akkor annyi H rszhalmaz van, amennyi az x
n1
rtk.
Ezrt x
n
=x
n2
+x
n1
, s kapjuk, hogy x
n
=F
n+2
, ahol n 1.
A tovbbiakban kpletet adunk F
n
-re. Legyen
F(x) =

n=0
F
n
x
n
=x+x
2
+2x
3
+3x
4
+5x
5
+8x
6
+13x
7
+...
az a hatvnysor, amelynek egytthati a Fibonacci-szmok. Ezt a hatvnysort a Fibonacci szmsorozat
genertorfggvnynek nevezzk. Szorozva x-szel, majd x
2
-tel kapjuk, hogy:
F(x) = F
0
+F
1
x+F
2
x
2
+F
3
x
3
+F
4
x
4
+F
5
x
5
+...
xF(x) = F
0
x+F
1
x
2
+F
2
x
3
+F
3
x
4
+F
4
x
5
+...
x
2
F(x) = F
0
x
2
+F
1
x
3
+F
2
x
4
+F
3
x
5
+....
Adjuk ssze az utols kt sort: xF(x)+x
2
F(x) =F
0
x+(F
0
+F
1
)x
2
+(F
1
+F
2
)x
3
+(F
2
+F
3
)x
4
+
+(F
3
+F
4
)x
5
+... = F
0
x+F
2
x
2
+F
3
x
3
+F
4
x
4
+F
5
x
5
+..., amibl kivonva az els sort: xF(x) +
+x
2
F(x)F(x) =F
1
x =x, teht
F(x) =
x
1xx
2
.
Bontsuk F(x)-et elemi trtek sszegre. Itt az 1xx
2
=0 egyenlet gykei x
1,2
=
1

5
2
. Jellje:
=
1+

5
2
, =
1

5
2
, ahol +=1, =1. Akkor 1xx
2
=(xx
1
)(xx
2
) =(x+)(x+).
Legyen F(x) =
A
x+
+
B
x+
. Az A s B rtkeket meghatrozva kapjuk, hogy
F(x) =

5(x+)
+

5(x+)
=
1

5
_
1
1x

1
1x
_
,
ami kzvetlenl is ellenrizhet.
Most hasznljuk, hogy 1+x+x
2
+... +x
n
+... =
1
1x
, ahol [x[ <1 (geometriai sor). Innen
1
1ax
= 1+ax+a
2
x
2
+... +a
n
x
n
+...,
amit alkalmazva F(x) elbbi felrsra kapjuk, hogy
F(x) =
1

5
_
(1+x+
2
x
2
+...)(1+x+
2
x
2
+...)
_
=
1

5
_
()x+(
2

2
)x
2
+...
_
,
F(x) =
1

n=0
(
n

n
)x
n
.
Azonostva az F(x) hatvnysorban az egytthatkat kvetkezik, hogy F
n
=
1

5
(
n

n
), ahol
n 0. sszefoglalva az eddigieket:
45
I.6.4. Ttel.
i) A Fibonacci-szmok genertorfggvnye F(x) =
x
1xx
2
.
ii) (Binet-kplet)
1
Az F
n
Fibonacci-szmokra F
n
=
1

5
(
n

n
), n 0 , ahol =
1+

5
2

1,618033, =
1

5
2
0,618033.
Megjegyezzk, hogy s az x
2
x1 = 0 egyenlet gykei. Ezek a szmok kapcsolatosak az
aranymetszssel. Aranymetszst akkor vgznk, ha egy szakaszt gy osztunk kt rszre, hogy a
nagyobbik arnya az egsz szakaszhoz egyenl legyen a kisebbik rsznek a nagyobbik rszhez val
arnyval. Ha x s y jelli ezeket a rszeket, ahol x > y, akkor a felttel :
x
x+y
=
y
x
= a. Innen
x
2
xy y
2
= 0 s az x/y = u jellssel u
2
u1 = 0, u
1
= > 0, u
2
= < 0, teht u = x/y = .
Az a tkletes arny rtke pedig y/x=1/=0,618033 (ez szmos kpzmvszeti alkots
elemei kztt meggyelhet).
Mivel [[ <1, kvetkezik, hogy lim
n

n
= 0, gy lim
n
F
n

n
/

5
= 1, azaz F
n

n
/

5, olvasd: F
n
aszimptotikusan egyenl
n
/

5-tel.
Teht nagy n rtkekre F
n
kzeltleg
n
/

5. St, ez mr kis n-ekre is nagyon j kzeltst ad.


Pl. n=10-re F
10
=55,
10
/

5 =55,003636, n=20-ra F
20
=6765,
20
/

5 6765,000029, n=25-re
F
25
= 75025,
25
/

5 = 75024,999997.
Mi tbb, [
n
/

5[<1/2, ahol n0, gy a Binet-kpletbl [F


n

n
/

5[<1/2, teht F
n
tvolsga

n
/

5-tl 1/2-nl kisebb minden n 0-ra. Kvetkezik, hogy F


n
=
_

5
+
1
2
_
, n 0 , ahol [x] az
x szm egszrszt jelli, ami egy jabb zrt alak F
n
-re. Az is igaz, hogy ha n pros, akkor
F
n
<
n
/

5, ha pedig n pratlan, akkor F


n
>
n
/

5.
I.6.5. Feladat. Igazoljuk, hogy
n

k=0
_
n
k
_
F
k
= F
2n
, ahol n 0.
Megolds. Hasznlva a Binet-kpletet s a binomilis-ttelt:
n

k=0
_
n
k
_
F
k
=
1

5
n

k=0
_
n
k
_
(
k

k
) =
1

5
n

k=0
_
n
k
_

5
n

k=0
_
n
k
_

k
=
=
1

5
((1+)
n
(1+)
n
) =
1

5
(
2n

2n
) = F
2n
.
Az I.6.4 Ttelbeli Binet-kplet teljes indukcival kzvetlenl is igazolhat, de ehhez szksges,
hogy azt elre megsejtsk (ami nehz). A Fibonacci-sorozat genertorfggvnyt hasznl fenti
mdszer elnye az, hogy az F
n
-re vonatkoz kplet gy vezethet le, hogy azt (vagy annak alakjt)
elre nem sejtjk. A levezetsben elg, ha formlis hatvnysorokkal dolgozunk, a konvergencia
vizsglata nlkl.
ltalnosan az (a
n
)
n0
vals szmsorozat genertorfggvnye az A(x) =a
0
+a
1
x+a
2
x
2
+
+...+a
n
x
n
+... formlis hatvnysor, az a
0
, a
1
, ..., a
n
vges vals szmsorozat genertorfggvnye
az f(x) = a
0
+a
1
x+a
2
x
2
+... +a
n
x
n
polinom.
1
Ezt a kpletet Leonhard Euler publiklta 1765-ben, Jacques Binet 1843-ban jra felfedezte.
46 I.6. FEJEZET. FIBONACCI-SZMOK
Pl. a binomilis egytthatk
_
n
0
_
,
_
n
1
_
, ...,
_
n
n
_
sorozathoz tartoz genertorfggvny a kvetkez
polinom:
_
n
0
_
+
_
n
1
_
x+... +
_
n
n
_
x
n
= (1+x)
n
.
A formlis hatvnysorokra, illetve a polinomokra vonatkoz szoksos mveleti szablyok (pl.
az sszeadsra, szorzsra, hatvnyozsra vonatkoz szablyok) alkalmazsval lehetsg van a
vizsglt sorozatok tulajdonsgainak viszglatra (ltalnos tag, rekurzis kpletek). Tovbb a
matematikai analzis eszkzei is alkalmazhatk (dierencils, integrls). A mveletek tbbsge
attl fggetlenl elvgezhet, hogy a fellp sorok konvergensek-e vagy sem.
A genertorfggvnyek vizsglata egy fontos s hatkony mdszer nemcsak a kombinatorikban,
hanem a matematika ms terletein is.
Megjegyezzk, hogy vgtelen sok tag sorozatok esetn a genertorfggvny elnevezs kiss
flrevezet, mert A(x)=a
0
+a
1
x+a
2
x
2
+...+a
n
x
n
+...-et fggvnyknt tekintve az A(x) fggvnyrtk
konvergencia problmkat vet fel, amelyekkel nem szksges foglalkoznunk, ha formlis hatvnysorokkal
dolgozunk.
I.6.6. Feladat. A binomilis egytthatk (1+x)
n
genertorfggvnybl kiindulva igazoljuk, hogy
a)
_
m
0
__
n
r
_
+
_
m
1
__
n
r1
_
+
_
m
2
__
n
r2
_
+... +
_
m
r
__
n
0
_
=
_
m+n
r
_
, ahol r 0, r m, r n, lsd I.2.3.4
Ttel,
b)

n
k=1
k
_
n
k
_
= n 2
n1
, ahol n 1, lsd I.2.1.6 Feladat.
Megolds. a) (1+x)
m+n
=(1+x)
m
(1+x)
n
, itt a bal oldalon x
r
egytthatja
_
m+n
r
_
. Vizsgljuk
meg mennyi x
r
egytthatja a jobb oldalon.
b)

n
k=0
_
n
k
_
x
k
=(1+x)
n
, derivlva:

n
k=1
k
_
n
k
_
x
k1
=n(1+x)
n1
, s x=1-re

n
k=1
k
_
n
k
_
=n2
n1
.
A Fibonacci-sorozat ltalnos tagjra vontkoz kplet mskppen is levezethet. A pldatri
rszben szerepl 3 Feladatbeli eljrs alkalmas az x
n+1
=ax
n
+bx
n1
, n 1 msodrend lland
egytthats lineris rekurzikkal adott sorozatok n-edik tagjnak a meghatrozsra is, ahol
x
0
=c s x
1
= d adott rtkek.
Ha a =b =1, c =0, d =1 akkor visszakapjuk a Fibonacci-sorozatot. Ha a =b =1, c =2, d =1,
akkor a kvetkez sorozatot kapjuk:
(L
n
)
n0
, L
n+1
= L
n
+L
n1
, ahol L
0
= 2, L
1
= 1, ezt Lucas-sorozatnak nevezzk. Itt teht
L
0
= 2, L
1
= 1, L
2
= 3, L
3
= 4, L
4
= 7, L
5
= 11, L
6
= 18,... .
I.7. fejezet
Catalan-szmok
I.7.1. Feladat. Adottak az a
0
, a
1
, a
2
, ..., a
n
vltozk (szmok), ahol n 0, s tekintsk ezek
a
0
a
1
a
2
a
n
szorzatt. Hnyflekppen zrjelezhet egyrtelmen ez a szorzat ? Pontosabban:
Hnyflekppen zrjelezhet ez a szorzat n1 zrjelpr alkalmazsval ?
Jelljk C
n
-nel ezt a szmot. Ha n = 2, akkor kt lehetsg van: (a
0
a
1
) a
2
s a
0
(a
1
a
2
),
teht C
2
= 2. Ha n = 3, akkor a lehetsgek: ((a
0
a
1
) a
2
) a
3
, (a
0
(a
1
a
2
)) a
3
, a
0
((a
1
a
2
) a
3
),
a
0
(a
1
(a
2
a
3
)), (a
0
a
1
) (a
2
a
3
), ezek szma C
3
=5. Tovbb C
1
=1 (kt tnyez), C
0
=1 (ekkor
egy tnyez van). Mi lesz C
n
?
A tovbbiakban a genertorfggvny mdszerrel igazoljuk, hogy C
n
=
1
n+1
_
2n
n
_
minden n
0-ra. Ezeket a szmokat, amelyek szmos kombinatorikai problmban fellpnek, Catalan-
szmoknak nevezzk.
1
A Catalan-szmok sorozata C
0
=1, C
1
=1, C
2
=2, C
3
=5, C
4
=14, C
5
=42, C
6
=132, C
7
=429,
C
8
= 1430, ... . Jellje C(x) = C
0
+C
1
x+C
2
x
2
+... a C
n
szmok genertorfggvnyt.
I.7.2. Ttel.
1) Ha n 1, akkor C
n
= C
0
C
n1
+C
1
C
n2
+... +C
n1
C
0
.
2) C(x) =
1

14x
2x
=
2
1+

14x
.
Bizonyts. 1) Legyen n1 tetszleges s tekintsk a
0
, a
1
, a
2
, ..., a
n
valamely zrjelezst. Figyeljk
meg, hogy pontosan egy olyan szorzsjel van, amely minden zrjelen kvl esik. Ha ez a szorzsjel
az a
k
s a
k+1
kz esik, akkor az eltte lev a
0
, a
1
, ..., a
k
vltozk C
k
-flekppen zrjelezhetk, az
utna lev nk vltoz pedig C
nk1
-flekppen, gy a lehetsgek szma rgztett k-ra C
k
C
nk1
,
sszesen pedig

n1
k=0
C
k
C
nk1
.
2) Az 1) pontbeli kplet alapjn
C(x) =

n=0
C
n
x
n
= 1+

n=1
(
n1

k=0
C
k
C
nk1
)x
n
= 1+

k=0
C
k
x
k

n=k+1
C
nk1
x
nk
=
= 1+x

k=0
C
k
x
k

n=k+1
C
n(k+1)
x
n(k+1)
,
1
Catalan, Eugne Charles (18141894) belga matematikus neve utn. Catalan ezeket a szmokat Segner-
szmoknak nevezte. Segner, Jnos Andrs (17041777) magyar tuds volt, Pozsonyban szletett, Jnban,
Gttingenben, Hallban volt egyetemi tanr, orvosi, zikai s matematikai eladsokat tartott. 1758-ban adta
meg az a
n
=a
0
a
n1
+a
1
a
n2
+... +a
n1
a
0
rekurzit Eulernek a konvex sokszgekre vonatkoz problmjra, lsd
I.7.4 Feladat.
47
48 I.7. FEJEZET. CATALAN-SZMOK
ahol

n=k+1
C
n(k+1)
x
n(k+1)
=C
0
+C
1
x+C
2
x
2
+... = C(x), gy kapjuk, hogy
C(x) = 1+xC
2
(x), s innen C(x) =
1

14x
2x
=
2
1

14x
.
Melyik a megfelel eljel ? Mivel C(0) = C
0
= 1 kvetkezik, hogy C(x) =
1

14x
2x
=
2
1+

14x
a
helyes kplet.
Az 1) pontbeli rekurzv kpletbl (amelynek jobb oldala egy konvolcis sszeg) meghatrozhatk
az egymst kvet C
n
szmok.
I.7.3. Ttel. Minden n 0 szmra C
n
=
1
n+1
_
2n
n
_
.
Bizonyts. Hasznljuk az ltalnostott binomilis kpletet, lsd az I.2.1.7 eltti kpletet, ahonnan
= 1/2-re

14x =

k=0
_
1/2
k
_
(4x)
k
s az I.7.2 Ttel szerint
C(x) =
1
2x
_
1

k=0
_
1/2
k
_
(4x)
k
_
=
1
2x

k=1
_
1/2
k
_
(4)
k
x
k
=
1
2

k=1
_
1/2
k
_
(4)
k
x
k1
,
s a k1 = n helyettestssel
C(x) = 2

n=0
_
1/2
n+1
_
(4)
n
x
n
.
Adjuk meg most
_
1/2
n+1
_
rtkt:
_
1/2
n+1
_
=
1
(n+1)!
_
1
2
__

1
2
__

3
2
__

5
2
_

_

2n1
2
_
= (1)
n
1 3 5 (2n1)
(n+1)! 2
n+1
=
= (1)
n
(2n)!
2
n+1
(n+1)! 2 4 (2n)
= (1)
n
(2n)!
2
2n+1
(n+1)! n!
= (1)
n
1
2
2n+1
(n+1)
_
2n
n
_
.
Behelyettestve kvetkezik, hogy
C(x) =

n=0
1
n+1
_
2n
n
_
x
n
,
ahonnan kapjuk a C
n
-re vonatkoz kpletet.
Ismertetnk tovbbi nhny olyan kombinatorikai feladatot, amelyekben a Catalan-szmok
lpnek fel.
I.7.4. Feladat. (Euler) Hnyflekppen lehet egy n oldal konvex sokszget hromszgekre bontani
olyan tlkkal, amelyek a sokszg belsejben nem metszik egymst ?
49
Megolds. Jelljk a keresett szmot E
n
-nel. Itt E
2
=1 (megllapods szerint), E
3
=1, E
4
=2,
E
5
= 5. Tekintsk az A
1
A
2
. . . A
n
sokszg egy hromszgekre bontst. Legyen az A
1
A
n
oldal
hromszg harmadik cscsa A
k
(5. bra). Az A
1
A
k
s A
k
A
n
tlk az n-szget az A
1
A
2
...A
k
k-
szgre, az A
1
A
k
A
n
hromszgre s az A
k
A
k+1
...A
n
(nk +1)-szgre bontjk. Mivel a k-szget
E
k
-flekppen, az (n k +1) szget E
nk+1
-flekppen bonthatjuk fel hromszgekre ezrt az
A
1
A
k
A
n
hromszget tartalmaz hromszgekre bontsok szma E
k
E
nk+1
, ahol 2 k n1.
sszegezve: E
n
=
n1

k=2
E
k
E
nk+1
, ahol n 3.
Ha n helyett (n+2)-t runk (a feladatban n oldal sokszg helyett n+2 oldal sokszget
tekintnk), akkor kapjuk, hogy E
n+2
=
n+1

k=2
E
k
E
nk+3
, innen az E

n
=E
n+2
jellssel E

n
=
n+1

k=2
E

k2
E

nk+1
,
azaz
E

n
=
n1

j=0
E

j
E

nj1
,
ami ugyanaz, mint a I.7.2 Ttelben szerepl rekurzi. Mivel a kezdeti rtkekre E

1
= 1 = C
1
,
kvetkezik, hogy E

n
=C
n
minden n 1-re.
Teht E
n
=C
n2
=
1
n1
_
2n4
n2
_
, n 3.
I.7.1. bra. Konvex sokszg felbontsa hromszgekre
Kzvetlenl is belthatjuk, hogy minden n-re bijektv megfeleltets van egy (n +2)-oldal
konvex sokszg hromszgekre bontsai s az a
0
a
1
a
n
szorzat zrjelezsei kztt, ahonnan
kvetkezik, hogy E
n+2
=C
n
. Valban, ha adott egy (n+2)-oldal konvex sokszgnek egy hromszgekre
bontsa, akkor kvessk az albbi eljrst:
a) rjuk fel a sokszg oldalaira egymsutn az a
0
, a
1
, ..., a
n
vltozkat,
b) keressnk egy olyan hromszget, amelynek kt oldala a sokszg szomszdos megjellt
oldalai (biztosan van ilyen hromszg),
c) trljk le az brrl ezt a kt oldalt, s a kt oldalon lev vltozk szorzatt rjuk fel a
hromszg harmadik oldalra (az eredeti sokszg egy tljra),
d) a kapott sokszgre ismteljk meg a b), c), d) lpseket.
gy a a
0
a
1
a
n
szorzat egy zrjelezst kapjuk. Belthat, hogy klnbz hromszgekre
bontsokhoz klnbz zrjelezsek tartoznak, s minden zrjelezst megkapunk.
50 I.7. FEJEZET. CATALAN-SZMOK
I.7.5. Feladat. Hnyflekppen juthatunk el a koordintarendszerben a (0,0) pontbl a (2n,0)
pontba gy, hogy lpseink az f =(1,1) s =(1, 1) vektorok lehetnek s ne menjnk a vzszintes
tengely al? (f = tlsan fel egyet, = tlsan le egyet a rcspontok kztt.)
I.7.6. Feladat. Hny olyan 2n-tag x
1
, x
2
, ..., x
2n
sorozat van, ahol a tagok mindegyike +1 vagy
1, x
1
+x
2
+... +x
2n
= 0, s az x
1
, x
1
+x
2
, x
1
+x
2
+x
3
, ..., x
1
+x
2
+... +x
2n
parcilis sszegek
mindegyike 0?
Megolds. Ez ugyanaz a problma, mint az I.7.5 Feladatbeli. Az f = fel lpst jellje +1, az
= le lpst jellje 1, ahol az n1 db. +1-et s az n1 db. (1)-et egyms mell rva sszegk
0 lesz. Az, hogy az t nem megy a vzszintes tengely al ppen azzal ekvivalens, hogy a parcilis
sszegek mind 0 rtkek.
Vlasz: C
n
=
1
n+1
_
2n
n
_
.
I.7.7. Feladat. Hnyflekppen juthatunk el az nn-es sakktbln a bal als sarokbl a jobb fels
sarokba gy, hogy egyszerre egyet lphetnk felfel vagy jobbra s nem lphetnk a (bal als sarkot
a jobb fels sarokkal sszekt) mellktl fl?
Megolds. Legyen a
n
s b
n
az sszes lehetsges utak szma, ill. a mellktl alatt marad
utak szma. Itt a
1
= 1, a
2
= 2, a
3
= 6, a
4
= 20, ... s b
1
= 1, b
2
= 1, b
3
= 2, b
4
= 5, ... .
Jellje A s B a bal als sarkot, ill. a jobb fels sarkot. Az A-t B-vel sszekt utak (trtt
vonalak) szma a
n
=
_
2n2
n1
_
, mert 2n2 lpsre van szksg, ebbl n1 lps felfel s n1 lps
jobbra. Azt kell megadni pl., hogy mikor lpnk jobbra. A 2n2 lpsbl kell teht kivlasztani
(n1)-et, a sorrend nem szmt.
Ezek kzl hny olyan AB t van, amely a mellktl alatt marad? Nevezzk ezeket j
utaknak. Tekintsk a nem ilyen, a rossz utakat. Minden rossz t a mellktl fl kerl, teht
metszi az I.7.2 brn lthat e egyenest. Jellje P azt a pontot, ahol egy rossz t elszr metszi
az e egyenest. A rossz t gy az AP s P B rszekbl ll. Az AP trtt vonalat tkrzzk
az e egyenesre, legyen a tkrkp az A

P trtt vonal. Az A

P egytt a P B-vel (ez utbbi


vltozatlanul) egy A

B t.
I.7.2. bra. Sakktbla s a Catalan-szmok
51
Belthat, hogy gy bijektv megfeleltetst ltestettnk az AB rossz utak s az A

B (sszes
lehetsges) utak kztt. Itt A

-bl B-be gy juthatunk, hogy a 2n2 lpsbl megadjuk azt az


n-et, amikor jobbra lpnk. Ezek szma
_
2n2
n
_
, hasonlan, mint a megolds elejn.
A j utak szma gy b
n
=
_
2n2
n1
_

_
2n2
n
_
=
1
n
_
2n2
n1
_
=C
n1
. Ez egy kzvetlen bizonyts,
itt nem hivatkoztunk a Catalan-szmok elzetesen levezett tulajdonsgaira.
Innen kvetkezik az I.7.6 Feladat megoldsa is. Valban, n helyett n+1-et rva tekintsk az
(n+1)(n+1)-es sakktblt. A jobbra lpst jellje +1, a felfele trtn lpst 1. Itt az n db.
+1 s az n db. (1)-et egyms mell rva sszegk 0 lesz. Az, hogy az t a mellktl alatt marad
ppen azzal ekvivalens, hogy a parcilis sszegek mind 0 rtkek. Teht az I.7.6 Feladatra a
vlasz: C
n
=
1
n+1
_
2n
n
_
.
Megmutatjuk, hogy az eredeti I.7.1 Feladatra is adhat a genertorfggvny mdszert nem
hasznl, elemi megolds gy, hogy visszavezetjk a most kzvetlenl megoldott I.7.6 Feladatra.
Bijektv megfeleltets van az a
0
a
1
a
n
szorzat zrjelezett alakjai s az I.7.6 Feladat feltteleinek
eleget tev 2n-tag x
1
, x
2
, ..., x
2n
sorozatok kztt.
Valban, tekintsk az a
0
a
1
a
n
szorzat egy tetszleges zrjelezett alakjt.
1. lps: tegynk ki mg egy pr zrjelet az elejre s a vgre,
2. lps: a szorzsjelek helyett rjunk +1-et,
3. lps: a jobb oldali zrjelek helyett rjunk 1-et,
4. lps: a bal oldali zrjeleket s az a
0
, a
1
, ..., a
n
vltozkat trljk le.
Ekkor a kapott +1 s 1 szmokbl ll sorozat teljesti a 15.6. Feladat feltteleit. Klnbz
zrjelezsekhez klnbz sorozatok tartoznak s minden sorozatot megkapunk. Pl. n = 3-ra
(a
0
a
1
) (a
2
a
3
)-bl kiindulva ((a
0
a
1
) (a
2
a
3
)) s +11+1+111 addik,
a
0
(a
1
(a
2
a
3
))-bl kiindulva (a
0
(a
1
(a
2
a
3
))) s +1+1+1111 addik.
52 I.7. FEJEZET. CATALAN-SZMOK
I.8. fejezet
Stirling-szmok
I.8.1. Msodfaj Stirling-szmok
Elszr az n. msodfaj Stirling-szmokkal foglalkozunk.
Legyen A egy tetszleges vges halmaz s legyenek B
1
, B
2
, ..., B
k
A. Ha ezek a rszhalmazok
nem resek, pronknt diszjunktak s unijuk az A halmaz, akkor azt mondjuk, hogy B
1
, B
2
, ..., B
k
az A halmaz egy osztlyfelbontst (ms nven osztlyozst vagy partcijt) alkotjk. A B
i
rszhalmazokat osztlyoknak vagy blokkoknak nevezzk. Az osztlyfelbontsok kpzst az adott
halmaz particionlsnak is nevezzk.
Legyen A egy n-elem halmaz (n1). Hny k rszhalmazra val osztlyfelbontsa van A-nak?
Ha pl. k = 2, akkor A-t kt nemres valdi rszhalmazra kell bontanunk. Egy ilyen rszhalmazt
(2
n
2)-flekppen lehet kivlasztani (az res halmaz s az A nem jk), ekkor a msik rszhalmaz
meghatrozott. De a kt halmaz sorrendje lnyegtelen, ezrt a lehetsgek szma 2
n
2 fele, azaz
2
n1
1.
Pl. A=1,2,3 kt rszhalmazra val partcii : 1,23, 1,32, 2,31, ezek szma
3 = 2
31
1.
Az A = 1,2,3,4 kt rszhalmazra val partcii : 1 2,3,4, 2 1,3,4, 3 1,2,4,
41,2,3, 1,23,4, 1,32,4, 1,42,3, ezek szma 7 = 2
41
1.
I.8.1.1. Denci. Legyenek n1, k1. Egy n-elem halmaz k rszhalmazra val osztlyfelbontsainak
szmt msodfaj Stirling-szmnak nevezzk, jells
n
k
vagy S(n, k). Az sszes partcik
szma a B(n)-nel jellt Bell-szm, amelyre B(n) =
n

k=1
_
n
k
_
.
Megjegyezzk, hogy B(n) nem ms, mint egy n elem halmazon denilhat ekvivalenciarelcik
szma.
A denci szerint azonnali, hogy ha k>n, akkor
n
k
=0. Tovbb
n
1
=1 (n1),
n
n1
=
_
n
2
_
(n 2),
n
n
= 1 (n 1). Lttuk, hogy
n
2
= 2
n1
1 (n 2).
Megllapods szerint (*)
n
0
= 0 minden n 1-re s (**)
0
0
= 1.
I.8.1.2. Ttel. Ha 1 k n, akkor
_
n
k
_
=k
_
n1
k
_
+
_
n1
k1
_
.
Bizonyts. Legyen 2 k n. Tekintsk egy n-elem halmaz k rszhalmazra (k blokkra) val
partciit. Az n-edik elem vagy nmaga alkot egy blokkot, vagy nem.
53
54 I.8. FEJEZET. STIRLING-SZMOK
n
n
1

n
2

n
3

n
4

n
5

n
6
B(n)
1 1 1
2 1 1 2
3 1 3 1 5
4 1 7 6 1 15
5 1 15 25 10 1 52
6 1 31 90 65 15 1 203
I.8.1. tblzat. Msodfaj Stirling-szmok s Bell-szmok
Ha n egy blokk, akkor n1 elemet kell mg k 1 blokkra osztani s ez
n1
k1
-flekppen
lehetsges.
Ha n nem blokk, akkor particionljuk elszr az 1,2, ..., n1 halmazt k blokkra, ezt
n1
k
-
flekppen lehet megtenni. Majd mind a k blokkhoz vegyk hozz az n elemet, ez k-flekppen
lehetsges.
A partcik szma teht
n1
k1
+k
n1
k
.
Ha k =1, akkor
n
1
=
n1
1
+
n1
0
. Innen n2-re a fenti (*) konvenci szerint 1 =1+0, ami
igaz, n =1-re pedig (**) szerint 1 =0+1, ami igaz (ezrt clszerek a (*) s (**) konvencik).
Az I.8.1 tblzat a msodfaj Stirling-szmokat s a Bell-szmokat tartalmazza:
Az elbbi rekurzv kplet szerint (amely hasonl a binomilis egytthatk addicis kplethez)
itt minden
n
k
bels szm egyenl a felette ll szm k-szorosnak s az attl egy hellyel balra
ll szm sszegvel. Ennek alapjn a tblzat knnyen kiegszthet jabb sorokkal.
I.8.1.3. Feladat. Adjuk meg az 5. Tblzat kvetkez hrom sort.
A Bell-szmokra vonatkozik a kvetkez rekurzi. Megjegyezzk, hogy az I.8.1.1 Denci s az
elbbi (*) konvenci szerint B(n)=
n
0
+
n
1
+...+
n
n
(n1). Ha n=1, akkor innen B(0)=
0
0
=1
(**) szerint, ezrt legyen (***) B(0) = 1.
I.8.1.4. Ttel. Ha n 0, akkor
B(n+1) =
n

k=0
_
n
k
_
B(k) .
Bizonyts. Nzzk az 1,2, ..., n, n+1 halmaz sszes partciit aszerint, hogy az n+1 elem blokkja
hny elemet tartalmaz.
Tegyk fel, hogy az n+1 blokkjban j szm elem van, ahol 1j n+1. Akkor a blokk tbbi
j 1 szm elemt
_
n
j1
_
-flekppen lehet megvlasztani, s ha ez megtrtnt, akkor a tovbbi
n+1j szm elemet sszesen B(n+1j)-flekppen lehet particionlni (ha itt j =n+1, akkor
n+1j = 0, ide kell, hogy B(0) = 1). gy
B(n+1) =
n+1

j=1
_
n
j 1
_
B(n+1j) =
n+1

j=1
_
n
nj +1
_
B(n+1j) =
n

k=0
_
n
k
_
B(k).
A kvetkez kpletbl az
n
k
msodfaj Stirling-szmok kzvetlenl is szmthatk.
I.8.1. MSODFAJ STIRLING-SZMOK 55
I.8.1.5. Ttel. Ha 1 k n, akkor
_
n
k
_
=
1
k!
s
n,k
=
1
k!
k1

j=0
(1)
j
_
k
j
_
(kj)
n
,
ahol s
n,k
az f : 1,2, ..., n1,2, ..., k szrjektv fggvnyek szma (lsd I.3.5, n s k felcserlve).
Bizonyts. Ha az f :1,2, ..., n1,2, ..., k fggvny szrjektv, akkor f
1
(1), f
1
(2), ..., f
1
(k)
az 1,2, ..., n halmaz egy olyan partcijt adjk, amely k blokkot tartalmaz. Figyeljk meg, hogy
a klnbz szrjektv f fggvnyek minden k blokkot tartalmaz partcit k!-szor adnak meg,
mert az f
1
(i) halmazokat permutlva a partci nem vltozik, de f megvltozik. Kvetkezik,
hogy a tekintett szrjektv fggvnyek szma s
n,k
=k!
n
k
, s lsd az I.3.5 Feladat megoldsban
adott kpletet.
I.8.1.6. Feladat. Ha n 1, akkor
x
n
=
n

k=1
_
n
k
_
[x]
k
,
ahol [x]
k
= x(x1) (xk+1).
Megolds. 1. md. Ellenrzs: n = 1-re x =
_
1
1
_
[x]
1
x = x igaz, n = 2-re x
2
=
_
2
1
_
[x]
1
+
+
_
2
2
_
[x]
2
x
2
=x+x(x1) igaz, n = 3-ra x
3
=
_
3
1
_
[x]
1
+
_
3
2
_
[x]
2
+
_
3
3
_
[x]
3
x
3
=x+3x(x
1)+x(x1)(x2) igaz.
Az I.8.1.2 Ttelbeli rekurzi alapjn n szerinti indukcival. Tegyk fel, hogy a kplet igaz
n1-re s igazoljuk n-re, ahol n 2.
n

k=1
_
n
k
_
[x]
k
=
_
n
n
_
[x]
n
+
n1

k=1
k
_
n1
k
_
[x]
k
+
n1

k=2
_
n1
k1
_
[x]
k
,
ahol hasznlva, hogy [x]
k
= [x]
k1
(xk+1), a j = k1 index-cservel kapjuk, hogy
_
n
n
_
[x]
n
+
n1

k=2
_
n1
k1
_
[x]
k
=
_
n1
n1
_
[x]
n
+
n1

k=2
_
n1
k1
_
[x]
k1
(xk+1)=
n1

j=1
_
n1
j
_
[x]
j
(xj)=
=x
n1

j=1
_
n1
j
_
[x]
j

n1

j=1
j
_
n1
j
_
[x]
j
= x x
n1

n1

k=1
k
_
n1
k
_
[x]
k
,
az indukcis felttel szerint s visszahelyettestve ksz.
2. md. A kplet kt n-edfok polinom egyenlsge. Ezt elegend minden xN

esetn igazolni,
mert ha a kt polinom minden x N

-ra egyenl, akkor minden x R-re is egyenl. Legyen teht


x N

. Kombinatorikusan bizonytunk: Legyen A =1,2, ..., n, B =1,2, ..., x. Akkor x


n
az f :
: A B fggvnyek szma. Ezeket csoportostsuk f(A) szmossga szerint: ha [f(A)[ = k, akkor
az f : Af(A) szrjektv fggvnyek szma s
n,k
=k!
_
n
k
_
, lsd I.8.1.5 Ttel, s az f(A)-ban lev k
elem megvlasztsra
_
x
k
_
lehetsg van. Teht [f(A)[ =k esetn a fggvnyek szma k!
_
n
k
__
x
k
_
=
=
_
n
k
_
[x]
k
, ahol 1 k n, ksz.
56 I.8. FEJEZET. STIRLING-SZMOK
I.8.2. Elsfaj Stirling-szmok
Trjnk t az n. elsfaj Stirling-szmok vizsglatra. Ezek n elemk ciklusba val rendezhetsgeinek
szmt adjk meg (a msodfaj Stirling-szmok n elemk nemres rszhalmazba val elrendezseinek
szmt jelentik).
Tekintsk pl. a
=
_
1 2 3 4 5 6 7 8 9
3 2 6 9 4 1 8 7 5
_
9-edfok permutcit (lsd I.1.1.8). Itt 1-nek megfelel 3, azaz (1)=3, hasonlan (3)=6, (6)=1,
visszakapjuk az 1-et. Nem szerepelt a 2, amelyre (2) = 2, tovbb, (4) = 9, (9) = 5, (5) = 4,
visszaadja a 4-et. Nem volt mg a 7: (7) =8, (8) =7. Teht: 1361, 22, 4954,
7 8 7.
gy a kvetkez n. ciklusokat kapjuk: (1 3 6), (2), (4 9 5), (7 8). A ciklusokban minden elemnek
a rkvetkez felel meg, az utolsnak pedig az els. Egy ciklust brmelyik elemvel lehet kezdeni,
pl. (4 9 5) = (9 5 4) = (5 4 9). A permutcit gy adjuk meg:
= (1 3 6)(2)(4 9 5)(7 8).
A ciklusok sorrendje megvltoztathat, rhat pl. gy is: = (7 8)(1 3 6)(4 9 5), vagy =
= (4 9 5)(7 8)(1 3 6), stb.
Hasonlkppen, minden n-edfok permutci felbonthat ciklusokra s a felbonts egyrtelm,
eltekintve a ciklusok sorrendjtl s a ciklusok kezdelemtl.
Valjban a permutci felbonthat ezeknek a diszjunkt ciklusoknak a szorzatra, ahol a
ciklusokat is permutciknak tekintjk s a szorzs a permutciknak, mint fggvnyeknek a
kompozcijt (sszettelt) jelenti, de ezekre az algebrai fogalmakra s tulajdonsgokra itt nincs
szksgnk.
Azt mondjuk, hogy a n-edfok permutci tpusa (k
1
, k
2
, ..., k
n
), ha felbonthat k
1
szm 1 hosszsg, k
2
szm 2 hosszsg, ..., k
n
szm n hosszsg diszjunkt ciklusra, akol
k
1
+2k
2
+... +nk
n
=n. Itt teht k
i
az i hosszsg ciklusok szma.
A fenti pldban adott 9-edfok permutci tpusa (1,1,2,0,0,0,0,0,0), ahol n=9, k
1
=k
2
=1,
k
3
= 2, k
4
=... =k
9
= 0 s 1 1+2 1+3 2 = 9.
A =
_
1 2 3 4 5 6
5 6 2 3 4 1
_
6-odfok permutcira 1 5 4 3 2 6 1, ezrt egy ciklus
van, =(1 5 4 3 2 6). Azt mondjuk, hogy ez egy ciklikus permutci, s ennek tpusa (0,0,0,0,0,1).
Az e=
_
1 2 3 4 5 6
1 2 3 4 5 6
_
6-odfok identikus permutcira e=(1)(2)(3)(4)(5)(6), ennek tpusa
(6,0,0,0,0,0).
I.8.2.1. Ttel. (Cauchy) A (k
1
, k
2
, ..., k
n
) tpus n-edfok permutcik szma
T(k
1
, k
2
, ..., k
n
) =
n!
k
1
! k
2
! k
n
! 1
k
1
2
k
2
n
k
n
.
Bizonyts. Egy (k
1
, k
2
, ..., k
n
) tpus permutcit rjunk a ciklusok hosszainak nvekv sorrendjben:
= ()...()
. .
k
1
()...()
. .
k
2
... (
n
..
...)...(
n
..
...)
. .
k
n
,
ahol a csillagok az 1,2, ..., n szmokat jellik. Ezt gy kapjuk, hogy az 1,2, ..., n szmok kzl
kivlasztunk k
1
-et, amelyek a k
1
szm 1-hosszsg ciklust alkotjk, kivlasztunk 2k
2
szmot,
I.8.2. ELSFAJ STIRLING-SZMOK 57
amelyek a k
2
szm 2-hosszsg ciklust alkotjk, stb. Ez sszesen P
n
= n! lehetsg (ha a
zrjeleket trljk, akkor az 1,2, ..., n elemek permutciit kapjuk). De a k
i
szm i-hosszsg
ciklus egyms kztt felcserlhet, ezrt osztani kell k
i
!-sal minden i-re. Tovbb, mind a k
i
szm
i-hosszsg ciklust i-flekppen rhatjuk, hiszen brmelyik elemmel kezdhetjk, ezrt osztani kell
i
k
i
-nel minden i-re.
I.8.2.2. Denci. Legyen 1 k n. Azoknak az n-edfok permutciknak a szmt, amelyek k
ciklust tartalmaznak elsfaj Stirling-szmnak nevezzk, jells [
n
k
] vagy s(n, k).
A denci szerint azonnali, hogy [
n
n
] = 1 (az e identikus permutci, amelyben minden elem
a helyn marad). Tovbb [
n
1
] = (n1)!, mert ha egy ciklus van, azaz ciklikus permutcirl van
sz, akkor a tpus (0,0, ...,0,1) s a Cauchy-ttel szerint T(0,0, ...,0,1) =
n!
n
= (n1)!. Ugyanakkor
[
n
n1
] =
_
n
2
_
, mert ha n1 ciklus van, akkor a tpus (n2,1,0...,0) s a Cauchy-ttel szerint T(n
2,1,0, ...,0) =
n!
(n2)!2
1
=
n(n1)
2
. Megllapods szerint (*) [
n
0
] = 0 minden n 1-re s (**) [
0
0
] = 1.
A denci alapjn azonnali, hogy
n

k=1
_
n
k
_
= n! (n 1).
I.8.2.3. Ttel. Ha 1 k n, akkor
_
n
k
_
= (n1)
_
n1
k
_
+
_
n1
k1
_
.
Bizonyts. Tegyk fel, hogy 2 k n. Tekintsk a k ciklust tartalmaz n-edfok permutcikat.
Szmoljuk meg ezeket aszerint, hogy n xpont-e, azaz (n) ciklus-e vagy sem.
Ha (n) ciklus, akkor marad n1 elem, amely k1 ciklust alkot. Az ilyen permutcik szma
[
n1
k1
].
Ha (n) nem ciklus, akkor n tagja egy legalbb 2 hosszsg ciklusnak. Legyen egy k
ciklus (n1)-edfok permutci. Az n-et berhatjuk brmely a szm utn a ciklusokra val
felbontsban, s gy egy n-edfok permutcit kapunk, amelyben n nem xpont. Itt [
n1
k
] lehetsg
van megvlasztsra s (n1) lehetsg van az a szm megvlasztsra. Az ilyen permutcik
szma teht (n1)[
n1
k
].
A k ciklust tartalmaz n-edfok permutcikat szma teht [
n1
k1
] +(n1)[
n1
k
].
Ha most k =1, akkor [
n
1
] =(n1)[
n1
1
] +[
n1
0
]. Innen n 2-re a fenti (*) megllapods szerint
1 = 1 +0, ami igaz, n = 1-re pedig (**) szerint 1 = 0 +1, igaz (ezrt clszerek a (*) s (**)
konvencik).
Az I.8.2 tblzat az elsfaj Stirling-szmokat tartalmazza. A rekurzv kplet szerint itt minden
[
n
k
] bels szm egyenl a felette ll szm (n1)-szeresnak s az attl egy hellyel balra ll szm
sszegvel.
I.8.2.4. Feladat. Adjuk meg a 6. Tblzat kvetkez hrom sort.
I.8.2.5. 16.11. Feladat. i) Adjuk meg azokat a harmadfok permutcikat, amelyek k = 1, illetve
k = 2 ciklust tartalmaznak.
ii) Adjuk meg azokat a negyedfok permutcikat, amelyek k=2, illetve k=3 ciklust tartalmaznak.
Megolds. i) Ezek szma [
3
1
] = 2, ill. [
3
2
] = 3. A permutcik pedig (1 2 3), (1 3 2), ill. (1)(2 3),
(2)(1 3), (3)(1 2).
ii) Ezek szma [
4
2
]=11, ill. [
4
3
]=6. A permutcik pedig (1 2)(3 4), (1 3)(2 4), (1 4)(2 3), (1)(2 3 4),
(1)(2 4 3), (2)(1 3 4), (2)(1 4 3), (3)(1 2 4), (3)(1 4 2), (4)(1 2 3), (4)(1 3 2), ill. (1)(2)(3 4), (1)(3)(2 4),
(1)(4)(2 3), (2)(3)(1 4), (2)(4)(1 3), (3)(4)(1 2).
58 I.8. FEJEZET. STIRLING-SZMOK
n
_
n
1
_
n
2
_
n
3
_
n
4
_
n
5
_
n
6

1 1
2 1 1
3 2 3 1
4 6 11 6 1
5 24 50 35 10 1
6 120 274 225 85 15 1
I.8.2. tblzat. Elsfaj Stirling-szmok
Az [x]
n
= x(x1) (xn+1) polinom egytthati az elsfaj Stirling-szmok segtsgvel
adhatk meg a kvetkezkppen:
I.8.2.6. Ttel. Ha n 1, akkor
[x]
n
=
n

k=1
(1)
nk
_
n
k
_
x
k
.
Bizonyts. Ellenrzs: n = 1-re x =
_
1
1

x x = x igaz, n = 2-re [x]


2
=
_
2
1

x+
_
2
2

x
2
x(x
1) =x+x
2
igaz, n = 3-ra [x]
3
=
_
3
1

x
_
3
2

x
2
+
_
3
3

x
3
x(x1)(x2) = 2x3x
2
+x
3
igaz.
Az I.8.2.3 Ttelbeli rekurzi alapjn n szerinti indukcival. Tegyk fel, hogy a kplet igaz
n1-re s igazoljuk n-re, ahol n 2.
n

k=1
(1)
nk
_
n
k
_
x
k
=
_
n
n
_
x
n
+(n1)
n1

k=1
(1)
nk
_
n1
k
_
x
k
+
n1

k=2
(1)
nk
_
n1
k1
_
x
k
,
ahol
_
n
n

=
_
n1
n1

= 1, gy
_
n
n
_
x
n
+
n1

k=2
(1)
nk
_
n1
k1
_
x
k
=
n

k=2
(1)
nk
_
n1
k1
_
x
k
= x
n1

j=1
(1)
n1j
_
n1
j
_
x
j
.
Kvetkezik, hogy
n

k=1
(1)
nk
_
n
k
_
x
k
= (xn+1)
n1

j=1
(1)
n1j
_
n1
j
_
x
j
= (xn+1)[x]
n1
= [x]
n
,
az indukcis felttel szerint.
Az itt fellp (1)
nk
_
n
k

szmokat szoks elsfaj algebrai Stirling-szmoknak nevezni, a


_
n
k

szmokat pedig elsfaj abszolt Stirling-szmoknak. Ms szerzk az elbbieket nevezik


elsfaj Stirling-szmoknak.
I.8.2.7. Feladat. Ha n 1, akkor
[x]
n
=
n

k=1
_
n
k
_
x
k
,
ahol [x]
n
=x(x+1) (x+n1).
I.8.2. ELSFAJ STIRLING-SZMOK 59
Megolds. Azonnal kvetkezik x helyett (x)-et rva az I.8.2.6 Ttelbeli kpletbe.
I.8.2.8. Feladat. Jellje H
n
= 1 +
1
2
+
1
3
+... +
1
n
az n. harmonikus szmokat, ahol n 1.
Igazoljuk, hogy minden n 1-re H
n
=
_
n+1
2
_
/n! .
Megolds. Ellenrzs: n = 1-re H
1
= 1 =
_
2
2

igaz, n = 2-re H
2
= 3/2 =
_
3
2

/2 igaz, n = 3-ra
H
3
=11/6=
_
4
2

/6 igaz. Tegyk fel, hogy a kplet igaz n1-re s igazoljuk n-re, ahol n2. rhat,
hogy
H
n
=H
n1
+
1
n
=
_
n
2

(n1)!
+
1
n
=
n
_
n
2

+(n1)!
n!
=
n
_
n
2

+
_
n
1

n!
=
_
n+1
2

n!
,
hasznlva az elsfaj Stirling-szmokra vonatkoz rekurzit.
Megjegyzs. Igazolhat, hogy ha n 2, akkor H
n
nem egsz szm, s lim
n
(H
n
ln n) =, ahol
0,5772 az n. Euler-lland.
60 I.8. FEJEZET. STIRLING-SZMOK
I.9. fejezet
Grfelmleti fogalmak
I.9.1. A grfok szemlletes bevezetse
Tekintsk a kvetkez feladatokat:
I.9.1.1. Feladat. Egy 21 tag trsasg bizonyos tagjai kzfogssal dvzltk egymst. Igazoljuk,
hogy biztosan van olyan szemly, aki pros szm emberrel fogott kezet.
I.9.1.2. Feladat. Igazoljuk, hogy ebben a 21 tag trsasgban van legalbb kt ember, akik
ugyanannyiszor fogtak kezet.
I.9.1.3. Feladat. Mutassuk meg, hogy egy 6 fs trsasgnak mindig van 3 olyan tagja, akik
ismerik egymst, vagy 3 olyan tagja, akik nem ismerik egymst (az ismerettsgek klcsnsek.)
I.9.1.4. Feladat. Knigsberg vrosban (mai nevn Kalinyingrd) a Pregel folyn 7 hd vezetett
keresztl az I.9.1 bra szerint. Vgig lehetett-e stlni minden hdon pontosan egyszer gy, hogy a
sta vgre visszarkezznk a kiindulpontba?
Ezek a feladatok, s ezeken kvl sok ms, ltszlag teljesen klnbz jelleg problma
ugyanazzal a fontos kombinatorikai struktrval rhat le s vizsglhat. Ennek a neve: grfstruktra
vagy grf.
Ebben a szakaszban s a ksbbiekben visszatrnk ezekre a feladatokra s bemutatjuk a
megoldsukat.
Az I.9.1 brn jellje A, B s C a partokat, D a szigetet, a hidakat pedig jelljk a pontokat
sszekt vonalakkal. gy a kvetkez sematikus brt kapjuk (I.9.2 bra).
Az ilyen s ehhez hasonl alakzatokat grfoknak nevezzk, itt A, B, C, D az adott grf
cscsai (vagy pontjai, cscspontjai, csompontjai, szgpontjai), a cscsokat sszekt vonalak a
I.9.1. bra. Knigsbergi hidak
61
62 I.9. FEJEZET. GRFELMLETI FOGALMAK
I.9.2. bra. A knigsbergi hidak grfja
grf lei. Megtrtnhet, hogy a skbeli brn az lek olyan pontban is metszik egymst, amely
nem cscsa a grfnak.
A grf cscsait vastagtott pontokkal jelezzk. Kt cscs kztt tbb l is lehet, ezeket tbbszrs
leknek nevezzk. Egy cscs nmagval is ssze lehet ktve, az ilyen l neve hurokl.
Az I.9.1.4 Feladatban megfogalmazott krdst Knigsberg laki tettk fel Eulernek, a kor hres
matematikusnak. A fenti krds gy is megfogalmazhat: le tudjuk-e rajzolni ezt a grfot egy
vonallal gy, hogy kzben a ceruzt nem emeljk fel s minden vonalat csak egyszer rajzolunk le?
Euler vlaszknt igazolta (ltalnosan vizsglva a krdst), hogy nem lehet vgigmenni a
hidakon, azaz nem tudjuk lerajzolni a grfot a felttel szerint. Ugyanis brmely pontbl is indulunk
ki, ha egy msik pontba berkeznk, onnan ki is kell menni. Lehet, hogy utunk sorn jra rintjk
ezt a pontot, de akkor is kt l bejrsval. Ezrt szksges, hogy minden pontbl pros sok l
induljon ki. Ez nem teljesl a fenti grfra, mert minden ponthoz pratlan sok l tartozik.
Euler azt is megmutatta, hogy ez a felttel (minden pontbl pros sok l indul ki) nemcsak
szksges, hanem elgsges is a grf ilyen bejrshoz.
A grfelmlet kialakulst 1736-tl szmtjuk, attl a dolgozattl, amelyben Euler a knigsbergi
hidak problmjt megoldotta.
Egy grf valamely pontjnak fokszma vagy foka az adott pontbl kiindul lek szma. Jells:
d(A) a A pont fokszma. A I.9.2 brn pl. d(A) = d(B) = D(C) = 3, d(D) = 5. Egy grf izollt
pontja egy olyan pont, amelybl nem indul ki egyetlen egy l sem. Az izollt pontok teht a 0
fokszm pontok, az I.9.2 brn nincsenek ilyenek. Az I.9.3 brn lthat grfon d izollt pont,
az a s f pontokban egy-egy hurokl van.
Jellje [E[ egy adott grf leinek a szmt.
I.9.1.5. Ttel. Tekintsnk egy tetszleges n pont grfot, amelynek pontjai P
1
, P
2
, ..., P
n
.
i) A pontok fokszmainak sszege egyenl az lek szmnak a ktszeresvel : d(P
1
)+d(P
2
)+...+
+d(P
n
) = 2[E[.
ii) A pratlan fokszm pontok szma pros.
Bizonyts. i) Vegyk sorra a pontokat s szmoljuk ssze a pontokhoz tartoz leket. Ezek szma
sszesen a pontok fokszmainak sszege, msrszt ppen az lek szmnak ktszerese.
ii) Azonnali i) alapjn.
Vizsgljuk az I.9.1.1 Feladatot. Tekintsk azt a grfot, amelynek pontjait a trsasg tagjai
alkotjk, az leket pedig a kzfogsok. Ez egy 21 pont grf. Alkalmazva az elz Ttel ii) pontjt
I.9.1. A GRFOK SZEMLLETES BEVEZETSE 63
I.9.3. bra. Plda grfra
azt kapjuk, hogy pros azoknak a szma, akik pratlan sok emberrel fogtak kezet. De 21 pratlan
szm, ezrt biztosan van olyan szemly, akire ez nem igaz, aki teht pros sok emberrel fogott
kezet.
Az I.9.1.1 Feladatban megfogalmazott llts nemcsak egy 21 tag trsasgra, hanem egy n
tag trsasgra is igaz, ahol n > 1 tetszleges pratlan szm. Az is igaz, hogy ekkor pratlan
azoknak a szma, akik pros szm emberrel fogtak kezet.
Egy olyan grfot, amelyben nincsenek huroklek s tbbszrs lek egyszer grfnak neveznk,
lsd pl. az I.9.4 brt.
I.9.4. bra. Plda egyszer grfra
I.9.1.6. Ttel. Egy n pont egyszer grfban mindig van kt azonos fokszm pont.
Bizonyts. Tegyk fel, hogy ez nem igaz. Akkor a pontok fokszmai mind klnbzk. Egy pont
legfeljebb n1 msik ponttal lehet sszektve, ezrt a pontok fokszmai 0,1,2, ..., n1. De gy
van olyan pont, amely foka 0 (izollt pont) s van olyan pont is, amely foka (n1), azaz olyan,
amely minden ms ponttal ssze van ktve. Ez ellentmonds.
Trjnk r az I.9.1.2 Feladat megoldsra. A megfelel grf egyszer grfnak tekinthet (feltve,
hogy kt ember legfeljebb egyszer fog kezet s senki sem fog kezet sajt magval) s alkalmazzuk
az I.9.1.5 Ttelt. Az llts egy n tag trsasgra is igaz, ahol n 2 tetszleges szm. Ha n = 21,
akkor az I.9.1.2 Feladatot kapjuk.
Sok ms olyan problma is van, amelyek grfokkal modellezhetk, pl. thlzatok, telefonhlzatok,
elektromos ramkrk. Bizonyos problmkra iranytott grfokkal trtnik a modellezs, azaz
64 I.9. FEJEZET. GRFELMLETI FOGALMAK
olyan grfokkal, amelyeknek lei irnytottak. Sok esetben a grfelmleti mdszerek s eredmnyek
alkalmasak a felmerl problmk megvlaszolsra.
Irnytott grfnak tekinthet a vilghl is, a World Wide Web, amelynek pontjai a weblapok,
az lei pedig a weblapok kztti hiperlinkek. A Google s a hasonl keresk bonyolult grfelmleti
tleteket s mdszereket hasznlnak, hogy ennek a grfnak a pontjaira egy sorrendet lltsanak
fel gyelembe vve a grf globlis jellemzit.
I.9.2. Egyszer grfok
A tovbbiakban egyszer grfokkal foglalkozunk, ezek matematikai dencija a kvetkez:
I.9.2.1. Denci. Legyen V egy nemres halmaz s legyen V
(2)
=x, y : x, y V, x ,=y a V
ktelem rszhalmazainak halmaza. A G = (V, E) rendszert, ahol E V
(2)
egyszer grfnak, a
tovbbiakban rviden grfnak nevezzk. Itt V elemei a grf pontjai (vagy cscsai, cscspontjai),
E elemei a grf lei.
A G grfra vonatkozan jells: V =V
G
, E=E
G
, teht G=(V
G
, E
G
). Tovbbi jells: x, y=xy,
ahol xy = yx, s x V
G
helyett nha ezt rjuk: x G.
Ha e = xy E
G
a grf egy le, akkor azt mondjuk, hogy x s y az l vgpontjai. Az u s v
pontok szomszdos pontok, ha ltezik egy uv E
G
l. Az e
1
, e
2
E
G
szomszdos lek, ha van
kzs vgpontjuk, azaz e
1
=xy, e
2
=xz valamely x, y, z V -re.
Megjegyzs: Angolul a grf pontjai = [vertex, vertices], lek = [edge, edges], innen a V s E
jellsek.
Itt s a tovbbiakban egy A vges halmaz szmossgt (elemeinek szmt) gy jelljk: [A[
vagy #A.
Ha V vges halmaz, akkor [V
G
[, [E
G
[ vgesek s vges grfrl beszlnk, a tovbbiakban
ilyenekrl lesz sz. A V
G
halmaz [V
G
[ szmossgt, teht a grf pontjainak szmt a grf rendjnek
is nevezzk. Ha [V
G
[ = n, akkor azt mondjuk, hogy G egy n pont grf. Ha V vgtelen halmaz,
akkor G egy vgtelen grf.
A grfok brzolhatk a skban. Pl. az I.9.4 bra annak a G grfnak a kpe, amelyre V
G
=
=x
1
, x
2
, x
3
, x
4
, x
5
, x
6
, x
7
, x
8
, x
9
,
E
G
=x
1
x
2
, x
1
x
5
, x
1
x
7
, x
2
x
5
, x
3
x
5
, x
4
x
5
, x
4
x
6
, x
4
x
7
, x
5
x
7
, x
6
x
7
, x
6
x
8
, x
6
x
9
, x
7
x
8
,
ez egy 9 pont grf, az lek szma 13.
Egy x pont foka (vagy fokszma) az x-szel szomszdos pontok szma, jells: d
G
(x) = d(x),
azaz d
G
(x) = #y G: xy E
G
. Az x izollt pont, ha d(x) = 0.
A fokszmokra vonatkoz I.9.1.5 s I.9.1.6 Ttelek igazak egyszer grfokra, az elz szakaszban
adott bizonytsok megfelelnek az I.9.2.1 Dencinak.
I.9.2.2. Denci. Legyen G s H kt grf. Azt mondjuk, hogy ezek izomorf grfok, jells
G H, ha ltezik egy olyan : G H bijektv fggvny, amelyre xy E
G
(x)(y) E
H
(minden x, y E
G
-re), azaz xy akkor s csak akkor le G-nek, ha (x)(y) le H-nak.
Pldul az I.9.5 brn lev grfok izomorfak, itt egy izomorzmus a kvetkez: x
1
2, x
2
3,
x
3
5, x
4
1, x
5
4.
I.9.2.3. Feladat. Mennyi az n pont grfok szma?
Megolds. Annyi n pont grf van, ahnyflekppen az lek megvlaszthatk. Az lek maximlis
szma [V
(2)
[ =
_
n
2
_
. Kvetkezik, hogy az n pont grfok szma a V
(2)
halmaz rszhalmazainak a
szma, azaz 2
(
n
2
)
= 2
n(n1)/2
(n 1).
I.9.2. EGYSZER GRFOK 65
I.9.5. bra. Plda izomorf grfokra
I.9.2.4. Feladat. Rajzoljuk le az n pont grfokat, ahol n=1,2,3. Azonostsuk az izomorf grfokat.
Az I.9.1 tblzat azt mutatja, hogy adott n-re az n pont nemizomorf grfok szma jval
kisebb az sszes grf szmnl.
n 1 2 3 4 5 6 7 8
grfok szma 1 2 8 64 1 024 32 768 2 097 152 268 435 456
nemizomorf grfok szma 1 2 4 11 34 156 1044 12 346
I.9.1. tblzat. Az n pont grfok szma
I.9.2.5. Feladat. Rajzoljuk le az n = 4 pont nemizomorf grfokat (ezek szma 11).
A pontok fokszmai nem hatrozzk meg a grfot. Megtrtnhet, hogy a G = (V, E
G
) s H =
= (V, E
H
) grfokra d
G
(x) = d
H
(x) minden x V -re, de G s H nem izomorf grfok.
I.9.2.6. Feladat. Adjunk pldt ilyen G s H grfokra.
Tovbbi dencik s jellsek:
I.9.2.7. Denci. Legyen G egy grf. Azt mondjuk, hogy
G l nlkli grf, ha nincs egy le sem,
G teljes grf, ha E
G
= x, y : x, y V, x ,= y, azaz ha brmely kt pont szomszdos (ssze
van ktve llel). Az n pont teljes grf jellse: K
n
(komplett grf ),
G csillag, ha gy kapjuk, hogy egy pontjt sszektjk az sszes tbbivel,
G t, ha gy kapjuk, hogy megllaptva a pontok egy sorrendjt az els pontot sszektjk a
msodikkal, a msodikat a harmadikkal,..., az utols elttit az utolsval,
G kr, ha egy tnak az els s utols pontjt is sszektjk,
G regulris grf, ha minden pontja egyenl fokszm, ha ez a fokszm r, akkor G egy r-
regulris grf.
A K
n
teljes grf egy (n1)-regulris grf. Az I.9.6 brn a K
8
teljes grf lthat. A 3 pont
teljes grfot hromszgnek is nevezzk.
Az I.9.7 brn a 10-edred 3-regulris grf, az n. Petersen-grf szerepel.
I.9.2.8. Feladat. i) Hny le van az n pont csillagnak, tnak, krnek, illetve a K
n
teljes grfnak?
ii) Rajzoljuk meg a K
n
teljes grfokat, ahol 1 n 7.
iii) Adjunk meg egy 2-regulris grfot s egy 4-regulris grfot.
iv) Ltezik-e 3-regulris 5 pont grf ? s 7 pont?
66 I.9. FEJEZET. GRFELMLETI FOGALMAK
I.9.6. bra. K
8
grf
I.9.7. bra. Petersen-grf
I.9.2.9. Ttel. Ha n 4 pros szm, akkor ltezik n pont 3-regulris grf.
Bizonyts. n = 4-re a K
4
teljes grf 3-regulris. Legyen n = 2k. k-szerinti indukcival tegyk fel,
hogy G egy 2k2 pont 3-regulris grf. Legyen xy, xz E
G
a G kt egy pontbl kiindul kt le.
Vegynk kt jabb pontot, jellje ezeket u, v s kpezzk a H grfot, amelyre V
H
= V
G
u, v
s E
H
= (E
G
xy, xz) xu, xv, yu, zv, uv. Kvetkezik, hogy H egy 2k pont 3-regulris grf
(lsd I.9.8 bra).
I.9.2.10. Denci. Legyen G=(V
G
, E
G
) egy grf. Azt mondjuk, hogy a G grfnak H rszgrfja,
ha V
H
V
G
s E
H
E
G
. Jells: H G.
A H rszgrf kifeszti a G grfot, ha G minden pontja H-ban van, azaz V
H
=V
G
. Ekkor a G
grfnak H egy feszt rszgrfja.
G kiegszt grfja (komplementer grfja) a G= (V
G
, V
(2)
E
G
) grf.
Egy adott grfbl rszgrfot teht gy kapunk, hogy bizonyos pontokat s az ezekhez tartoz
sszes lt trljk. A kiegszt grfot gy kapjuk, hogy trljk G meglv leit s megrajzoljuk
az sszes tbbi lehetsges let (a pontokat nem vltoztatjuk).
I.9.2.11. Feladat. Adjuk meg a 10. s 13. brn lev grfok egy-egy rszgrfjt s kiegszt
grfjait.
I.9.2.12. Feladat. i) Igazoljuk, hogy ha G egy n pont grf, ahol n6, akkor ltezik G-ben vagy
G-ben hromszg.
ii) Igaz-e ez a tulajdonsg az 5 pont grfokra?
iii) Kapcsolat az I.9.1.3 Feladattal ?
I.9.2. EGYSZER GRFOK 67
I.9.8. bra. Az I.9.2.9 Ttel bizonytshoz
Megolds. i) A G grf leit szinezzk pirosra, a G komplementer grf leit pedig kkre. Legyen
a egy tetszleges pont, amelybl legalbb 5 l hzhat, mert n 6. Kvetkezik, hogy ezek kztt
biztosan van 3 azonos szn ab, ac, ad l, teht vagy mind a hrom piros (G-beli lek) vagy mind
a hrom kk (G-beli lek). Legyenek pl. ab, ac, ad piros lek. Ha bc vagy bd vagy cd piros l, akkor
van egy piros hromszg (ha pl. bd piros, akkor abd piros hromszg). Ellenkez esetben bcd kk
hromszg.
ii) Az llts 5 pont esetn nem igaz. Ehhez elegend egyetlen ellenpldt mutatnunk. Tekintsk
azt az tcscs egyszer grfot, melyben minden cscsbl pontosan kt l s kt nem-l indul ki.
(Az elz jellssel : Rajzoljunk olyan tcscs teljes grfot, melyben minden cscsbl pontosan kt
piros s kt kk l fut, lsd I.9.9 bra.) Ez a grf hromszgmentes (s izomorf a komplementervel).
I.9.9. bra. Ellenplda 5 pont grf esetn
iii) n = 6-ra ppen az I.9.1.3 Feladat lltst kapjuk!
I.9.2.13. Denci. Legyenek e
1
=x
0
x
1
, e
2
=x
1
x
2
,..., e
k
=x
k1
x
k
egy G grf egymshoz csatlakoz
klnbz lei. Azt mondjuk, hogy W = e
1
e
2
...e
k
egy k hosszsg sta, amelynek x
0
s x
k
a
vgpontjai (angolul sta = walk).
A W zrt sta (krsta), ha x
0
=x
k
, ellenkez esetben W nyitott sta.
A W sta nyitott t, ha az x
0
, x
1
, ..., x
k
pontok pronknt klnbzk.
A W sta zrt t (krt) vagy kr, ha az x
0
, x
1
, ..., x
k
pontok pronknt klnbzk, kivve
x
0
=x
k
(ennek hossza k 3).
Egy stt teht gy kapunk, hogy kiindulunk egy pontbl egy l mentn, berkeznk egy msik
pontba, onnan tovbb haladunk egy msik l mentn, s gy tovbb, mg eljutunk a sta msik
68 I.9. FEJEZET. GRFELMLETI FOGALMAK
I.9.10. bra. Plda grfra
vgpontjba. A bejrt lek nem ismtldhetnek. Ha az rintett pontok sem ismtldnek, kivve
esetleg a vgpontokat, akkor trl beszlnk.
Pldul az I.9.10 brn lev 20 pont grfban v
2
v
7
v
12
v
11
v
6
v
7
v
8
egy nyitott
sta. Pontosabban, megadva az leket, (v
2
v
7
)(v
7
v
12
)(v
12
v
11
)(v
11
v
6
)(v
6
v
7
)(v
7
v
8
) egy nyitott sta,
amelynek hosszsga 6, vgpontjai v
2
s v
8
. Ez nem t, mert a sta a v
7
pontot ktszer is rinti.
A v
2
-bl indul s ide visszarkez v
2
v
7
v
12
v
11
v
6
v
7
v
8
v
9
v
4
v
3
v
2
, pontosabban
(v
2
v
7
)(v
7
v
12
)(v
12
v
11
)(v
11
v
6
)(v
6
v
7
)(v
7
v
8
)(v
8
v
9
)(v
9
v
4
)(v
4
v
3
)(v
3
v
2
) sta egy zrt sta, amelynek hosszsga
10. Ez sem t.
A v
1
v
2
v
3
v
4
v
5
-nek megfelel (v
1
v
2
)(v
2
v
3
)(v
3
v
4
)(v
4
v
5
) sta egy 4 hosszsg nyitott t.
Av
2
v
3
v
4
v
9
v
8
v
7
v
2
-nek megfelel (v
2
v
3
)(v
3
v
4
)(v
4
v
9
)(v
9
v
8
)(v
8
v
7
)(v
7
v
2
) egy 6 hosszsg
zrt t, azaz kr.
I.9.2.14. Feladat. i) Igazoljuk, hogy ha egy grfban minden pont foka legalbb 2, akkor a grfban
van egy kr.
ii) Mit lehet mondani a grfrl, ha minden pont foka 2?
Megolds. i) Induljunk ki a grf egy tetszleges pontjbl s haladjunk az leken. Minden
pont foka 2, ezrt brmely j pontba jutva tovbb mehetnk mg be nem jrt len (gy egy stt
kapunk). Csak akkor akadhatunk el, ha olyan pontba jutunk, amelyet korbban mr rintettnk.
De akkor bejrtunk a grfban egy krt.
I.9.2.15. Ttel. Legyen G egy n pont grf. Ha G-nek van legalbb n le, akkor G-ben van kr.
Bizonyts. Ha n=1 vagy n=2, akkor a felttel nem teljeslhet (egyszer grfrl van sz). Ha n=3,
akkor a grfot 3 pont s 3 l alkotja, ez egy hromszg, s az llts igaz. n-szerinti indukcival
tegyk fel, hogy a tulajdonsg igaz minden n1 pont grfra. Legyen G egy olyan n pont grf,
amelynek van legalbb n le. Tekintsnk a G-beli utak kzl egy leghosszabbat (biztosan van
ilyen).
Ha ez egy zrt t, akkor egy kr, s kszen vagyunk.
Ha ez nyitott t, akkor mindkt vgpontja elsfok. Valban, ha nem gy lenne, akkor az egyik
vgponthoz jabb l csatlakoztatsval egy hosszabb utat kapnnk, ami ellentmond annak, hogy a
vlasztott t hossza maximlis. Trljk az egyik vgpontot a hozz tartoz llel egytt. Kapunk
egy olyan n1 pont grfot, amelynek van legalbb n1 le. Az indukcis felttel szerint ebben
van kr, s ez G-nek is rsze.
I.9.2.16. Denci. A G grf sszefgg, ha brmely kt u, v pontjra ltezik olyan t G-ben
(az t G-nek rszgrfja), amelynek vgpontjai u s v. Az egy pontbl ll grfot is sszefggnek
tekintjk.
I.9.3. FAGRFOK 69
A G maximlis, sszefgg rszgrfjait a G sszefgg komponenseinek nevezzk. Ugyanazon
komponenshez tartoznak azok a pontok, amelyek kztt van G-beli t (amelyek lekkel sszekthetk).
Megjegyzs. Az, hogy a grf kt u, v pontjra ltezik olyan t G-ben, amelynek vgpontjai u
s v egy relci a grf pontjainak V
G
halmazn. Ha megengedjk a 0 hosszsg utakat is, akkor
belthat, hogy ez egy ekvivalenciarelci a V
G
halmazon. Az gy meghatrozott ekvivalenciaosztlyok
lesznek a G sszefgg komponensei.
I.9.2.17. Feladat. Legyen G egy sszefgg grf s ebben legyen C egy kr. Akkor a C kr
brmely e lnek trlsvel kapott Ge grf is sszefgg.
Megolds. Legyen u, v a grf kt tetszleges pontja. Ha az ezeket sszekt tban szerepel a
trlt l, akkor vehetjk helyette a C krnek a trlt l elhagyasval kapott rszt.
I.9.2.18. Feladat. Legyen G egy n pont sszefgg grf (n2). Igazoljuk, hogy ha az lek szma
<n, akkor G-ben ltezik legalbb egy elsfok pont.
Megolds. Jellje G pontjait P
1
, P
2
, ..., P
n
. G sszefgg grf, ezrt nincs izollt pontja, azaz
d(P
i
) 1 minden i-re. Ha nem lenne elsfok pont, akkor d(P
i
) 2 minden i-re, s gy az I.9.1.5
Ttel szerint az lek szma [E[=(d(P
1
)+d(P
2
)+...+d(P
n
))/2(2n)/2=n lenne, ami ellentmonds.
I.9.2.19. Ttel. Brmely n pont sszefgg grfnak van legalbb n1 le (n 1).
Megolds. Ha n = 1,2,3, akkor ez igaz. n-szerinti indukcival tegyk fel, hogy minden n1
pont sszefgg grfnak van legalbb n2 le. Legyen G egy n pont sszefgg grf. Azt kell
beltnunk, hogy G-nek van legalbb n1 le. Tegyk fel, hogy ez nem gy van. Akkor G-nek
kevesebb, mint n1 le van, s az I.9.2.18 Feladat szerint van egy P elsfok pont. A P pontot a
hozz tartoz llel egytt trlve egy n1 pont sszefgg grfot kapunk, amelynek az indukcis
felttel szerint van legalbb n2 le. A trlt llel egytt akkor G-nek van legalbb n1 le, ami
ellentmonds.
A grfok reprezentlsra ksztett brk tetszetsek, a feladatok megoldsa sorn gyakran
ilyen brkat vzolva gondolkodunk, de ezek programozsi clokra nem hasznlhatk.
Egy lehetsg a grfok szmtgpes trolsra a kvetkez. Ha x
1
, x
2
..., x
n
egy G grf pontjai,
legyen A = (a
ij
)
1i,jn
az az nn-es mtrix, amelyre a
ij
= 1, ha x
i
x
j
E
G
(x
i
x
j
a grf le),
s a
ij
= 0 ellenkez esetben (x
i
x
j
a grfnak nem le). Ezt a grf szomszdsgi mtrixnak
nevezzk, amely szimmetrikus (a ftlra nzve) s a ftlban nullk vannak. Pl. az I.9.5 bra
els grfjnak szomszdsgi mtrixa:
A =
_
_
_
_
_
_
0 1 0 1 1
1 0 1 0 0
0 1 0 1 0
1 0 1 0 1
1 0 0 1 0
_
_
_
_
_
_
.
I.9.3. Fagrfok
I.9.3.1. Denci. Azokat az (egyszer) grfokat, amelyek sszefggek s nem tartalmaznak
krt fagrfoknak, rviden fknak nevezzk.
I.9.3.2. Ttel. Egy G grf akkor s csak akkor fa, ha brmely kt klnbz pontja kztt pontosan
egy t vezet.
70 I.9. FEJEZET. GRFELMLETI FOGALMAK
I.9.11. bra. Fagrfok
Bizonyts. Tegyk fel, hogy G egy fa. Akkor G sszefgg (denci), ezrt brmely kt klnbz
pontja kztt van t. Tbb t nem lehet, mert akkor G-ben lenne kr, ami ellentmonds.
Fordtva, tegyk fel, hogy G-ben brmely kt klnbz pont kztt pontosan egy t vezet.
Akkor G sszefgg (mert van t) s krmentes (mert csak egy t van), teht G fa.
Az I.9.11 brn nhny fagrf lthat, az utols egy csillag(grf), az eltte lev egy t(grf).
A fkat denil kt tulajdonsg azt vonja maga utn, hogy a fknak nem lehet tl kevs lk,
hiszen sszefggek, de tl sok lk se lehet, mert krmentesek. Mskpp: A fagrfok minimlis
sszefgg grfok s maximlis krmentes grfok. Pontosabban:
I.9.3.3. Ttel. Legyen G egy egyszer grf. Akkor egyenrtkek a kvetkez felttelek:
1) G fa(grf ),
2) G sszefgg, de ha brmely lt trljk, akkor nem marad sszefgg,
3) G krmentes, de brmely j l hozzvtele esetn a keletkez grf tartalmaz krt.
Bizonyts. 1) 2) Tegyk fel, hogy G egy fa. Akkor G sszefgg (denci). Tegyk fel, hogy
egy xy lt trlve a kapott G

grf sszefgg marad. Eszerint a G

grfban van x-et s y-t


sszekt P t. De akkor az xy lt visszatve a P t s az xy l egy krt alkot, ami ellentmond a
fa dencijnak.
2) 1) Tegyk fel, hogy teljesl a 2)-beli tulajdonsg. Akkor G sszefgg s azt kell
megmutatnunk, hogy G-ben nincs kr. Indirekt mdon, tegyk fel, hogy G-ben van egy C kr.
Akkor C-nek egy lt elhagyva a grf sszefgg marad, lsd I.9.2.17 Feladat, ami ellentmonds.
I.9.3.4. Feladat. Igazoljuk az I.9.3.3 Ttelben az 1) 3) s 3) 1) implikcikat.
I.9.3.5. Ttel. Ha G egy n pont fa, akkor G-nek (n1) le van.
Bizonyts. Ha G egy n pont fa, akkor G sszefgg, ezrt G-nek van legalbb (n1) le, lsd
I.9.2.19 Ttel. Ugyanakkor G krmentes, ezrt G-nek legfeljebb (n1) le van, lsd 10.2.15. Ttel.
Kvetkezik, hogy G-nek pontosan (n1) le van.
I.9.3.6. Ttel. Brmely n 2 pont fnak van legalbb kt elsfok pontja.
Bizonyts. Induljunk ki a grf egy tetszleges pontjbl s haladjunk az leken gy, hogy egy
j pontba rkezve mindig mg be nem jrt len megynk tovbb. Nem trtnhet meg, hogy
olyan pontba rjnk, amelyet korbban mr rintettnk, mert akkor bejrtunk volna egy krt,
ami lehetetlen. Csak akkor akadhatunk el, ha egy elsfok pontba jutunk (vesd ssze az ref18.14
Feladat megoldsval).
Ezzel igazoltuk, hogy ltezik legalbb egy elsfok pont. Hogyan mutathat meg, hogy biztosan
van mg egy elsfok pont?
I.9.3. FAGRFOK 71
Ha a tekintett fa egy t, akkor ennek pontosan kt elsfok pontja van, a vgpontjai.
I.9.3.7. Feladat. Rajzoljuk le az n pont fkat, ahol n = 1,2,3,4. Mennyi ezeknek a szma?
Azonostsuk az egymssal izomorf fkat. Mennyi ezeknek a szma?
Krdsek: Hny n pont fa ltezik? Hny n pont, egymssal nem izomorf fa ltezik?
Tekintsk pl. az I.9.12 brn lev kt 5 pont ft. Ha e kt fnak ugyanolyan sorrendben
megszmozzuk (megcimkzzk) a pontjait, pl. az 1,2,3,4,5 szmokkal, akkor azt mondjuk, hogy
ezek klnbz cimkzett fk (vagy szmozott fk). ltalban kt n pont cimkzett fa akkor
klnbz, ha van olyan (i, j) szmpr, hogy az egyik fban van ij l, a msikban pedig nincs.
Ha elhagyjuk a pontok szmozst, akkor cimkzetlen fkrl (vagy szmozatlan fkrl)
beszlnk. Kt cimkzetlen fa akkor klnbz, ha nem izomorfak. Az I.9.12 brn lev kt fa
egymssal izomorf, ezrt cimkzetlen fkknt tekintve ket nem klnbzek.
I.9.12. bra. Plda izomorf fagrfokra
Ennek megfelelen az elbbi krdsek gy is megfogalmazhatk:
Mennyi az n pont cimkzett fk szma? Mennyi az n pont cimkzetlen fk szma?
Az I.9.2 tblzat az n pont cimkzett s cimkzetlen fk szmt tartalmazza.
n 1 2 3 4 5 6 7 8
cimkzett fk szma 1 1 3 16 125 1 296 16 807 262 144
cimkzetlen fk szma 1 1 1 2 3 6 11 23
I.9.2. tblzat. Az n pont fagrfok szma
Megllapthat, hogy adott n-re az n pont cimkzetlen fk szma jval kisebb a cimkzett
fk szmnl.
Jellje x
n
az n pont cimkzett fk szmt Az I.9.2 tblzat szerint x
2
= 1 = 2
22
, x
3
= 3 =
=3
32
, x
4
=16=4
42
, x
5
=125=5
52
. Innen mr megsejthet a kvetkez, Cayley-ttelknt ismert
eredmny: x
n
= n
n2
minden n 1-re (ez n = 1-re is igaz). Ez egy meglepen egyszer kplet,
amelyet bizonytani fogunk, a bizonyts viszont nem olyan egyszer.
I.9.3.8. Feladat. Hny n pont cimkzett csillag ltezik? Hny n pont cimkzett t ltezik?
Megolds. A vlasz: n, illetve n! /2. Mirt pp ennyi ?
Az n pont cimkzetlen (teht nemizomorf) fk T
n
szmra nincs hasonl egyszer kplet,
csak becslsek adhatk. Mivel egy n pont cimkzetlen fa legfeljebb n!-flekppen cimkzhet
meg, azonnal kvetkezik, hogy T
n
x
n
/n! = n
n2
/n! minden n 1-re. Az is igazolhat, hogy
T
n
4
n1
minden n 1-re.
Az I.9.13 brn a 6 pont cimkzetlen fk lthatk, ezek szma T
6
= 6.
A tovbbiakban ismertetnk egy mdszert, amely lehetv teszi a cimkzett fk megadst a
megfelel bra nlkl, az n. Prfer-kd segtsgvel. A fa persze megadhat az lek felsorolsval
72 I.9. FEJEZET. GRFELMLETI FOGALMAK
I.9.13. bra. A 6 pont cimkzetlen fk
vagy a szomszdsgi mtrixszal is, de ez egy tmrebb reprezentci, amely ugyanakkor elvezet a
Cayley-ttel bizonytshoz is.
Tekintsnk egy n pont cimkzett grfot, a cimkk legyenek a 0,1,2, ..., n1 szmok. Akkor a
fa Prfer-kdja egy, a 0,1,2, ..., n1 szmok kzl (n2) szmbl ll k
1
k
2
... k
n2
alak kd,
ahol fontos a szmok sorrendje.
A 0 cimkj pontot a fa gykernek nevezzk, ez a fa tetszleges rgztett pontja lehet. A
gykrtl klnbz elsfok pontokat a fa leveleinek nevezzk. Biztosan van legalbb egy levl.
Mi tbb, ha a gykr nem elsfok, akkor ltezik legalbb kt levl, lsd I.9.3.6 Ttel.
A Prfer-kdot a kvetkezkppen kapjuk meg:
Megkeressk a levelek kzl a legkisebb cimkjt (t
1
) s lerjuk ennek a szomszdjt (k
1
).
Ez lesz a kd els szma (k
1
). Trljk a t
1
pontot a hozztartoz llel egytt. Megkeressk a
megmaradt fa levelei kzl a legkisebb cimkjt (t
2
) s lerjuk ennek a szomszdjt (k
2
). Ez lesz
a kd msodik szma (k
2
). Most trljk a t
2
pontot a hozztartoz llel egytt. Keressk meg
a megmaradt fa levelei kzl a legkisebb cimkjt (t
3
) s lerjuk ennek a szomszdjt (k
3
). Ezt
ismteljk (sorra letpjk a leveleket) mindaddig, amg egy kt pontbl ll fa nem marad. gy
sszesen (n2) pontot trltnk, a trlt pontok szomszdaibl ll kd teht valban (n2)
szmbl ll.
I.9.14. bra. Plda/1/Prfer-kd
Pldul, adott az I.9.14 brn lev n=9 pont fa. Elszr a 2 pontot kell trlni : t
1
=2, ennek
szomszdja a k
1
= 4, trljk az 5 pontot: t
2
= 5, ennek a szomszdja a k
2
= 4, tovbb t
3
= 6 s
k
3
=0, t
4
=7 s k
4
=3, t
5
=3 s k
5
=0, t
6
=8 s k
6
=1, t
7
=1 s k
7
=4 (rendre letpjk a megmaradt
fa leveleit). Ksztsnk egy tblzatot gy:
2 5 6 7 3 8 1
4 4 0 3 0 1 4
A kd teht ennek a msodik sora: 4 4 0 3 0 1 4 . Megjegyezzk, hogy gy vgl megmarad a
40 l, a 4 trlsvel a 0-t is lerhatnnk, de ez felesleges, mert mindig a 0 gykr az utols szm.
Ebben az eljsban az az igazn gyelemre mlt, hogy egy ilyen kd megadsval rekonstrulhat
a fa. Teht
I.9.3. FAGRFOK 73
I.9.3.9. Ttel. A Prfer-kd egyrtelmen meghatrozza a ft.
Bizonyts. Megmutatjuk, hogy a Prfer-kd ismeretben egyrtelmen megadhat a fa n1 le.
Tekintsk a kvetkez pldt. Adott az 5 2 4 1 0 2 3 1 kd. Ez 8 szmbl ll, ezrt n2 =8, teht
egy n =10 pont frl van sz. Melyek voltak vajon azok a t
1
, t
2
, ..., t
8
pontok, amelyek trlsvel
ezt kaptuk? Mi llhat az albbi tblzat els sorban?
t
1
t
2
t
3
t
4
t
5
t
6
t
7
t
8
5 2 4 1 0 2 3 1
Lehet-e t
1
= 1? Nem, mert mert ekkor az els lpsben trlnnk kellett volna, s nem
szerepelhetne a tblzat msodik sorban. Lehet-e t
2
= 2? Nem, ugyanezen oknl fogva. gy t
1
,=
= 1,2,3,4,5. Kvetkezik innen, hogy t
1
= 6? Igen, mert ha a 6 nem lenne elsfok pont, akkor
szomszdja lenne valaminek, s bekerlt volna a msodik sorba. Teht t
1
= 6.
Mi lehet t
2
? Az els oszlop elhagysval az 5 kikerl a msodik sorbl, s az 5 lesz az a legkisebb
szm, amely nem szerepel a msodik sorban. Teht t
2
=5. Tovbb hasonlan: t
3
=7, t
4
=4, t
5
=8,
t
6
= 9, t
7
= 2, t
8
= 3, azaz:
6 5 7 4 8 9 2 3
5 2 4 1 0 2 3 1
A szably teht ez: az els sor minden eleme az a legkisebb pozitv szm, amely nem szerepel
korbban az els sorban vagy ksbb a msodik sorban.
Hinyzik mg az 1, ez alkotja a 0 gykrrel az utols let. Ezzel kiegsztve kapjuk a bvitett
tblzatot:
6 5 7 4 8 9 2 3 1
5 2 4 1 0 2 3 1 0
Az oszlopok jelentik az leket s ennek alapjn rekonstrulhat a fa. Kezdjk az utols oszloppal
s haladjunk visszafel, gy egy egyre bvl ft kapunk(!), lsd az I.9.15 brt.
Ezek a meggondolsok teljesen ltalnosak s a tulajdonsg bizonytst adjk.
I.9.15. bra. Plda/2/Prfer-kd
I.9.3.10. Feladat. Melyik az a fa, amelynek a Prfer-kdja 5 3 2 3 4 1 7 2 0 3 ?
Nem lehetne egyszerbben kdolni ? Felejtsk el egy rvid idre a Prfer-kdot s tekintsk a
kvetkez feladatot:
74 I.9. FEJEZET. GRFELMLETI FOGALMAK
I.9.3.11. Feladat. Prblkozzunk a kvetkez egyszerbbnek tn kdolssal : Adott egy cimkzett
fa, ennek rgztett a gykere. Vegyk sorra az leket s rjuk le elbb a gykrtl tvolabbi pontot,
majd a gykrhez kzelebbi pontot. Az leket rendezzk az elsknt lert pontok nvekv sorrendjben.
Pl. az I.9.14 brbl kindulva ezt kapjuk:
1 2 3 4 5 6 7 8
4 4 0 0 4 0 3 1
A prbakd legyen a tblzat msodik sora: 4 4 0 0 4 0 3 1. Egy n pont fnak gy megfeleltetnk
n1 szmot.
a) Biztos az, hogy az els sorban mindegyik szm egyszer szerepel ?
b) Rekonstrulhat-e ennek a prbakdnak alapjn a fa? Vizsgljuk a kvetkez prbakdokat :
0 1 2 3 4 5 6 7 8, 8 7 6 5 4 3 2 1 0, 0 0 0 0 0 0, 3 1 2 5 4 5.
c) Mirt nem j ez a mdszer? Mirt nem lesz ez kd?
Trjnk vissza a Prfer-kdhoz s igazoljuk a Cayley-ttelt.
I.9.3.12. Ttel. (Cayley-ttel) Az n pont cimkzett fk szma x
n
= n
n2
, n 1 .
Bizonyts. Ahogy azt lttuk, a Prfer-kd bijektv megfeleltetst ltest az n pont cimkzett fk
s azok kztt az n2 tag szmsorozatok kztt, amelyek minden tagja a 0,1,2, ..., n1 szmok
valamelyike. Az ilyen sorozatok szma V
n2
n
=n
n2
(ismtlses varicik szma). Kvetkezik, hogy
ugyanennyi az n pont cimkzett fk szma is.
I.9.3.13. Denci. Egy krmentes, de nem felttlenl sszefgg grf neve erd. Rgtn addik,
hogy egy erd sszefgg komponensei fk (innen szrmazik az elnevezs).
I.9.3.14. Feladat. Hny le van egy n pont, k komponens erdnek?
I.9.4. Fesztfk, Kruskal-algoritmus
Ebben a szakaszban is egyszer grfokkal foglalkozunk.
I.9.4.1. Denci. Egy sszefgg grf fesztfjnak nevezzk a grf olyan feszt rszgrfjt,
amely fa (lsd az I.9.2.10 Dencit).
Mskpp: Egy sszefgg grf fesztfja a grf olyan rszgrfja, amely az adott grf minden
pontjt tartalmazza (a grf lei kzl csak bizonyos leket), s amely fa. A fesztfa ms elnevezse:
favz.
I.9.4.2. Ttel. Minden sszefgg grfnak ltezik fesztfja.
Bizonyts. Legyen G egy tetszleges sszefgg grf. Ha G krmentes, akkor G egy fa s G lesz
a fesztfa. Ebben az esetben G az egyedli fesztfa.
Ha G nem krmentes, akkor van benne egy C kr. A C kr egy e lt elhagyva G e
sszefgg marad, lsd I.9.2.17 Feladat. Ha Ge krmentes, akkor egy fa, s ez lesz egy fesztfa.
Ellenkez esetben hagyjuk el egy kr valamelyik lt. Ezt ismtelve vges sok lps utn a G egy
fesztfjhoz jutunk.
I.9.4. FESZTFK, KRUSKAL-ALGORITMUS 75
Ha egy G sszefgg grfban van kr, akkor G-ben tbb fesztfa is van, s ezek kztt nem
izomorfak is lehetnek.
Nzzk a kvetkez gyakorlati problmt: Adott n telepls, amelyek kztt vzvezetk hlzatot
akarnak ltesteni gy, hogy az minden teleplst ellsson az egyik A teleplsen lev vzforrsbl.
Nem akarjk felttlenl A-t sszektni az sszes tbbivel, a hlzaton keresztl azonban mindegyik
szmra biztostani akarjk a vzzel val elltst. Az egyes teleplsek kztti kzvetlen vezetkek
ptsi kltsgei ismertek (ezeket a tvolsg, a domborzati viszonyok s egybb tnyezk hatrozzk
meg).
Krds: Mely teleplseket kell kzvetlenl sszekapcsolni gy, hogy a teljes ptsi kltsg a
lehet legkisebb legyen?
Grfelmleti fogalmakat hasznlva: Adott egy n pont teljes grf, amelynek minden lhez
hozzrendelnk egy pozitv vals szmot, az ptsi kltsget.
Feladat: Hatrozzuk meg ennek a grfnak egy olyan rszgrfjt, amely
- sszefgg,
- minden pontot tartalmaz,
- leihez rendelt kltsgek sszege minimlis.
Rgtn addik, hogy a keresett rszgrf egy fa, mert sszefgg (felttel) s krmentes, hiszen
ha lenne benne kr, akkor annak legkltsgesebb lt megszntetve a rszgrf sszefgg marad,
az sszkltsg pedig cskken. Kvetkezik, hogy a keresett rszgrf az adott n pont teljes grf egy
fesztfja, amelynek n1 le van.
Hogyan lehet egy ilyen fesztft megadni ?
Tudjuk, hogy sszesen n
n2
szm n pont cimkzett fa ltezik (Cayley-ttel). Ha sorra
akarnnk venni a lehetsgeket, akkor pl. mr n = 10-re 10
8
= 100 000 000 lehetsges ft kellene
megvizsglnunk. Ennl egyszerbb s hatkonyabb mdszerre van szksg, s ilyen eljrs ltezik.
A gyakorlatban bizonyos teleplsek kztti kzvetlen vezetk (kapcsolat) valamilyen meggondols
miatt ki van zrva. A teljes grf helyett teht adott egy sszefgg grf, s ennek kell meghatrozni
egy minimlis kltsg fesztfjt. ltalban tbb ilyen fa ltezik.
Bemutatjuk a Kruskal-algoritmust, amely alkalmas ennek a feladatnak a megoldsra s amelynek
lnyege a kvetkez: kivlasztunk a legkisebb kltsg (legolcsbb) lek kzl egyet, majd ismtelten
a mg ki nem vlasztott, legolcsbb lek kzl kivlasztunk egy olyan lt, amely nem hoz ltre
krt a mr kivlasztott lekkel.
Pontosabban, legyen G egy n pont sszefgg grf, amely leinek halmaza E(G) s legyen
c : E(G) (0, ) egy kltsgfggvny a G lein. Hatrozzunk meg egy olyan F fesztft, amelyre
a c(F) =

eE(F)
c(e) teljes kltsg minimlis.
Kruskal-algoritmus:
Rendezzk az leket a kltsgek nvekv sorrendjben:
E(G) =e
1
, e
2
, ..., e
m
, ahol c(e
1
) c(e
2
) ... c(e
m
).
Legyen A = E(G), A lesz a kivlasztott lek halmaza.
Minden i=1,2, ..., mesetn az e
i
lt vagy kivlasztjuk, vagy sem az Ahalmazba a kvetkezkppen:
- Ha az Ae
i
lhalmaz nem tartalmaz krt, akkor az e
i
lt kivlasztjuk: A :=Ae
i
.
- Ha az Ae
i
lhalmaz tartalmaz krt, akkor az e
i
lt nem vlasztjuk ki, ekkor A nem
vltozik.
Az algoritmus egy olyan F rszgrfot eredmnyez, amelyet a G grf sszes pontja s az A-beli
lek alkotnak (E(F) = A).
I.9.4.3. Ttel. A Kruskal-algoritmus ltal eredmnyezett F rszgrf egy minimlis kltsg
fesztfa.
76 I.9. FEJEZET. GRFELMLETI FOGALMAK
Bizonyts. Az algoritmus sorn minden lps utn krmentes rszgrfot kapunk, gy a F output
rszgrf is krmentes.
Tegyk fel, hogy F nem sszefgg. Legyen F

az F egy sszefgg komponense. G sszefgg,


ezrt G-nek van olyan e
i
le, amelynek egyik vgpontja F

-beli, a msik vgpont pedig nem F-beli.


Ez az e
i
l nem kerlt kivlasztsra, azaz e
i
/ A. Ugyanakkor ez az e
i
l nem alkot krt F-ben,
ezrt az algoritmus sorn ezt ki kellett volna vlasztani. Ez ellentmondst jelent.
Kvetkezik, hogy F egy fa, s mivel F pontjai kztt a G minden pontja szerepel (Indokoljuk
meg!), ezrt F a G egy fesztfja.
Most igazoljuk, hogy F egy minimlis kltsg fesztfa. Legyenek F lei a
1
, a
2
, ..., a
n1
a
kivlaszts sorrendjben, azaz c(a
1
) c(a
2
) ... c(a
n1
). Legyen ugyanakkor T egy tetszleges,
F-tl klnbz fesztfa, amely leinek halmaza E(T) =t
1
, t
2
, ..., t
n1
.
Legyen a
i
(i 1) az els olyan l az algoritmus sorn, amely nem le T-nek (biztosan van ilyen
l, mert T ,=F), teht a
1
, a
2
, ..., a
i1
E(T), de a
i
/ E(T). Itt i =1 is lehet, ebben az esetben mr
a
1
/ E(T).
Az a
i
lt a T fhoz hozzvve egy C krt kapunk (lsd I.9.3.3 Ttel). Ez a C kr nem lehet
teljes egszben F-ben, mert F egy fa, ezrt C-nek ltezik legalbb egy olyan le e le, amely nem
le F-nek: e / E(F), s le T-nek: e E(T). Legyen T
1
az a grf, amelyet gy kapunk T-bl,
hogy T leihez hozzvesszk a
i
-t s elhagyjuk e-t, azaz E(T
1
) =E(T)a
i
e. Akkor T
1
is egy
fesztfa (Igazoljuk ezt!).
Megmutatjuk, hogy itt c(a
i
) c(e). Tegyk fel, hogy ez nem gy van, azaz c(e) < c(a
i
). Ha
itt i = 1, akkor c(e) < c(a
1
) ellentmond az algoritmus els lpsnek. Ha i = 2, akkor c(e) < c(a
2
)
ismt ellentmonds, mert e az a
1
-gyel nem alkothat krt, ezrt a msodik lpsben az e lt kellett
volna vlasztani. Legyen i 3 s c(e) <c(a
i
). Az algoritmus az i-edik lpsben nem az olcsbb e
lt vlasztotta, hanem a drgbb a
i
lt, aminek csak az a magyarzata lehet, hogy az e l krt
alkot az F fa mr kivlasztott a
1
, a
2
, ..., a
i1
leivel. De ezek az lek mind lei a T fnak, ezrt itt
is ellentmondsra jutunk. Kvetkezik, hogy valban c(a
i
) c(e). Innen pedig azt kapjuk, hogy T
1
kltsge nem tbb, mint T kltsge: c(T
1
) c(T).
Cserljk ki a T ft a T
1
fra. A T
1
fesztfnak tbb kzs le van F-fel, mint T-nek, mert
T
1
-et gy kaptuk, hogy T egy nem F-beli lt (az e lt) egy F-beli lre (az a
i
lre) cserltk.
Ha T
1
klnbzik F-tl, akkor ugyanezt az eljrst ismtelve olyan T
2
, T
3
,... fesztfkat kapunk,
amelyeknek egyre tbb kzs lk van F-fel s ... c(T
3
) c(T
2
) c(T
1
) c(T).
Vges sok lps (csere) utn olyan T
k
ft kapunk, amelyre T
k
=F s kvetkezik, hogy F kltsge
nem tbb, mint T kltsge: c(F) = c(T
k
) c(T).
Megjegyzsek. 1. A fenti eljrst moh algoritmusnak is nevezzk, mert az minden lpsben a
mg nem vizsglt lek kzl a legolcsbbat vlasztja. Csak akkor nem hasznlja a legolcsbbat,
ha az mr nem megengedett. Ez a loza sok esetben, gy a fentiekben is (de nem mindig!)
eredmnyre vezet.
2. Ha az eredeti G grf lei mind klnbz kltsgek, azaz c(e
1
) <c(e
2
) <... <c(e
m
), akkor
a fentiek szerint kvetkezik, hogy a minimlis kltsg fesztfa egyrtelmen meghatrozott.
3. Az R. C. Prim ltal adott algoritmus a kvetkez: Legyen G egy sszefgg grf. Tekintsk
ennek egy a
1
tetszleges pontjt. Az a
1
-bl kiindul lek kzl vlasszunk egy minimlis kltsgt,
gy egy a
1
, a
2
pontokbl ll ft kapunk. Vlasszunk egy olyan minimlis kltsg lt, amelynek
egyik vgpontja a
1
vagy a
2
, a msik vgpontja pedig egy ezektl klnbz a
3
pont, gy egy
hrompont ft kapunk. Ezt ismtelve, ha mr van egy a
1
, a
2
, ..., a
k
pontokbl ll T fa, akkor
vlasszunk egy olyan minimlis kltsg lt, amelynek egyik vgpontja T-beli, a msik vgpontja
T-n kvli, s ezt az lt vegyk hozz T-hez. Az eljrst addig folytatjuk, mg egy fesztft nem
kapunk.
I.9.5. MULTIGRFOK, GRFOK BEJRSA 77
Igazolhat, hogy az gy kapott fesztfa minimlis kltsg.
4. Egy ms algoritmus: Legyen G egy sszefgg grf. Rendezzk az leket a kltsgek cskken
sorrendjben: E(G) =e
1
, e
2
, ..., e
k
, ahol c(e
1
) c(e
2
) ... c(e
k
).
Legyen B =E(G), B lesz a kivlasztott lek halmaza.
Minden i =1,2, ..., k esetn az e
i
lt vagy trljk a B halmazbl, vagy sem a kvetkezkppen:
- Ha a B lhalmaz tartalmaz olyan krt, amelynek egyik le e
i
, akkor az e
i
lt trljk:
B :=Be
i
.
- Ha a B lhalmaz nem tartalmaz olyan krt, amelynek egyik le e
i
, akkor B nem vltozik.
Igazolhat, hogy ennek az algoritmusnak az outputja egy minimlis kltsg fesztfa.
I.9.5. Multigrfok, grfok bejrsa
Ebben a szakaszban olyan (irnyts nlkli) grfokat is vizsglunk, amelyeknek tbbszrs leik
s hurokleik is lehetnek. Ezeket multigrfoknak is nevezzk. Dencijuk a kvetkez:
I.9.5.1. Denci. Legyenek V s E tetszleges nemres halmazok, : E x, x, y : x, y
V, x ,= y pedig egy tetszleges fggvny. A G = (V, E, ) rendszert (irnyts nlkli) grfnak,
vagy multigrfnak nevezzk, ahol V elemei a grf pontjai, E elemei a grf lei. Ha egy e E lre
(e) =x, y, ahol x,=y, akkor x s y az e l vgpontjai. Ha (e) =x, akkor az e l hurokl.
I.9.5.2. Feladat. Legyen pldul V =1,2,3,4,5, E=e
1
, e
2
, e
3
, e
4
, e
5
, e
6
, e
7
, e
8
s (e
1
) =1,2,
(e
2
) =1,3, (e
3
) =1,3, (e
4
) =3, (e
5
) =4,5, (e
6
) =4,5, (e
7
) =4, (e
8
) =3,5.
Rajzoljuk meg ezt a grfot.
A sta, t s kr fogalmt hasonlkppen deniljuk, mint az egyszer grfok esetn.
I.9.5.3. Denci. Legyen G egy (multi)grf. Egy olyan stt, amely G minden lt tartalmazza
(pontosan egyszer) Euler-stnak vagy Euler-vonalnak nevezzk. Ez zrt Euler-sta, ha vgpontjai
azonosak, ellenkez esetben nyitott Euler-sta.
A G grf Hamilton-kre egy olyan kr, amely G minden pontjn thalad.
A knigsbergi-hidak problmja gy is megfogalmazhat: Van-e a ref8abra brn lev grfban
zrt Euler-sta?
I.9.5.4. Ttel. (Euler) Legyen G egy sszefgg grf.
i) Ha G-ben kettnl tbb pratlan fok pont van, akkor G-ben nincs Euler-sta.
ii) Ha G-ben pontosan kett pratlan fok pont van, akkor G-ben ltezik Euler-sta. Minden
Euler-sta nyitott s vgpontjai a pratlan fok pontok.
iii) Ha G-ben nincs pratlan fok pont, akkor G-ben ltezik Euler-sta. Ekkor minden Euler-sta
zrt.
Bizonyts. iii) Tegyk fel, hogy G-ben minden pont foka pros. Algoritmust adunk a grf bejrsra,
amely egy zrt Euler-sthoz vezet.
Induljunk el egy tetszleges v
1
pontbl egy tetszleges len s haladjunk ameddig csak lehet
mg be nem jrt leken. Szmozzuk meg a bejrs sorrendjben a bejrt leket. A feltel miatt
csak v
1
-ben akadhatunk el. Ha nem jrtuk be az sszes lt, akkor van olyan be nem jrt l, amely
egy mr rintett v
2
pontra illeszkedik. Induljunk el v
2
-bl be nem jrt leken haladva. Csak v
2
-ben
akadhatunk el. Szmozzuk jra a bejrt leket gy, hogy megtartjuk az elz szmozst, amg v
2
-
be nem jutunk, majd a msodszor bejrt leken folytatjuk a szmozst a v
2
-ben val elakadsig,
ezutn pedig az elszr bejrt leken folytatjuk a szmozst v
2
-tl kezdden. Ezt az eljrst
ismtelve vgl egy zrt Euler-stt kapunk.
78 I.9. FEJEZET. GRFELMLETI FOGALMAK
I.9.5.5. Feladat. Klnbz grfokra vizsgljuk meg az elz Ttel alkalmazhatsgt. Alkalmazzuk
az elbbi algoritmust.
I.9.5.6. Feladat. Nem szerepel az elbbi Ttelben az az eset, amikor a G grfnak pontosan egy
pratlan fok pontja van. Vajon mirt ?
Ezt a Ttelt gyakran gy adjuk meg:
I.9.5.7. Kvetkezmny. Egy sszefgg grfban akkor s csak akkor van zrt Euler-sta, ha a
grf minden pontjnak a foka pros.
A Hamilton-krk problmja hasonlt az Euler-stkra. Krds: Hogyan dnthet el, hogy
egy grfnak van-e Hamilton-kre?
Erre vonatkozan nem ismernk szksges s elgsges felttelt. Egy elgsges felttelt ad a
kvetkez
I.9.5.8. Ttel. (Dirac Gbor) Ha egy n pont (n3) egyszer grfban minden pont foka n/2,
akkor a grfnak van Hamilton-kre.
Bizonyts. Tegyk fel, hogy egy n pont G egyszer grfban minden pont foka n/2 s G-
ben nincs Hamilton-kr. Adjunk hozz a grfhoz jabb leket egszen addig, amg tovbbra is
fennll az, hogy a kapott grfokban nincs Hamilton-kr. Az sszes lehetsges lt biztosan nem
tartalmazhatja egy gy kapott grf, mert az a teljes grfot jelenten, amelyben van Hamilton-kr,
ehhez elg sorravenni a pontokat (tetszleges sorrendben).
Biztosan eljutunk teht egy olyan G

grfhoz, amelyben mg nincs Hamilton-kr, de brmely


jabb l hozzvtele esetn mr van Hamilton-kr. Egy ilyen G

grfot nevezznk teltett grfnak,


vizsgljuk ezt a tovbbiakban. jabb lek hozzvtelvel a pontok foka nem cskken, gy a G

grfra is igaz, hogy minden pont foka n/2.


Ha xy nem le G

-nak, akkor gy xy hozzvtele esetn Hamilton-krt kapunk, jellje ezt


[x =v
1
, v
2
, ..., v
n1
, v
n
= y].
Ksztsnk rajzot! Ha G

-ban x szomszdos v
j
-vel (azaz G

-ban xv
j
egy l), akkor y nem
szomszdos v
j1
-gyel, mert ellenkez esetben v
j1
y egy l s [x=v
1
, v
2
, ..., v
j1
, y=v
n
, v
n1
, v
n2
, ..., v
j
]
Hamilton-kr G

-ban, ami ellentmonds.


Tegyk fel, hogy x foka d(x) =k s x szomszdos a v
j
1
, v
j
2
, ..., v
j
k
pontokkal. Kvetkezik, hogy
y foka: d(y) n1k, mert y nem szomszdos a v
j
1
1
, v
j
2
1
, ..., v
j
k
1
pontokkal. Innen d(x) +
+d(y) n1.
Msrszt, a felttelbl d(x) n/2, d(y) n/2, ahonnan d(x)+d(y) n, ami ellentmonds.
Megjegyzs. Ebbl kvetkezik, hogy ha egy trsasgban mindenki ismeri legalbb a trsasg
tagjainak a felt, akkor a trsasg tagjai leltethetk egy kr alak asztal kr gy, hogy mindenkinek
ismerse legyen mind a kt szomszdja.
Msodik rsz
Feladatgyjtemny
79
II.1. fejezet
Permutcik, varicik, kombincik
II.1.1. Kidolgozott pldk
1.1. Anna, Bea, Csilla s Dra egytt mennek moziba. Hnyflekppen helyezkedhetnek el ngy
egyms mellett lv szken? rjuk le a lehetsges eseteket!
Megolds:
A B C D B A C D C A B D D A B C
A B D C B A D C C A D B D A C B
A C B D B C A D C B A D D B A C
A C D B B C D A C B D A D B C A
A D B C B D A C C D A B D C A B
A D C B B D C A C D B A D C B A
Ezzel az sszes lehetsges sorrendet ellltottuk, ami a konstrukcibl is nyilvnval, de
igazolhatjuk azzal is, ha beltjuk, hogy sszesen 24 elrendezs van.
4 elem ismtlsnlkli permutciirl van van sz. Ezek szma P
4
= 4! = 24.
Szoks a permutcikat fa-grfon is brzolni :
81
82 II.1. FEJEZET. PERMUTCIK, VARICIK, KOMBINCIK
A B-vel, C-vel, illetve D-vel, kezdd permutcik hasonl rszgrfokba rendezhetk.
1.2. Ksztsnk az 1; 2; 5; 6; 9 szmjegyekbl tjegy szmokat gy, hogy minden szmjegyet
egyszer hasznlhatunk.
a) Hny tjegy szm kpezhet gy?
b) Hny 15-tel kezdd szm kpezhet gy?
c) Hny olyan szm kpezhet gy, melyben az 1-es s az 5-s szmjegy egyms mellett van,
mghozz ilyen sorrendben?
d) Hny pros (tjegy) szm kpezhet a fenti mdon?
e) Hny pratlan (tjegy) szm kpezhet a fenti mdon?
f) Hny nggyel oszthat szm kpezhet a fenti mdon?
g) Hny hrommal oszthat szm kpezhet a fenti mdon?
Megolds:
a) A felsorolt szmjegyek minden sorbarendezse egy-egy tjegy szmot ad. Ezek szma
P
5
= 5!.
b) 15-tel kezdd szmokat gy tudonk kpezni, ha az els kt jegyet xljuk, majd a
maradk hrom helyre a megmarad hrom szmjegyet elhelyezzk. Ez P
3
= 3!-flekppen
tehet meg.
c) A 15 prt egy szmjegynek tekintjk. (Mintha szmkrtykat ksztettnk volna az
bra szerint.)
II.1.1. KIDOLGOZOTT PLDK 83
Az gy kapot 4 krtyt P
4
= 4!-flekppen rendezhetjk sorba.
d) Ha pros szmot szeretnnk kpezni, akkor az utols jegy pros kell, hogy legyen. gy az
utols helyre ktfle szmjegy rhat (2-es vagy 6-os), a tbbi helyre a maradk 4 jegyet
P
4
=4!-flekppen rakhatjuk be. A szorzsi szablyt felhasznlva az sszesen 24! klnbz
tjegy pros szm kpezhet a megadott jegyekbl.
e) 1. Megolds
sszes tjegy szm: 5!
a prosak: 2 4!
pratlanok: 5! 2 4!
2. Megolds
Ha pratlan szmot szeretnnk kszteni, akkor az utols helyre 3 szmjegy kzl vlaszthatunk
(1; 5; 9). A maradk 4 helyirtkre 4 szmjegynk maradt, ezeket P
4
= 4!-flekppen
rendezhetjk el. A szorzsi szably alapjn sszesen 3 4! pratlan szm kpezhet.
f) Pontosan akkor kapunk 4-gyel oszthat szmot, ha az utols kt jegybl ksztett szm
oszthat 4-gyel. A megadott jegyekbl a kvetkez 6 szmpr alkalmas az utols kt
helyirtk kitltsre:
12 52 92
16 56 96
g) Egy szm akkor s csak akkor oszthat 3-mal, ha a szmjegyeinek sszege oszthat 3-mal.
A permutciknt kapott tjegy szmok mindegyiknl a jegyek sszege:
1+2+5+6+9 = 23,
azaz nincs olyan a megadott jegyekbl kszthet tjegy szm, amely oszthat lenne
3-mal.
1.3. Hny hatjegy ttel oszthat szm kpezhet a 0; 1; 2; 3; 4; 5 szmjegyekbl, ha minden
szmjegy csak egyszer fordulhat el?
1. Megolds:
Akkor beszlnk valdi hatjegy szmrl, ha az nem kezddik 0-val. Ha 5-tel oszthat szmot
szeretnnk ellltani a megadott szmjegyekbl, akkor a szmnak 5-re, vagy 0-ra kell vgzdnie.
Szmoljuk ez alapjn kln az eseteket:
Ha 0-ra vgzdik, akkor a maradk 5 helyre a fennmarad 5 szmjegyet P
5
= 5!-flekppen
rendezhetjk el. (Hiszen itt nem kell arra gyelnnk, hogy valdi hatjegy szmot ksztsnk.)
Ha a szm 5-re vgzdik, akkor vigyznunk kell arra, hogy a szm ne kezddjn 0-val. Ezt
gy tudjuk biztostani, ha az els helyre csak 4 szmjegy (1; 2; 3; 4) kzl vlasztunk. A
maradk 4 helyre a fennmarad 4 szmjegyet P
4
= 4!-flekppen rendezhetjk el. A szorzsi
szably alapjn teht sszesen 4 4! olyan valdi hatjegy szm van, amely 5-re vgzdik.
A kt diszjunkt esethez tartoz lehetsgeket az sszeadsi szably alapjn szmolhatjuk
ssze. Teht sszesen
5! +4 4! = 9 4!
5-tel oszthat valdi hatjegy szm kszthet a megadott felttelek mellett.
2. Megolds:
84 II.1. FEJEZET. PERMUTCIK, VARICIK, KOMBINCIK
1. lps: Szmoljuk ssze az sszes 5-tel oszthat szmot amit a fenti jegyek felhasznlsval
kszthetnk. (Termszetesen minden szmjegyet pontosan egyszer hasznlunk.) Ezek kztt
a szmok kztt teht lesznek nem-valdi hatjegy szmok.
Az utols helyirtkre ekkor kt szmjegy kzl vlaszthatunk (0; 5). A tbbi helyre a fenn-
marad 5 szmjegyet P
5
= 5!-flekppen rhatjuk be. gy sszesen 2 5! klnbz szmot
ksztettnk. Ne felejtsk, hogy ezek kztt mg vannak olyanok, amelyek nem tesznek eleget
a feladat feltteleinek.
2. lps: Szmoljuk ssze a rossz eseteket. Ezek olyan szmok, amelyek 0-val kezddnek s
5-re vgzdnek. A kzps 4 helyre a maradk 4 szmjegyet P
4
= 4!-fle sorrendben rhatjuk
be.
3. lps: A feltteleknek megfelel ellltsok szmt az 1. lpsben szmolt esetek s a 2.
lpsben ellltott esetek klnbsgeknt kapjuk:
2 5! 4! = 10 4! 4! = 9 4!
1.4. Az 1; 1; 1; 2; 2; 3; 3 szmjegyekbl
a) hny htjegy szmot lehet kszteni ?
b) hny 13-mal kezdd szm kpezhet?
Megolds:
a) Klnbztessk meg egymstl mondjuk sznezssel az azonos szmjegyeket!
Ekkor 7 klnbz elem sszes sorbarendezseit kell vizsglnunk, amely P
7
=7!-fle sorbarendezst
jelent. Ha most jra eltekintnk a sznezstl, akkor azt vehetjk szre, hogy vannak
olyan szmok, amelyeket tbbszr szmoltunk. Tekintsk pldul azokat a szmokat,
amelyekben a 2-esek s a hrmasok helye a sznezsekre is tekintettel megegyezik.
Ezekbl pontosan annyit szmoltunk, ahnyflekppen a 3 klnbz szn egyes a fennmarad
hrom helyre elhelyezhet. (3!) Ezek a felrsok valjban egyetlen, a feltteleknek megfelel
htjegy szmhoz tartoznak, gy minden olyan szmot, amely csak az egyesek sznezsben
trnek el 3!-szor szmoltunk. gy is meg lehet fogalmazni a fentieket, hogy ha az egyesek
sznezstl eltekintnk, de a 2-esektl s a 3-asoktl nem, akkor
7!
3!
klnbz htjegy
szmot kpezhetnk. A gondolatmenetet megismtelve tekintsnk el a 2-esek majd a 3-
asok sznezstl. gy sszesen
P
3,2,2
7
=
7!
3! 2! 2!
klnbz szm kpezhet a feladat eredeti feltteleinek megfelelen.
b) Elszr biztostsuk azt, hogy a szm 13-mal kezddjn! Ezt gy tehetjk meg, ha az
els helyre az egyik egyest, a msodik helyre pedig az egyik hrmast rgztjk. Ezutn a
feladat csupn annyi, hogy meghatrozzuk, hogy a fennmarad t helyre hnyflekppen
rendezhetjk el a maradk szmjegyeinket, azaz kt 1-est, kt 2-est s egy 3-ast. Ez az
elz feladat gondolatmenett kvetve
P
2,2,1
5
=
5!
2! 2!
klnbz elrendezst jelent.
II.1.1. KIDOLGOZOTT PLDK 85
1.5. Egy 88-as sakktbla bal fels mezjbl indulva hny klnbz ton juthatunk el a jobb
als sarokba, ha minden lpsben egy mezt jobbra, vagy egy mezt lefel lpnk.
1. Megolds:
Amg a bal fels sarokbl eljutunk a jobb als sarokba, addig 7 alkalommal kell jobbra
lpnnk egy mezt s ht alkalommal lefel egyet. A feladatnak megfelel utak egyrtelmen
jellemezhetk azzal, hogy hnyadik lpsben lpnk lefel illetve mikor jobbra. Jelljk a
jobbra-lpseket jellel s a lefele lpseket jellel. Ezzel a feladatnak megfelel utakhoz
bijektv mdon hozzrendelhetnk egy-egy 7 -bl s 7 -bl ll jelsorozatot. Ezek szma
pedig
P
7,7
14
=
14!
7! 7!
2. Megolds:
A problmt tekinthetjk ismtls nlkli kombincinak is. A bal fels sarokbl indulva 14
lpsen keresztl juthatunk a jobb als sarokba. Az elbb trgyaltak alapjn a 14 lps kztt
pontosan ht jobbra s pontosan ht lefele tpus lps szerepel. Egy tvonalat egyrtelmen
meghatrozhatunk, ha kijelljk azon ht lps sorszmt, melyek sorn jobbra mozdultunk
el. (Ez nyilvnvalan meghatrozza azon lpsek sorszmt is, amelyekben lefele lptnk.)
gy a lpsek sorszmai kzl (1-14-ig) vlasztunk a kvetkez felttelek mellett:
14, pronknt klnbz elem kzl vlasztunk,
7 elemet vlasztunk,
visszatevs nlkl vlasztunk,
a kivlaszts sorrendjre nem vagyunk tekintettel.
Azaz valban ismtls nlkli kombincirl van sz, gy a klnbz kivlasztsok (s gy a
klnbz tvonalak) szma:
C
7
14
=
_
14
7
_
=
14!
7! 7!
.
1.6. Szmozzunk meg 6 piros s 5 kk korongot!
a) Hnyflekppen rakhatjuk sorba a 11 korongot gy, hogy azonos sznek ne kerljenek
egyms mell?
b) Hnyflekppen tehetjk ezt meg akkor, ha a korongok szmozstl eltekintnk?
Megolds:
a) A feladatnak megfelel sorbarendezsek mindegyike piros koronggal kezddik. Az elrendezsekben
felvltva kvetik egymst a piros s kk korongok s gy termszetesen az utols korong is
piros.
86 II.1. FEJEZET. PERMUTCIK, VARICIK, KOMBINCIK
Szmozott korongok esetn ms s ms elrendezsnek szmt, ha a fenti sorban az azonos
szn korongokat ms s ms sorrendben rjuk. A piros korongokat egyms kztt cserlgetve
6! klnbz sorrendben rhatjuk, mg a kk korongok felcserlseivel 5! klnbz elrendezst
hozhatunk ltre. Ezeket a vltoztatsokat egymstl fggetlenl megtehetjk, gy a szorzsi
szably alapjn
6! 5!
elrendezs felel meg a feladatnak.
b) Amennyiben a korongok szmozstl eltekintnk, akkor egyetlen elrendezs a fenti
brn lthat felel meg a feltteleknek.
1.7. 6 hzaspr (6 fr s 6 n) rkezik egy trsasgba. Hnyflekppen lhetnek le
a) egy 12 szemlyes hossz padra?
b) egy 12 szemlyes hossz padra, ha azonos nemek nem lhetnek egyms mell?
c) egy 12 szemlyes hossz padra, ha A r a felesge mell szeretne lni ?
d) egy 12 szemlyes hossz padra, ha mindenki a prja mell szeretne lni ?
e) egy 12 szemlyes hossz padra, ha csak az szmt frrl vagy nrl van-e sz? (Hnyflekppen
rakhat sorba 6 fehr s 6 fekete goly?)
f) egy 12 szemlyes hossz padra, ha azonos nemek nem lhetnek egyms mell s csak az
szmt frrl vagy nrl van-e sz?
g) egy kerek asztal krl, ahol 12 egyforma szk van?
h) egy kerek asztal krl, ahol 12 egyforma szk van, ha azonos nemek nem lhetnek egyms
mell?
i) egy kerek asztal krl, ahol 12 egyforma szk van, ha A r a felesge mell szeretne lni ?
j) egy kerek asztal krl, ahol 12 egyforma szk van, ha mindenki a prja mell szeretne
lni ?
k) egy kerek asztal krl, ahol 12 egyforma szk van, ha azonos nemek nem lhetnek egyms
mell s csak az szmt frrl vagy nrl van-e sz?
A kerekasztal krli ltetseket akkor tekintjk klnbznek, ha van olyan szemly akinek
a kt ltets sorn megvltozik a bal vagy a jobb oldal szomszdja. Rviden azt mondhatjuk,
hogy kt ltets ekvivalens, ha ltezik olyan forgats, amely az egyiket a msikba viszi.
Szoks nha gy rtelmezni a problmt, hogy akkor is ekvivalens a kt ltets, ha tengelyes
tkrzssel vihetk egymsba. (A feladat sorn nem erre gondoltunk, de akinek van kedve
utnagondolhat, mennyiben vltozik a problma ilyen krlmnyek kztt.)
Megolds:
a) A krds gyakorlatilag az, hnyflekppen lehet sorbarendezni 12 klnbz elemet. Ez
pedig a korbbiakhoz hasonlan P
12
= 12!-flekppen tehet meg.
b) A problma a 6a feladathoz hasonlan trgyalhat. Lnyeges klnbsg azonban, hogy
itt ugyanannyi fr s n tallhat a csoportban. Az elz feladatban pratlan sok elemet
kellett gy sorbarendezni, hogy az azonos tpsak ne kerljenek egymsmell. Ott szksgszeren
II.1.1. KIDOLGOZOTT PLDK 87
addott, hogy a sort abba a tipusba tartoz elemmel kell kezdennk, amely tipusbl
eredetileg tbb elem volt. Itt a kt osztlyba (frak illetve nk) ugyanannyi elem van,
gy ha mondjuk nvel kezdjk az letetst, akkor a pad msik szlre biztosan fr kerl.
Ez azt is jelenti, hogy az ltetseket kt csoportba oszthatjuk, aszerint, hogy nvel, vagy
frvel kezddnek.
Ha az els ember nemt rgztettk (legyen mondjuk fr), akkor, a 6a feladathoz hasonlan
a sajt osztlyon belli cserk
6! 6!
klnbz ltetst generlnak. Ne felejtsk el azonban, hogy ugyanennyi ltetst tudunk
ellltani akkor is, ha a sort nvel kezdjk. gy az sszes ltetsek szma:
2 6! 6!
c) Jelljk A r felesgt a-val. Azt, hogy k egymsmell kerljenek a legknnyebben gy
biztosthatjuk, ha ket egyetlen elemnek tekintjk. gy 11 elem sszes elrendezseit kell
meghatroznunk. Ez P
11
=11! klnbz ltetst jelent. Minden egyes esethez, amelyet az
elbb sszeszmoltunk kt valdi ltets tartozik aszerint, hogy A r felesge a frje jobb,
vagy baloldaln l-e. (Aa vagy aA eset.) gy az sszes lehetsgek szma:
2 11!
d) Az elz gondolatmenethez hasonlan 6 darab dupla elemet ksztnk (Aa, Bb,
Cc,. . . ) Ekkor 6 elem ismtlsnlkli permutciival talljuk szemben magunkat. Ezekbl
6! van. De most is gyelmen kvl hagytuk azt, hogy az egyes prokon bell milyen
sorrendben lnek a tagok. Ez minden pr esetn egy 2-es faktort jelent. gy az sszes
lehetsgek szma, a szorzsi szably alapjn
2
6
6!
e) Induljunk ki az sszes lehetsges ltetsekbl. Ezek szmt a feladat a) rszben mr
meghatroztuk (12!). Ha most csak a rsztvevk nem az rdekes, az identitsuktl eltekintnk,
akkor az eredeti sszeszmlls sorn klnbz eseteknek szmolt ltetsek valjban
megklnbztethetetlenek. Knnyen lthat, hogy itt csak egy esetnek szmt minden
olyan a) feladatbeli ltets, amelyek azonos nemek cserjvel egymsba vihet. Az ilyen
sszetartoz elrendezsek szma 6!6!, azaz az itt klnbznek tekintett ltetsek szma:
12!
6! 6!
f) A feladat a 6b feladthoz hasonlan oldhat meg. Itt is gyelnnk kell arra, hogy a pros
elemszm miatt az ltets kezddhet akr nvel, akr frvel. gy kt klnbz ltets
felel meg a feltteleknek.
g) A problmnak kt megkzeltst szoks alkalmazni.
1. mo
Szmozzuk meg a szkeket, ezzel megklnbztetjk egymstl a helyeket, gy 12 elem
sorbarendezseit kell vizsglnunk. Ezutn vegyk gyelembe a forgsi szimmetrit. Ha a
helyeket jra egyformnak tekintjk akkor azok az esetek, amelyek valamely 2 szgnl
kisebb forgatssal, (az irny a feladat szempontjbl rdektelen) egymsba vihetk nem
88 II.1. FEJEZET. PERMUTCIK, VARICIK, KOMBINCIK
jelentenek klnbz ltetst. Ezltal osztlyozhatjuk az ltetseket. Kerljenek egy osz-
tlyba azok az esetek, amelyek a fenti felttelnek megfelel forgatssal szrmaztathatk
egymsbl. Knnyen lthat, hogy minden osztlyba pontosan 12 ltets tartozik. gy az
osztlyok szma, ami megegyezik a kerekasztal krli klnbz ltetsek szmval :
12!
12
= 11!
2. mo
Az els embert ltessk le az asztalhoz (A urat). A forgsi szimmetria miatt gyakorlatilag
mindegy, hogy melyik szkre l. Ezutn az asztal minden szke klnbznek tekinthet,
hiszen ms s ms poziciban vannak A rhoz kpest. gy 11 klnbz helyre kell 11
klnbz elemet elhelyeznnk. Ez pedig az ismert formula alapjn 11!-flekppen tehet
meg.
h) ltessk le most is az els szemlyt, A urat az asztalhoz. Az elzekhez hasonlan most 11
klnbz helynk van, melyre 6 nt s 5 frt kell leltetnnk gy, hogy azonos nemek
ne kerljenek egymsmell (azaz felvltva). Ez nyilvnvalan csak gy tehet meg, ha a
sort hlggyel kezdjk s fejezzk is be. A feladat innentl teljesen azonos a 6a feladattal.
(A fraknak felelnek meg a kk korongok, a nknek a pirosak.) gy az sszes lehetsges
ltetsek szma
5! 6!
i) 1. mo
Kssk ssze A urat a felesgvel, azaz ksztsnk egy specilis Aa elemet. Majd az
gy keletkez 11 elemnket (10 hagyomnyos elem s az A csaldot jelkpez dupla elem)
rendezzk el egy 11-szkes kerekasztal krl. Ez az elbbiek alapjn 10!-flekppen tehet
meg. Ne felejtsk el azonban most sem, hogy a dupla elemnkn bell 2-fle lsi sorrend
lehetsges. gy az sszes elrendezsek szma:
2 10!
2. mo
Elszr ltessk A urat az asztalhoz, mivel a helyek egyenrtkek, gyakorlatilag mindegy
melyik szkre l. Ezzel azonban a maradk 11 hely mindegyike klnbznek tekintehet
A r szkhez viszonytott helyzettl fggen.
A r felesge (a) a frjvel szomszdos kt szk brmelyikre lhet. (2 lehetsg)
A maradk 10 szkre a tbbiek 10!-flekppen lhetnek le. (a mindkt pozcijhoz ilymdon
a tbbieknek 10! lehetsges elhelyezkedse tartozik.)
Az sszes ltetsek szma teht 2 10!
j) Most 6 dupla elemet kell ksztennk. Ezek a kerekasztal krl 5!-flekppen helyezkedhetnek
el. Minden pron bell ktflekppen lhetnek a tagok. gy a szorzsi szably alapjn az
sszes elrendezsek szma:
2
6
5!
k) A h) feladat megoldsa sorn mr vgiggondoltuk ezt a problmt s arrajutottunk, hogy
egyetlen elrendezs felel meg a feltteleknek.
1.8. Adott n pronknt klnbz elem. Kt egymstl s az eddigiektl klnbz elemet hozzjuk
vve a permutcik szma 156-szorosra n. Mekkora n rtke?
II.1.1. KIDOLGOZOTT PLDK 89
Megolds:
A feladatban megfogalmazott llts az albbi alakban rhat:
156 P
n
= P
n+2
156 n! = (n+2)! = (n+2) (n+1) n!
156 = (n+2) (n+1)
0 = n
2
+3n154 = (n+14) (n11)
Mivel az n =14 megolds rtelmetlen, ezrt az egyetlen megolds n = 11.
1.9. Hnyflekppen oszthatunk szt egy pakli magyar krtyt (32 lap) 4 jtkosnak gy, hogy
mindegyikk 8-8 lapot kapjon?
1. Megolds:
Rakjuk ki a krtykat egyms mell s rjuk al, hogy melyik jtkosnak adtuk (1; 2; 3; 4).
gy minden, a feladat feltteleinek megfelel kiosztshoz egy olyan 32 hosszsg szmsorozat
rendelhet, amely pontosan 8 1-est, 8 2-est, 8 3-mast s 8 4-est tartalmaz. Teht pontosan
annyi kioszts ltezik, amennyi ismtlses permutcija van a jellemzett szm-harminckettesnek:
P
8,8,8,8
32
=
32!
8! 8! 8! 8!
A feladatra mg visszatrnk.
1.10. Az szs dntbe 8-an jutottak be. Hnyfle lehet a dobogs helyek elosztsa, ha felttelezzk,
hogy nem volt holtverseny?
Megolds:
Jelljk a 8 dntbe jutott versenyzt az ABC betivel : A; B; C; D; E; F; G; H. Ekkor
az els helyezett 8 szemly kzl kerlhet ki, a msodik helyezettre mr csak 7 versenyz
kzl vlaszthatunk (aki els lett itt mr nem jn szmtsba), a harmadik helyre mr csak
6 lehetsg van, hiszen az els kt helyezett itt mr nem futhat be.
gy V
3
8
= 8 7 6 fle sorrend lehet a dobogn.
Termszetesen most is brzolhatnnk a lehetsgeket fa-grfon.
1.11. Az 1; 2; 3; 4; 5; 6 szmjegyekbl, hny klnbz
a) hromjegy szm kpezhet,
b) hromjegy pros szm kpezhet,
c) szm kpezhet,
ha minden szmjegy legfeljebb egyszer hasznlhat fel ?
Megolds:
90 II.1. FEJEZET. PERMUTCIK, VARICIK, KOMBINCIK
a) Az egyesek helyre 6 szmjegy kzl vlaszthatunk. A tzeseket mr csak 5 kzl, mg a
szzasokhoz mr csak 4 jegy marad. gy a szorzsi szably alapjn
V
3
6
= 6 5 4
hromjegy szm kszthet.
b) Ha pros szmokat szeretnnk generlni, akkor az utols jegyet a hrom pros szmjegy
kzl (2; 4; 6) kell vlasztani. A msik kt helyirtket az a) feladathoz hasonlan vlaszthatjuk.
gy az sszes elllthat pros szm:
3 5 4.
c) A problma legegyszerbben diszjunkt esetekre val sztvlasztssal trgyalhat. Azaz
sszeszmoljuk, hogy hny pontosan egyjegy, pontosan ktjegy s gy tovbb, vgl
hny pontosan hatjegy szm kpezhet a megadott jegyekbl.
egyjegy szmok: 6 db
ktjegy szmok: Az elz feladatokhoz hasonlan: 6 5 db
hromjegy szmok: 6 5 4 db
ngyjegy szmok: 6 5 4 3 db
tjegy szmok: 6 5 4 3 2 db
hatjegy szmok: 6 5 4 3 2 1 db
Az sszeadsi szablyt alkalmazhatjuk, gy sszesen:
6+30+120+360+720+720 = 1956
szm kpezhet.
1.12. Az 0; 1; 2; 3; 4; 5 szmjegyekbl, hny klnbz
a) hromjegy szm kpezhet,
b) hromjegy pros szm kpezhet,
c) szm kpezhet,
ha minden szmjegy legfeljebb egyszer hasznlhat fel ?
Megolds:
A feladat az elz problmtl annyiban klnbzik, hogy a felhasznlhat szmjegyek kztt
most szerepel a 0 is. Ez a ltszlag apr eltrs a megolds menett komolyan befolysolja,
hiszen gyelnnk kell arra, hogy az ellltott szm nem kezddhet 0-val.
a) A permutciknl megismert ktfle megkzelts alkalmazhat itt is.
1. mo
A szzasok helyn 5 szmjegy kzl vlaszthatunk (mivel a 0-t nem rhatjuk ide). A tzesek
helyn ismt 5 lehetsgnk van, hiszen a szzasok kztt vlasztott jegyet nem rhatjuk,
de helyette hasznlhat a 0. Az utols helyirtkre a megmaradt 4 szmjegy kzl lehet
vlasztani. gy a szorzsi szably alapjn
5
2
4
II.1.1. KIDOLGOZOTT PLDK 91
valdi hromjegy szm llthat el
2. mo Vegyk az sszes harmadosztly ismtlsnlkli permutci szmt, majd vonjuk
ki azok szmt, amelyek 0-val kezddnek.
Az sszes elllthat legfeljebb hromjegy szm (bent vannak a 0-val kezddk is):
V
3
6
= 6 5 4
A 0-val kezdd hromjegy szmok gy llthatk el, hogy a szsok helyn rgztem a
0 jegyet, majd a fennmarad 5 szmjegybl ktjegy szmokat ksztek:
V
2
5
= 5 4
gy a valdi hromjegy szmok szma:
6 5 45 4 = 5 5 4.
b) Ha pros szmokat szeretnnk generlni, akkor az utols jegyet a hrom pros szmjegy
kzl (0; 2; 4) kell vlasztani. rdemes az eseteket klnvlasztani aszerint, hogy az utols
helyre 0-t rtunk-e. (Ha 0 ll az utols helyirtken, akkor nem kell azzal foglalkozni, hogy
valdi hromjegy szmot lltsunk el, hiszen ez automatikusan teljesl.)
0-ra vgzdik:
Az egyesek helyn a 0-t rgztjk a tzesek helyre ekkor 5, mg a szzasok helyre 4 jegy
kzl vlaszthatunk. gy 5 4 ilyen szm llthat el.
nem -ra vgzdik:
Az egyesek helyn a 2 jegy kzl vlaszthatunk (2; 4) a szzasok helyre 4 (nem vlaszthat
a mr elhasznlt jegy s a 0 sem), mg a tzeseket 4 jegy kzl lehet vlasztani. gy 2 4 4
ilyen szm llthat el.
Az sszeadsi szably alapjn:
5 4+2 4 4 = 4 13 = 52
a feltteleknek megfelel szm llthat el.
c) 1. mo
A problma az elz feladat c) rszhez hasonlan trgyalhat, csak itt gyelnnk kell
arra, hogy a szmok ne kezddjenek 0-val (kivve az egyjegy szmokat).
egyjegy szmok: 6 db
ktjegy szmok: Az elz feladatokhoz hasonlan: 5 5 db
hromjegy szmok: 5 5 4 db
ngyjegy szmok: 5 5 4 3 db
tjegy szmok: 5 5 4 3 2 db
hatjegy szmok: 5 5 4 3 2 1 db
Az sszeadsi szablyt alkalmazhatjuk, gy sszesen:
6+25+100+300+600+600 = 1631
szm kpezhet.
2. mo Az elz feladatban ellltott 1956 szm
1
6
-a kezddik 1-essel,
1
6
-a 2-essel, s gy
tovbb. Ha a 6-os jegyet 0-ra cserljk, ahogy az a feladat kirsban szerepel, akkor azt
92 II.1. FEJEZET. PERMUTCIK, VARICIK, KOMBINCIK
llthatjuk, hogy ppen az esetek
1
6
-a kezddik 0-val. Ezek a hibs szmok s a 0. gy a
feladat feltteleinek megfelelen elllthat szmok szma:
5
6
1956+1 = 1631.
1.13. Az 1; 2; 3; 4; 5; 6 szmjegyekbl, hny klnbz
a) hromjegy szm kpezhet,
b) hromjegy pros szm kpezhet,
c) legfeljebb hromjegy szm kpezhet,
ha a szmjegyek tbbszr is felhasznlhatk?
Megolds:
A feladat megoldsa a 11. feladathoz meglehetsen hasonl. Figyelnnk kell arra, hogy itt az
egyes szmjegyek tbbszr is felhasznlhatk. (Ha a klasszikus esetek kz szeretnnk sorolni
a problmt, itt ismtlses varicit kell alkalmazni.)
a) Az sszes helyirtkre a rendelkezsre ll 6 szmjegy brmelyike berhat. gy a szorzsi
szably alapjn
V
3
6
= 6
3
hromjegy szm kszthet.
b) Ha pros szmokat szeretnnk generlni, akkor az utols jegyet a hrom pros szmjegy
kzl (2; 4; 6) kell vlasztani. A msik kt helyirtket az a) feladathoz hasonlan vlaszthatjuk.
gy az sszes elllthat pros szm:
3 6
2
.
c) A problma legegyszerbben diszjunkt esetekre val sztvlasztssal trgyalhat. Azaz
sszeszmoljuk, hogy hny pontosan egyjegy, pontosan ktjegy s vgl hny pontosan
hatjegy szm kpezhet a megadott jegyekbl.
egyjegy szmok: 6 db
ktjegy szmok: Az elz feladatokhoz hasonlan: 6
2
db
hromjegy szmok: Az elz feladatokhoz hasonlan: 6
3
db
Az sszeadsi szablyt alkalmazhatjuk, gy sszesen:
6+6
2
+6
3
= 258
szm kpezhet.
Megjegyzs: ltalnosan, pontosan k-jegy szmot 6
k
darabot tudunk generlni. gy a
legfeljebb n-jegy szmok szma az sszeadsi szablyt alkalmazva:
n

k=1
6
k
=
6 (6
n
1)
5
.
1.14. Az 0; 1; 2; 3; 4; 5 szmjegyekbl, hny klnbz
a) hromjegy szm kpezhet,
b) hromjegy pros szm kpezhet,
II.1.1. KIDOLGOZOTT PLDK 93
c) legfeljebb hromjegy szm kpezhet,
ha a szmjegyek tbbszr is felhasznlhatk?
Megolds:
a) Az els helyre a 0 kivtelvel brmelyik szmjegy rhat, gy 5 lehetsg kzl vlaszthatunk.
A tbbi helyirtket a 6 szmjegy brmelyikvel kitlthetjk, gy az sszes lehetsgek
szma:
5 6
2
b) Az els helyre a 0 kivtelvel brmelyik szmjegy rhat (5 lehetsg), mg az utols
jegyet csak a pros szmjegyek kzl vlaszthatom (3 lehetsg). A tzesek helyre brmely
szmjegy berhat (6 lehetsg). gy, a szorzsi szably alapjn sszesen
5 3 6 = 90
szm llthat el.
c) 1. mo
A korbbi feladatokhoz hasonlan.
Pontosan egyjegy szm 6 darab llthat el.
Pontosan ktjegy szm 5 6 darab,
pontosan hromjegy szm 5 6
2
darab.
gy legfeljebb hromjegy szmot a fenti diszjunkt esetek egyestsvel kaphatunk. Az
sszeadsi szably szerint
6+30+180 = 216
klnbz szm llthat el.
2. mo
A valdi ktjegy szmok felfoghatk olyan hromjegy szmokknt, melyeknek az els
jegye 0. s hasonlan az egyjegy szmok felfoghatk olyan hromjegy szmokknt,
melyeknek az els kt helyirtken (szzasok, tzesek) 0 ll. gy az sszes legfeljebb hromjegy
szm gy generlhat, hogy nem zrjuk ki a 0-val kezdd szmokat. gy
V
3
6
= 6
3
.
klnbz szmot tudunk ellltani. Termszetesen szerepel a 0 is, amit most 000 alakban
ksztettnk el.
Megjegyzs: A fentiekkel teljesen analg problmra vezet, ha megprblunk vlaszolni arra
a krdsre, hogy egy adott szmrendszerben (ha kln nem hangslyozzuk, akkor a 10-es
szmrendszerben vizsgldunk) hny, bizonyos felttelnek megfelel k- jegy szm van.
1.15. Hnyflekppen olvashat ki az albbi brbl a MATBSC sz, ha a bal fels sarokban
tallhat M betbl indulunk s minden lpsben jobbra, vagy lefel lpnk egy mezt:
M A T B S C
A T B S C
T B S C
B S C
S C
C
Megolds:
94 II.1. FEJEZET. PERMUTCIK, VARICIK, KOMBINCIK
A sz kiolvasshoz 5 lps szksges. Minden lps sorn kt lehetsg kzl vlaszthatunk
(vagy jobbra lpnk, vagy lefel). Minden a feladat feltteleinek megfelel kiolvassi-thoz
bijektv mdon hozzrendelhetnk egy 5 karakterbl ll jelsorozatot, amely s elemekbl
pl fel. Ilyen jelsorozatot V
5
2
= 2
5
-flekppen tudunk kszteni.
1.16. Az origbl indulva hnyflekppen juthatunk el az e : y =x+5 egyenes egsz-koordintj
pontjaiba, ha az albbi szablyok alapjn haladunk:
i) Minden lps sorn pozcink x s y koordinti kzl pontosan az egyiket vltoztatjuk
meg, mghozz eggyel nveljk.
ii) Nem lpjk t az e egyenest (s a tengelyeket sem, de ez az els felttelbl nyilvnval).
Megolds:
A feladat az elz problma analogonja.
Minden helyes t pontosan 5 lpsbl ll. Minden lps sorn kt lehetsg kzl vlaszthatunk
(vagy jobbra lpnk, vagy felfel). Minden a feladat feltteleinek megfelel thoz bijektv
mdon hozzrendelhetnk egy 5 karakterbl ll jelsorozatot, amely s elemekbl pl
fel. Ilyen jelsorozatot V
5
2
= 2
5
-flekppen tudunk kszteni.
1.17. n elem 3-adosztly ismtlses s ismtlsnlkli variciinak arnya 9: 5. Mekkora n rtke?
Megolds:
II.1.1. KIDOLGOZOTT PLDK 95
rjuk fel a feladat feltteleit:
V
3
n
V
3
n
=
9
5
n
3
n (n1) (n2)
=
9
5
5n
3
= 9n (n1) (n2) n ,= 0
5n
2
= 9n
2
27n+18
0 = 4n
2
27n+18
0 = (4n3) (n6)
n
1
=
3
4
/ N rtelmetlen
n
2
= 6
1.18. Hny hagyomnyos TOT szelvnyt kell kitltennk, hogy biztosan legyen egy telitalla-
tunk?
Megolds:
A telitallatot gy tudjuk garantlni, ha az sszes lehetsges kitltst jtkba kldjk. gy a
feladat krdse tulajdonkppen az, hnyflekppen tlthet ki a hagyomnyos tot-szelvny.
Az egyszersg kedvrt tekintsk gy, hogy a tot-szelvnyen 14 mrkzs kimenetelt kell
eltallnunk, azaz 14 helyre kell a rendelkezsnkre ll hrom jel (1; 2; x) valamelyikt berni.
Ez pedig 3
14
-flekppen tehet meg. gy a biztos telitallathoz ennyi szelvny megvsrlsa
s kitlts szksges.
Megjegyzs:
Termszetesen a fenti mdon nem csak a telitallatot tudjuk biztostani. Emellett 28
szelvnyen lesz 13 tallatunk, 364-en 12 tallatunk s gy tovbb. A problma rszletes
meggondolsra a jv rn, a kombincik trgyalsa sorn visszatrnk.
Mg mieltt brki azt hinn, hogy a meggazdagods gyors receptjt kaptuk meg, gondoljunk
bele, hogy a szelvnyek rnak bezetsn tl ami nmagban elg lenne ahhoz, hogy
ne rje meg gy jtszani ki is kellene tltennk 3
14
darab szelvnyt. Ha egy szelvny
kitltsre 14 msodpercet szmolunk, akkor a kitltshez szksges id:
1116026,1 perc = 18600,435 ra = 775,018125 nap,
ami tbb mint 2 v.
1.19. Hny olyan pozitv egsz szm van, amely nem oszthat 1-nl nagyobb ngyzetszmmal, sem
10-nl nagyobb prmszmmal ?
Megolds:
A feladat feltteleinek megfelel szmok prmtnyezs felbontsa a kvetkez alak:
n = 2
k
1
3
k
2
5
k
3
7
k
4
k
i
0, 1 i = 1, . . . ,4.
Ekkor a szmok ellltsa egyenrtk a fenti 4 kitev kivlasztsval. Azaz 4 helyre kell a
0; 1 szimblumok valamelyikt berni. Megintcsak egy ismtlses varicival van dolgunk. Az
sszesen teht
V
4
2
= 2
4
szm llthat el a feltteleknek megfelelen.
96 II.1. FEJEZET. PERMUTCIK, VARICIK, KOMBINCIK
1.20. Egy pakli magyar krtybl hnyflekppen adhatunk egy jtkosnak 8 lapot?
Megolds:
32 lap kzl szeretnnk 8-at kivlasztani gy, hogy a sorrendre nem vagyunk tekintettel.
1. lps
Elszr vlasszunk 8 lapot s tekintsnk klnbznek kt kivlasztst akkor is, ha csak a
kivlasztott lapok sorrendjben klnbznek. Ez a mlt rn trgyalt ismtlsnlkli varici
tmakr egy tpus-feladatra vezet. gy az ismert formula alapjn az ilyen kivlasztsok
szma:
V
8
32
= 32 31 30 29 28 27 26 25 =
32!
24!
=
32!
(328)!
2. lps Ekkor azonban egyes leosztsokat tbbszr is sszeszmoltunk, hiszen a jtkos
szempontjbl teljesen rdektelen, hogy a kezben lv lapokat milyen sorrendben kapta.
Egy-egy a feladat kirsa alpjn azonosnak tekintet leosztst pontosan annyiszor szmoltunk
meg, ahnyflekppen a jtkos kezben lv 8 klnbz lapot sorba lehet rakni. Ezen
sorbarendezsek szm P
8
=8!. Knnyen lthat, hogy minden tnylegesen klnbz leosztst
pontosan ennyiszer szmoltunk meg, ahelyett, hogy csak egyszer vettk volna gyelembe. gy
ha a kioszts sorrendjtl el szeretnnk tekinteni, akkor ezzel az rtkkel kell elosztani az els
lpsben kapott eredmnynket:
C
8
32
=
V
8
32
P
8
=
32!
(328)! 8!
=
_
32
8
_
1.21. Hnyflekppen oszthatunk szt egy pakli magyar krtyt (32 lap) 4 jtkosnak gy, hogy
mindegyikk 8-8 lapot kapjon?
Megolds:
Ezt a problmt mr trgyaltuk az ismtlses permutcik tmakrben (9. feladat). Az akkor
kapott eredmnynk:
P
8,8,8,8
32
=
32!
8! 8! 8! 8!
.
Most az elz feladat tlete alapjn induljunk el. Elszr vlasszuk ki azt a 8 lapot, amit az
1-es jtkos kap. Ez az elbb trgyalt gondolatmenet miatt
_
32
8
_
-flekppen lehetsges. Majd
a megmaradt 328 = 24 lap kzl vlasszuk ki azt a 8-at, amit a 2-es jtkos kap, majd a
3-as jtkosnak vlasszunk 8-at a maradk 16 lap kzl, vgl a 4-es jtkosnak vlasztunk
8-at a maradk 8-bl (vagyis az sszes megmaradt lapot nekiadjuk). Ezeket a lehetsgeket
a szorzsi szably alapjn kapcsoljuk ssze, gy az sszes lehetsges kioszts szma:
_
32
8
_

_
24
8
_

_
16
8
_

_
8
8
_
=
32!
24! 8!

24!
16! 8!

16!
8! 8!
1 =
32!
8! 8! 8! 8!
.
1.22. Egy kalapban klnbz szmkrtyk vannak, 1-tl 15-ig szmozva. A kalapbl visszatevs
nlkl hzunk 5-t. Hny esetben lesznek a kihzott elemek nvekv sorrendben?
Megolds: sszesen V
5
15
= 15 14 13 12 11-fle lehet a hzs eredmnye. Ebben persze
benne vannak azok is amelyeknl nem nvekv sorrendben vannak az elemek. Osztlyozzuk
a kivlasztsokat a kivlasztott elemek szerint, azaz kerljenek egy osztlyba azok az esetek,
amelyekben ugyanazon elemeket vlasztottuk, csak ms-ms sorrendben. Knnyen lthat,
hogy osztlyonknt pontosan egy kivlaszts tesz eleget a feladat feltteleinek. Msrszt a
II.1.1. KIDOLGOZOTT PLDK 97
fenti osztlyok mindegyike megfeleltethet az 1; 2; . . . ; 15 elemek egy-egy tdosztly,
ismtlsnlkli kombincijnak. gy az osztlyok szma
C
5
15
=
_
15
5
_
.
Megjegyzs: Az 1; 2; . . . ; n elem ismtlsnlkli kombincinak felrsa sorn az elemeket
ltalban nvekv sorrendben szoks rni.
1.23. Egy dobozban 500 csavar van, melyek kzl 10 darab selejtes. Hnyflekppen vlasztha-
tunk ki 15 csavart gy hogy 6 selejtes legyen kztk? s hogy legyen kztk 6 selejtes? (Az
els krds gy fogalmazhat meg msknt, hogy pontosan 6 selejtes, a msodik pedig, hogy
legalbb 6 selejtes legyen a kivlasztott csavarok kzt.)
Megolds:
a) Pontosan 6 selejtes lesz a kivlasztott 15 csavar kztt, ha a vlasztst a kvetkez mdon
vgezzk. Elszr vlasztunk 6-ot a 10 selejtes csavar kztt, majd 156 =9-et a 490 j
csavar kzl. Az elbbit
_
10
6
_
, mg az utbbit
_
490
9
_
-flekppen lehet megtenni. A szorzsi
szably alapjn a feladat feltteleinek megfelel kivlasztsok szma teht:
_
10
6
_

_
490
9
_
.
b) Legalbb 6 selejtes csavart hzunk az albbi diszjunkt esetek mindegyikben:
pontosan 6 selejtes:
_
10
6
_

_
490
9
_
pontosan 9 selejtes:
_
10
9
_

_
490
6
_
pontosan 7 selejtes:
_
10
7
_

_
490
8
_
pontosan 10 selejtes:
_
10
10
_

_
490
5
_
pontosan 8 selejtes:
_
10
8
_

_
490
7
_
Az sszeadsi szably alapjn ezen esetekhez tartoz lehetsgek sszege adja a feladatnak
megfelel kivlasztsok szmt:
_
10
6
_

_
490
9
_
+
_
10
7
_

_
490
8
_
+
_
10
8
_

_
490
7
_
+
_
10
9
_

_
490
6
_
+
_
10
10
_

_
490
5
_
.
1.24. Egy csomag francia krtya 52 lapbl ll, amelyek kzl 13-13 azonos szn. (A lapok rtkei
2, . . . ,10, J, Q, K, A).
a) Hnyflekppen vlaszthatunk ki kzlk 4, pronknt klnbz szn lapot?
b) Hnyflekppen vlaszthatunk ki kzlk 4 klnbz szn lapot, ha mg azt is megkveteljk,
hogy ne legyen kztk kt azonos rtk?
c) Hnyflekppen vlaszthatunk ki a krtyacsomagbl 4 lapot gy, hogy legyen kztk
legalbb kt sz?
Megolds:
98 II.1. FEJEZET. PERMUTCIK, VARICIK, KOMBINCIK
a) A feladat tulajdonkppen az, hogy mind a ngy sznbl vlasszunk egyet-egyet. Bontsuk
a krtyacsomagot 4 kis paklira, mghozz sznenknt. Ekkor minden kupacbl egyet-egyet
kell hznunk. Ez a korbbiak alapjn
_
13
1
_
4
= 13
4
-flekppen tehet meg.
Megjegyzs: A feladat felfoghat ltalnostott ismtlses variciknt is. Azrt kerlt
mgis ebbe a tmakrbe, mert a kombincik jval ltalnosabb trgyalst tesznek lehetv.
b) Az elz feladat gondolatmenett kvetve megint szinenknt vlasztunk egyet-egyet. gy
az els lap vlasztsra most is
_
13
1
_
lehetsgnk van. A msodik lap vlasztsakor az
els lappal azonos rtk lapot ki kell hagynunk, gy mr csak 12 lap kzl vlasztunk,
amire
_
12
1
_
lehetsgnk van. s gy tovbb. . . sszesen teht
_
13
1
_

_
12
1
_

_
11
1
_

_
10
1
_
= 13 12 11 10.
c) Most is bontsuk kis kupacokra a krtynkat. Az egyikbe kerljn a 4 sz, a msikba pedig
a tbbi lap. A hzst gy vgezzk, hogy elszr vlasztunk az szok kzl, majd a tbbi
lap kzl. A felttelnek (legalbb kt sz legyen a kivlasztottak kztt) az albbi diszjunkt
esetek felelnek meg.
pontosan 2 sz:
_
4
2
_

_
48
2
_
pontosan 3 sz:
_
4
3
_

_
48
1
_
pontosan 4 sz:
_
4
4
_

_
48
0
_
gy az sszeadsi szably alapjn:
_
4
2
_

_
48
2
_
+4 48+1
lehetsg van a kivlasztsra.
1.25. Hnyfle lyukaszts llthat be a buszjegy-lyukasztn, ha a szerkezet legalbb 2, de legfeljebb
4 szmot lyukaszt ki a 9 kzl ?
1 2 3
4 5 6
7 8 9
II.1.1. KIDOLGOZOTT PLDK 99
Megolds:
Bontsuk a lyukak szma alapjn diszjunkt esetekre a problmt. Kt lyukat
_
9
2
_
, hrom lyukat
_
9
3
_
, mg 4 lyukat
_
9
4
_
-flekppen lehet lyukasztani. gy a lehetsgek szma az sszeadsi
szably alapjn
_
9
2
_
+
_
9
3
_
+
_
9
4
_
= 246.
1.26. A fenti elrendezs buszjegyekbl szeretnnk sszegyjteni egy kszletet, mellyel az elz
felttelekkel belltott lyukasztknl biztosra mehetnk, azaz mindig tallunk egy olyan jegyet,
amely vagy j, vagy azt llthatjuk rla, hogy fordtva helyeztk a kszlkbe. Hny jegyre
van szksgnk?
Megolds:
Az elz feladat alapjn 246 jegy biztosan elg lenne. Nzzk, mennyit tudunk a trkknkkel
sprolni. A lyukasztsok kztt vannak szimmetrikusak, ezeket csak egyszer tudjuk hasznlni,
mg vannak aszimmetrikusak, melyek kt lyukaszt-bellts esetn is jk. Az tlet az lehet,
hogy az aszimmetrikus lyukasztsok kzl elegend csak minden msodikat megtartani. A
szimmetrikus eseteket valamivel knnyebb sszeszmolni. Kezdjk ezzel a feladat megoldst.
2 lyuk esetn:
Vagy mindkt lyuk a kzps oszlopban van (
_
3
2
_
=3 lehetsg), vagy az egyik lyuk a jobboldali
oszlopban van, mg a msik a baloldali oszlopban. Knnyen lthat, hogy ahhoz, hogy a
szimmetriai felttelnek megfeleljen a lyukaszts, a msodik lyuk helye mr nem vlaszthat
tetszlegesen, hiszen ugyanabban a sorban kell elhelyezkednie, amelyben az els lyuk is van.
Ilyen lyukasztsbl
_
3
1
_
= 3 darab van.
Szimmetrikus lyukasztsok 2 lyuk esetn: 3+3 = 6.
3 lyuk esetn:
Vagy mindhrom lyuk a kzps oszlopban van (
_
3
3
_
= 1 lehetsg), vagy az egyik lyuk
a jobboldali oszlopban, a msik a kzpsben, mg a harmadik a baloldali oszlopban. A
szimmetriai felttel miatt az utols lyuk helye mr nem vlaszthat tetszlegesen, hiszen
ugyanabban a sorban kell elhelyezkednie, amelyben az els lyuk is van. Ilyen lyukasztsbl
_
3
1
_

_
3
1
_
= 9 darab van.
Szimmetrikus lyukasztsok 3 lyuk esetn: 1+9 = 10.
4 lyuk esetn:
Vagy 2 lyuk van a kzps oszlopban (s ekkor egy-egy a jobb illetve a baloldali oszlopban),
vagy egy sincs s ekkor kt-kt lyuk van a szls oszlopokban. Az elbbi tpusbl a korbbi
indokls miatt
_
3
1
_

_
3
2
_
= 9, az utbbibl pedig
_
3
2
_
= 3 darab jegy tallhat.
Szimmetrikus lyukasztsok 3 lyuk esetn: 9+3 = 12.
sszes szimmetrikus elhelyezkedsek szma: 28. Ezek azok a lyukasztsok, amelyekhez egy-
egy jegyre mindenkppen szksg van.
Az aszimmetrikus lyukasztsok szma pedig 24628 = 218. Ezekhez feleennyi (109) jegy is
elg.
gy sszesen 137 jegyre van szksgnk.
1.27. 18 egyforma 10 forintost osztunk szt 4 gyerek kztt.
100 II.1. FEJEZET. PERMUTCIK, VARICIK, KOMBINCIK
a) Hnyfle mdon tehetjk ezt meg?
b) Hny eset van akkor, ha kiktjk, hogy minden gyerek kapjon legalbb egyet?
Megolds:
a) 1. mo:
Minden pnzrme al rjuk oda, hogy melyik gyerek kapta (1, 2, 3, 4 jelek valamelyikt).
Mivel az rmk darabszma rdekes csak a feladat szempontjbl, az elbbi jelsorozatot
rendezzk nvekv sorrendbe:
Sorszmozzuk be az rmket 1-tl 18-ig, majd adjuk ssze az rme sorszmt s az rme
alatti jelet. Knnyen lthat, hogy ekkor
-Minden rme alatt ms-ms szm ll
-Az rtkekre 2 k 18+4 = 22
Azaz 21 elem ismtlsnlkli 18-adosztly kombincijt kaptuk
_
21
18
_
2. mo:
A 10 forintosokat vlasszuk el egymstl vonalakkal. Az els vonal eltti rmket az 1-
es gyerek kapja stb. az utols vonal utniakat a 4-edik gyerek. Ekkor 18 rme s a 3
vlasztvonal minden klnbz elrendezshez az rmk egy kiosztsa rendelhet (s
viszont), gy a keresett rtk C
18
4
=P
3,18
21
=
21!
18!3!
(Az brn lthat elrendezs annak az esetnek felel meg, amikor az els s a msodik
gyerek 20-20 forintot kap, a harmadik gyerek nem kap semmit, a negyedik pedig megkapja
a maradk 140 forintot.)
3. mo:
Feleltessk meg egy tblzat sorait a gyerekeknek s az oszlopait a 10 forintosoknak. A k-
adik gyerek sorban lpjnk annyit jobbra, ahny tzforintost kapott, majd lpjnk egyet
lefel. Krds, hogy hnyflekppen juthatunk el az els sor nulladik helyrl a negyedik
sor 18-dik mezjbe.
A feladattal mr tallkoztunk az korbban (5. feladat). Itt 18 jobbralps s 3 lefele lps
elrendezseut vizsgljuk. gy a lehetsgek szma: P
3,18
21
=
21!
18!3!
.
II.1.1. KIDOLGOZOTT PLDK 101
b) A feladat megoldsa nem klnbzik lnyegben az elz rsztl. Mindenekeltt rdemes
biztostani az j felttel teljeslst. Ezt a legknnyebben gy tehetjk meg, ha minden
gyereknek az eloszts eltt adunk egy-egy 10 forintost. Ezutn a megmaradt rmket (14
darab) osztjuk szt az a) rszben trgyalt mdon. gy
C
14
4
=
17!
14! 3!
helyes kioszts van.
1.28. Hnyflekppen lehet 2011-es szmot
a) ht nemnegatv egsz szm
b) ht pozitv egsz szm
sszegre bontani, ha kt flbontst akkor is klnbznek tekintnk, ha csak a tagok sorrendjben
trnek el egymstl ?
Megolds:
Gyakorlatilag az elz feladat tfogalmazsrl van sz.
a) C
2011
7
b) C
2011
7
1.29. Hnyflekppen helyezhetnk el 40 klnbz dszhalat 2 egyforma nagy s 4 egyforma kicsi
akvriumban gy, hogy a nagy akvriumokban 10-10 a kicsikben pedig 5-5 hal legyen.
Megolds:
Ha az akvriumokat pronknt klnbznek tekintenm (mondjuk beszmozva ket), akkor
az elrendezsek szma
_
40
10
_

_
30
10
_

_
20
5
_

_
15
5
_

_
10
5
_

_
5
5
_
lenne, de ekkor klnbznek szmolnnk
azokat az eseteket, amelyek az egyforma akvriumok cserjvel egymsba vihetk. gy valjban
_
40
10
_

_
30
10
_

_
20
5
_

_
15
5
_

_
10
5
_

_
5
5
_
2! 4!
klnbz elrendezs van.
1.30. 8 goly kzl 4 piros s 1-1 kk, zld, fehr illetve fekete. Hnyflekppen llthatunk ssze ezek
kzl egy 4 golybl ll sorozatot, ha az azonos szn golyk egymstl megklnbztethetetlenek?
Megolds:
Bontsuk szt az eseteket aszerint, hogy hny piros golyt vlasztottunk ki.
4 piros:
Ekkor msik golyra nincs szksgnk, gy a golyk kivlasztsra egyetlen lehetsg addik
s mivel a golyk megklnbztethetetlenek, a sorbarendezsek sem adnak j lehetsget: 1
lehetsg.
3 piros:
Ekkor a negyedik goly a msik 4 sznes goly brmelyike lehet, gy a sznsszellts
kivlasztsra 4 lehetsgnk van. Minden sznsszellts sorn P
1,3
4
=4 sorbarendezs lehetsges.
gy sszesen a szorzsi szably alapjn 4 4 jabb lehetsghez jutottunk.
102 II.1. FEJEZET. PERMUTCIK, VARICIK, KOMBINCIK
2 piros:
Ekkor a harmadik goly a msik 4 sznes goly brmelyike lehet, mg a negyedik goly
vlasztsra 3 lehetsgnk van, gy 43=12-fle sznsszellts lehet. Minden sznsszellts
sorn P
1,1,2
4
=
4!
2!
=12 sorbarendezs lehetsges. gy sszesen a szorzsi szably alapjn 12
2
=144
jabb lehetsghez jutottunk.
II.1.2. Tovbbi gyakorl feladatok
1.31. Egy fut versenyen nyolc fut kerlt a dntbe. A dntben hny klnbz befutsi sorrend
lehetsges, ha felttelezzk, hogy nem volt holtverseny?
eredmny
1.32. Hny olyan tjegy szm kpezhet az 1, 2, 3, 4, 5 szmjegyekbl, amelyben az 1-e s a 3-as
jegy egyms mellett helyezkedik el s minden szmjegyet pontosan egyszer hasznlhatunk
fel ? s ha a 2-es szmjegyet 0-ra cserljk?
eredmnyek
1.33. Hny tjegy pros szm kpezhet a 0, 1, 2, 3, 4 szmjegyekbl, ha minden szmjegy csak
egyszer szerepelhet?
eredmny
1.34. Hnyflekppen lhet le 9 ember egy kilencszemlyes hossz padra?
eredmny
1.35. Hnyflekppen lhet le 4 n s 5 fr egy kilencszemlyes hossz padra, gy hogy azonos
nemek ne kerljenek egyms mell?
eredmny
1.36. Hnyflekppen lhet le 4 n s 5 fr egy kilencszemlyes hossz padra, gy hogy azonos
nemek ne kerljenek egyms mell, ha csak az szmt, frrl vagy nrl van-e sz?
eredmny
1.37. Hnyflekppen lhet le 9 ember egy kerekasztal krl, ahol kilenc szk van?
eredmny
1.38. Hnyflekppen lhet le 9 ember egy kerekasztal krl, ahol kilenc szk van, gy hogy kzlk
kt kitntetett (A s B) egyms mell kerljn?
eredmny
II.1.2. TOVBBI GYAKORL FELADATOK 103
1.39. Hnyflekppen lhet le 4 n s 5 fr egy kerekasztal krl, ahol kilenc szk van gy hogy
azonos nemek ne kerljenek egyms mell?
eredmny
1.40. Hnyfle kppen olvashat ki a kvetkez brbl KOMBINATORIKA sz, ha a tblzat bal
fels sarkbl indulunk s minden lpsben csak jobbra vagy lefel haladhatunk:
K O M B I N A
O M B I N A T
M B I N A T O
B I N A T O R
I N A T O R I
N A T O R I K
A T O R I K A
eredmny
1.41. Hnyflekppen lehet eljutni az origbl a (4, 7, 3) pontba, gy, hogy csak egysgnyi hossz
jobbra, fel s elre lpsek lehetsgesek?
eredmny
1.42. Adott n klnbz elem. Ha az elemekhez hozzvesznk kt, az utolsval megegyez elemet,
akkor az elemek sszes klnbz sorbarendezseinek szma 22-szerese lesz, mint az eredeti
n elem sorbarendezseinek a szma.
megolds
1.43. Hny permutcija van az ASZTALLAP, s a KUTYAFUTTAT szavaknak?
eredmny
1.44. A 0, 1, 1, 1, 2, 2, 5 szmjegyek permutlsval hny
a) pros,
b) pratlan,
c) nggyel oszthat,
d) ttel oszthat,
e) 12-vel oszthat
szm kpezhet, ha a szmok nem kezddhetnek 0-val ?
megoldsok
1.45. Hnyflekppen oszthatunk szt 25 klnbz trgyat 5 szemly kztt egyenl arnyban?
eredmny
104 II.1. FEJEZET. PERMUTCIK, VARICIK, KOMBINCIK
1.46. Tizenkt tanul kztt hnyflekppen lehet kiosztani t klnbz trgyat, ha egy tanul
a) legfeljebb egy trgyat kaphat?
b) tbb trgyat is kaphat?
megolds
1.47. Az 1; 2; 3; 4; 5; 6; 7; 8; 9 szmjegyekkel hny pros, tjegy szmot kpezhetnk, ha
brmely kt szomszdos jegy klnbz parits s
a) minden jegyet legfeljebb egyszer hasznlunk?
b) Ugyanaz a jegy tbbszr is felhasznlhat?
eredmny
1.48. Tizes-szmrendszerbeli hatjegy szmokat kpeznk.
a) Hny ilyen szm van?
b) Hny pros illetve hny pratlan szm van?
c) Hny 4-gyel oszthat szm van?
d) Hny 5-tel oszthat szm van?
e) Hny 3-mal oszthat szm van?
eredmnyek, megoldsok
1.49. Kettes-szmrendszerbeli hatjegy szmokat kpeznk.
a) Hny ilyen szm van?
b) Hny pros illetve hny pratlan szm van?
c) Hny 4-gyel oszthat szm van?
d) Hny 8-cal oszthat szm van?
eredmnyek, magyarzatok
1.50. Tizenhatos-szmrendszerbeli hatjegy szmokat kpeznk.
(A szmjegyek: 0; 1; 2; 3; 4; 5; 6; 7; 8; 9; A; B; C; D; E; F)
a) Hny ilyen szm van?
b) Hny pros illetve hny pratlan szm van?
c) Hny 4-gyel oszthat szm van?
d) Hny 32-vel oszthat szm van?
eredmnyek, magyarzatok
II.1.2. TOVBBI GYAKORL FELADATOK 105
1.51. Tekintsk az albbi brn lthat hinyos mtrixokot. (Tridiagonlis mtrix rtkes elemeket
tartalmaz mezi. Ha gy tetszik, akkor egy szttrtt sakktbla.) Hnyflekppen juthatunk
el a bal fels mezbl a jobb als mezbe, gy hogy minden lpsnl egy mezt jobbra, vagy
egy mezt lefel lpnk s a kijellt terletet nem hagyhatjuk el ?
ltalnostsuk a problmt s 88-as mtrix helyett vizsgljunk nn-es mtrixot!
megolds
1.52. Az 1; 2; 3; 4; 5; 6; 7; 8; 9 elemek hny negyedosztly ismtlsnlkli kombincija
tartalmazza az 1; 2 elemeket?
eredmnyek
1.53. Hny olyan tjegy szm van, melynek jegyei
a) nvekv sorrendben
b) nem cskken sorrendben
kvetkeznek egyms utn?
eredmnyek
1.54. Az ts, vagy a hatos lottn lehet kevesebb szelvnnyel biztosan telitallatot elrni. (Az ts
lott esetn 90 szm kzl kell tt eltallni, a hatoson 45-bl 6-ot.)
megolds
1.55. Hnyfle lehet a lott-hzs eredmnye, ha tudjuk, hogy csupa egymsutni szmot hztak
ki. (A lotthzs sorn 90 szm kzl hznak ki 5-t gy, hogy a kihzott elemek sorrendje
rdektelen.)
megolds
1.56. Hnyfle lehet a lott-hzs eredmnye, ha tudjuk, hogy kihztak kt pr egymsutni
szmot, de nem hztak hrom egyms utnit?
megolds
106 II.1. FEJEZET. PERMUTCIK, VARICIK, KOMBINCIK
1.57. Egy vfolyam 50 lny s 30 hallgatja ttag kldttsget vlaszt, spedig 3 lnyt s 2
t. Hnyflekppen teheteik ezt meg? Hnyflekppen vlaszthatnak kldttsget akkor, ha
Ancsa s Berci ppen haragban vannak, ezrt nem akarnak egytt bekerlni.
megolds
1.58. Egy postahivatalban 10-fle kpeslapot rulnak. Hnyflekppen vsrolhatunk
a) 8 klnbz kpeslapot?
b) 8 kpeslapot?
c) 12 kpeslapot?
eredmnyek
1.59. Hny sztrt kell kiadnunk, hogy kzvetlenl tudjunk fordtani 10 klnbz nyelv kzl
brmelyikrl brmelyik msikra?
eredmny
II.2. fejezet
A binomilis s a polinomilis ttel
II.2.1. Kidolgozott pldk
2.1. Mennyi lesz az x
2
y
3
egytthatja (2x+3y)
5
-ben?
Megolds:
A binomilis ttel alapjn az a
2
b
3
-s tag egytthatja az (a+b)
5
kifejtsben
_
5
2
_
. Ne feledkezznk
el azonban arrl, hogy a feladatban a = 2x, gy a
2
= 2
2
x
2
s hasonlan b = 3y, gy b
3
= 3
3
y
3
,
gy a krdses egytthat
_
5
2
_
2
2
3
3
2.2. Igazoljuk, hogy
_
n
k1
_
+2
_
n
k
_
+
_
n
k+1
_
=
_
n+2
k+1
_
.
Megolds:
1. Megolds:
_
n
k1
_
+2
_
n
k
_
+
_
n
k+1
_
=
n!
(k1)! (nk+1)!
+2
n!
k! (nk)!
+
n!
(k+1)! (nk1)!
=
=
(k+1) k n!
(k+1)! (nk+1)!
+
2 (k+1) (nk+1) n!
(k+1)! (nk+1)!
+
(nk+1) (nk) n!
(k+1)! (nk+1)!
=
=
n! ((k
2
+k)+(2nk2k
2
+2k+2n2k+2)+(n
2
2nk+n+k
2
k))
(k+1)! (nk+1)!
=
=
n! (n
2
+3n+2)
(k+1)! (nk+1)!
=
n! (n+1) (n+2)
(k+1)! (nk+1)!
=
(n+2)!
(k+1)! ((n+2)(k+1))!
=
=
_
n+2
k+1
_
2. Megolds:
Kombinatorikai megkzeltssel : Egy zskban n darab pronknt klnbz piros s 2 klnbz
fehr goly van. Hnyflekppen lehetsges kzllk k+1 darabot kihzni gy, hogy a kivlaszts
sorrendjre nem vagyunk tekintettel.
107
108 II.2. FEJEZET. A BINOMILIS S A POLINOMILIS TTEL
A lehetsgeket ktflekppen szmoljuk ssze. Elszr gy tekintjk, hogy n+2 pronknt
klnbz elem kzl kell k+1-et vlasztanunk a sorrendre val tekintet nlkl, amely ismert
sszeszmllsi problma. A lehetsgek szma
_
n+2
k+1
_
.
Msodszor a kivlasztsokat csoportostsuk a kivlasztott fehr golyk szma alapjn:
csak piros golykat vlasztunk, ekkor a lehetsgek szma:
_
2
0
_

_
n
k
_
=
_
n
k
_
k1 piros s 1 fehr golyt vlasztunk, ekkor a lehetsgek szma:
_
2
1
_

_
n
k1
_
= 2
_
n
k1
_
mind a kt fehr golyt kivlasztottuk, ekkor a lehetsgek szma:
_
2
2
_

_
n
k2
_
=
_
n
k2
_
A fenti diszjunkt esetek unija adja az sszes kivlasztsokat, gy az sszes lehetsgek szma
a fenti alesetekhez tartoz lehetsgek szmnak sszege. gy ppen a bizonytand lltst
kaptuk:
_
n
k1
_
+2
_
n
k
_
+
_
n
k+1
_
=
_
n+2
k+1
_
.
2.3. Igazoljuk, hogy
n

k=0
k
_
n
k
_
=n 2
n1
A feladatra az elmleti rsz trgyalsa sorn mr adtunk hrom megoldst, lsd I.2.1.6 s
I.6.6, most lssunk egy jabb megkzeltsi mdot:
Kombinatorikai megkzelts:
Egy n tag trsasgbl vlasszunk egy bizottsgot s a bizottsgnak egy elnkt. A feladat
ktflekppen oldhat meg.
1.) Elszr vlasszunk egy k tag bizottsgot s a bizottsg tagjai kzl vlasszunk egy
elnkt. Ez
_
n
k
_

_
k
1
_
=
_
n
k
_
k.
Ezeket sszegezzk, amg a k az sszes lehetsges rtken vgigfut, amelyeket a bizottsg
ltszma felvehet.
2.) Elszr az elnkt vlasztjuk ki, majd a maradk n 1 szemly esetn mindenkinl
eldnthejk, hogy tagja legyen-e a bizottsgnak. gy a lehetsgek szma:
_
n
1
_
2
n1
=n 2
n1
.
2.4. Igazoljuk, hogy
n

k=0
(1)
k
_
n
k
_
= 0.
Megolds:
A binomilis ttel (a+b)
n
=
n

k=0
_
n
k
_
a
k
b
nk
alakjba a =1 s b = 1 rtkeket helyettestve
az llts azonnal addik.
II.2.1. KIDOLGOZOTT PLDK 109
2.5. Mutassuk meg, hogy minden nem res halmaznak pontosan annyi pros elem rszhalmaza
van, mint ahny pratlanelem!
1. Megolds:
A feladat lltsa gyakorlatilag megegyezik a 4 feladatban bizonytott lltssal.
2. Megolds:
Tekintsk a halmaz egy rgztett x elemt. Minden A
i
rszhalmaz esetn, melyre x / A
i
egyrtelmen ltezik a B
i
= A
i
x halmaz. Knnyen lthat, hogy az A
i
s B
i
halmazok
klcsnsen egyrtelm mdon megfeleltethetk egymsnak. A halmaz minden rszhalmaza
besorolhat az A
i
vagy B
i
halmazok kz. Rgzitett i index esetn A
i
s B
i
elemszma
klnbz parits. A megfeleltets bijektv voltbl kvetkezik az llts.
2.6. Adjuk meg (2x+3y z)
3
kifejtsben az yz
2
-es tag egytthatjt!
A polinomilis ttel alapjn az bc
2
-es tag egytthatja az (a+b+c)
3
kifejtsben
3!
0!1!2!
. Ne
feledkezznk el azonban arrl, hogy a feladatban b =3y s c =z, gy c
2
=z
2
, azaz a krdses
egytthat
3!
0! 1! 2!
3 1 = 9.
Megolds:
A polinomilis ttel alapjn:
(a+b+c)
3
=
3!
0! 0! 3!
_
a
3
b
0
c
0
+a
0
b
3
c
0
+a
0
b
0
c
3
_
+
+
3!
0! 1! 2!
_
a
2
b
1
c
0
+a
2
b
0
c
1
+a
0
b
2
c
1
+a
0
b
1
c
2
+a
1
b
0
c
2
+a
1
b
2
c
0
_
+
+
3!
1! 1! 1!
a
1
b
1
c
1
2.7. Igazoljuk, hogy

k
_
r
k
__
s
nk
_
=
_
r +s
n
_
.
Megolds:
r hlgy s s r kzl vlasszunk n-tag kldttsget. Hnyflekppen llthat ssze a kldttsg?
Elszr szmoljuk ssze a lehetsgeket gy, hogy nem vagyunk tekintettel arra, hogy a
kldttsgbe hny hlgy kerlt. Ekkor gyakorlatilag arrl van sz, hogy r +s szemly kzl
kell (a sorrendre val tekintet nlkl) kivlasztani n-et. Ez
_
r+s
n
_
-flekppen lehetsges.
Osztlyozzuk most a kldttsgeket az alapjn, hogy hny hlgyet vlasztottunk be:
0 hlgy:
_
r
0
_

_
s
n
_
1 hlgy:
_
r
1
_

_
s
n1
_
.
.
.
n hlgy:
_
r
n
_

_
s
0
_
110 II.2. FEJEZET. A BINOMILIS S A POLINOMILIS TTEL
A felsorolt esetek egymst pronknt kizrjk gy a lehetsgek szma a fentiek sszege:
n

k=0
_
r
k
__
s
nk
_
.
Megjegyzsek: A problma trgyalsa sorn vagy gyelnnk kellene r, s, n viszonyra (a fenti
tblzat nminr, s esetet mutatja, vagy a binomilis egytthatk fogalmt ltalnostanunk
kell. Legyen a <b, ekkor
_
a
b
_
:= 0. Ezzel a kiegsztssel az eset sztvlaszts megsprolhat.
Ez nem ms, mint a Vandermonde-azonossg, lsd elmleti rsz, I.2.3.4.
2.8. Igazoljuk, hogy
n

k=1
(1)
k+1
k
_
n
k
_
= 0.
Megolds:
n

k=1
(1)
k+1
k
_
n
k
_
=
n

k=1
(1)
k+1
k
n!
k! (nk)!
=
n

k=1
(1)
k+1
n!
(k1)! (nk)!
=
=n
n

k=1
(1)
k+1
(n1)!
(k1)! ((n1)(k1))!
=n
n

k=1
(1)
k+1
_
n1
k1
_
=
=n
n1

j=0
(1)
j+2
_
n1
j
_
=n
n1

j=0
(1)
j
_
n1
j
_
. .
=0
= 0.
II.2.2. Tovbbi gyakorl feladatok
2.9. Igazoljuk, hogy
_
n
1
_
+
_
n
3
_
+
_
n
5
_
= 2
n1
.
tmutats
2.10. Igazoljuk, hogy ha 0 r n, akkor
_
n
0
__
n
r
_
+
_
n
1
__
n
r 1
_
+
_
n
2
__
n
r 2
_
+... +
_
n
r
__
n
0
_
=
_
2n
r
_
.
tmutats
2.11. Igazoljuk, hogy minden n 2-re
n

k=2
k(k1)
_
n
k
_
=n(n1)2
n2
.
Adjunk kombinatorikus bizonytst is.
tmutats
II.2.2. TOVBBI GYAKORL FELADATOK 111
2.12. Igazoljuk, hogy minden n 2-re
_
n
2
_
+2
_
n
3
_
+3
_
n
4
_
+. . . +(n1)
_
n
n
_
= (n2) 2
n1
+1.
tmutats
2.13. Szmtsuk ki az
n

i=k
_
n
i
__
i
k
_
(1)
ik
sszeget, ahol 1 k n.
tmutats
2.14. Legyen
a
n
=
1
_
n
0
_ +
1
_
n
1
_ + +
1
_
n
n
_, n 0.
Igazoljuk, hogy a
n+1
=
n+2
2(n+1)
a
n
+1, n 0, s a
n
2 mikor n .
megolds
2.15. Szmtsuk ki a
n

i=k
_
n
i
__
i
k
_
(1)
ik
sszeget, ahol 1 k n.
megolds
2.16. Igazoljuk, hogy az
_
n
k
_
=
_
n+k1
k
_
ismtlses kombincikra vonatkozan:
_
n
k
_
=
_
n1
k
_
+
_
n
k1
_
, (n, k 1).
megolds
112 II.2. FEJEZET. A BINOMILIS S A POLINOMILIS TTEL
II.3. fejezet
Szitakpletek
II.3.1. Kidolgozott pldk
3.1. Egy osztlyban angolul, franciul, nmetl rendre 16, 6, 7 tanul tanul. Ngyen angolul is s
nmetl is, hrman nmetl s franciul, ketten franciul s angolul is tanulnak. Egy tanul
mindhrom nyelvet tanulja. Hnyan vannak az osztlyban, ha mindenki tanul legalbb egy
nyelvet? Hnyan tanulnak csak angolul ?
Megolds:
Ha az angolul, franciul illetve nmetl tanulk szmt sszegezzk, akkor azokat akik kt
nyelvet is tanulnak ktszer, azokat, akik mindhrom nyelvrra jrnak pedig hromszor
szmoltuk bele a ltszmba. Ilyen esetekben alkalmazzuk a logikai szitaformult:
[AFN[ =[A[+[F[+[N[([AF[+[AN[+[FN[)+[AFN[ =16+6+7(2+4+3)+1=21.
3.2. Az egymillinl kisebb nemnegatv egsz szmok kzl hny tartalmazza a 3, 4, 7, 9 jegyek
mindegyikt?
Megolds:
Az 1000000 darab egymillinl kisebb nemnegatv egsz szm kzl elszr vonjuk ki azokat,
amelyek a felsorolt szmjegyek kzl legalbb az egyiket nem tartalmazzk:
10000004 9
6
,
de ekkor azokat a szmokat tbbszr hagytuk el, amelyek a fentiek kzl legalbb kettt nem
tartalmaznak, ezeket adjuk vissza:
10000004 9
6
+
_
4
2
_
8
6
.
Most viszont a kelletnl tbbszr szmoltuk be azokat a szmokat, amelyek a jegyek kzl
legfeljebb egyet tartalmaznak, ezeket meg csak le kell szmtani :
10000004 9
6
+
_
4
2
_
8
6

_
4
3
_
7
6
,
vgl azokat a szmokat, amelyek egyik vizsglt jegyet sem tartalmazzk jra vissza kell
adnunk, hiszen az elbb tbbszr vontuk le ket:
10000004 9
6
+
_
4
2
_
8
6

_
4
3
_
7
6
113
114 II.3. FEJEZET. SZITAKPLETEK
3.3. 1-tl 150-ig hny sszetett s hny prm szm van?
Megolds:
1-tl 150-ig az sszetett szmok az albbi prmek valamelyikvel oszthatk: 2, 3, 5, 7, 11,
mivel a kvetkez prm (13) mr nagyobb mint

150. Jellje S
k
a kval oszthatk szmt.
Ekkor a felsorolt prmek kzl legalbb eggyel oszthatk:
S
2
=
_
150
2
_
= 75
S
3
=
_
150
3
_
= 50
S
5
=
_
150
5
_
= 30
S
7
=
_
150
2
_
= 21
S
11
=
_
150
11
_
= 13
a felsorolt prmek kzl legalbb kettvel oszthatk:
S
2,3
=S
6
=
_
150
6
_
= 25
S
2,5
=S
10
=
_
150
10
_
= 15
S
2,7
=S
14
=
_
150
14
_
= 10
S
2,11
=S
22
=
_
150
22
_
= 6
S
3,5
=S
15
=
_
150
15
_
= 10
S
3,7
=S
21
=
_
150
21
_
= 7
S
3,11
=S
33
=
_
150
33
_
= 4
S
5,7
=S
35
=
_
150
35
_
= 4
S
5,11
=S
55
=
_
150
55
_
= 2
S
7,11
=S
77
=
_
150
77
_
= 1
a felsorolt prmek kzl legalbb hrommal oszthatk:
S
2,3,5
=S
30
=
_
150
30
_
= 5
S
2,3,7
=S
42
=
_
150
42
_
= 3
S
2,3,11
=S
66
=
_
150
66
_
= 2
S
2,5,7
=S
70
=
_
150
70
_
= 2
S
2,5,11
= S
110
=
_
150
110
_
= 1
S
2,7,11
=S
154
=
_
150
154
_
= 0
S
3,5,7
=S
105
=
_
150
105
_
= 1
S
3,5,11
=S
165
=
_
150
165
_
= 0
S
3,7,11
=S
231
=
_
150
231
_
= 0
S
5,7,11
=S
385
=
_
150
385
_
= 0
II.3.1. KIDOLGOZOTT PLDK 115
a felsorolt prmek kzl legalbb nggyel oszthatk:
S
2,3,5,7
= S
210
=
_
150
210
_
= 0
Az utols eredmnybl ltszik, hogy nincs olyan 150-nl kisebb szm amely legalbb ngy
vizsglt prmmel oszthat lenne. Ekkor a logikai szita-formula alapjn a prmek szma:
(150)=150(75+50+30+21+13)+(25+15+10+6+10+7+4+4+2+1)(5+3+2+2+1+1)+51=35.
Az utols kt mvelet kisebb magyarzatra szorul. A legalbb egy prm osztval rendelkez
sszetett szmok helyett valjban a vizsglt prmjeink tbbszrseit szmoltuk meg. Ezek
kztt termszetesen szerepelnek maguk a prmek is, akiket gy tvesen vontunk le.
A fennmarad szmok teht olyanok, amelyeknek nincs valdi prm osztjuk. Ezek egy
kivtellel mind prmek is. A kivtel az 1. Teht sszegezve 1-tl 150-ig 35 darab prm szm,
114 darab sszetet szm s az 1 tallhat.
3.4. Hnyflekppen ltethetnk le egy sorba hrom angolt hrom francit s hrom trkt gy,
hogy hrom azonos nemzetisg ne ljn egyms mell?
Megolds:
9!
_
3
1
_
7! 3! +
_
3
2
_
5! 3! 3!
_
3
3
_
3! 3! 3! 3!
3.5. Aladr levelet rt 10 bartjnak. Pont mikor bortkba akarta rakni a leveleket elaludt a
villany. A leveleket ekkor vletlenszeren rakta be a bortkokba. Hnyfle olyan elrendezs
van, hogy senki ne a neki szl levelet kapja meg?
Megolds:
10!
_
10
1
_
9! +
_
10
2
_
8!
_
10
3
_
7! +
_
10
4
_
6!
_
10
5
_
5! +
_
10
6
_
4!
_
10
7
_
3! +
_
10
8
_
2!
_
10
9
_
1! +
_
10
10
_
0!
3.6. Adott egy X alaphalmaz s annak A
1
, A
2
, . . . , A
n
rszhalmazai. Hny elem tartozik az A
i
-k
kzl pontosan 2-be?
Megolds:

[A
i
A
j
[
_
3
2
_

[A
i
A
j
A
k
[+
_
4
2
_

[A
i
A
j
A
k
A

[ +(1)
n

_
n
2
_
[A
1
A
2
. . .A
n
[
3.7. Hnyflekpen lhet le 8 hzaspr egy kerekasztal kr gy, hogy minden n kt fr kz l,
de egyik n sem l a frje jobboldalra?
Megolds:
Az sszes ltetsek szma: 7! 8!
Ha legalbb egy hlgy a frje jobbjn l :
_
8
1
_
7! 7!
s gy tovbb. . . Ekkor a helyes ltetsek szma a szitaformula alapjn az albbi mdon
szmolhat:
7! 8!
_
8
1
_
7! 7! +
_
8
2
_
7! 6!
_
8
3
_
7! 5! + +(1)
8

_
8
8
_
7! 0!
116 II.3. FEJEZET. SZITAKPLETEK
3.8. Hny olyan pozitv egsz szm van, amely osztja a 10
40
s 20
30
szmok valamelyiknek?
Megolds:
Szmoljuk ssze 10
40
s 20
30
osztit, majd vonjuk ki a kzs osztkat, mert ezeket dupln
szmoltuk!
10
40
= 2
40
5
40
oszti 2
p
5
q
alakak, ahol 0 p, q 40 egszek.
gy mind p, mind tle fggetlenl q vlasztsra 40 lehetsgnk van, vagyis 10
40
-nek 40
2
=
= 1600 osztja van.
Hasonlan 20
30
= 2
60
5
30
oszti 2
p
5
q
alakak, ahol 0 p 60 s 0 q 30 egszek.
gy 20
30
-nak 60 30 = 1800 osztja van.
A kzs osztk mindegyike osztja a legnagyobb kzs osztnak d=(10
40
,20
30
)=(2
40
5
40
,2
60
5
30
)=
= 2
40
5
30
.
d = 2
40
5
30
osztinak szmt az elzekhez hasonlan szmolhatjuk: 40 30 = 1200.
gy sszesen
1600+18001200 = 2200
olyan szm van, amely a fenti szmok valamelyiknek (esetleg mindkettnek) osztja.
3.9. Hnyflekppen tudunk eltenni 20 klnbz paprlapot egy piros, egy kk, egy zld s egy
srga irattartba gy, hogy mindegyik irattartba kerljn legalbb egy papr s a dosszin
bell a sorrend kznbs?
Megolds:
Logikai szitaformult alkalmazva:
Az sszes elrendezsek szma: 20
4
, hiszen minden paprlap esetn 4 irattart kzl vlaszthatunk.
Az olyan elrendezsek szma, ahol (legalbb) egy irattart resen marad:
_
4
1
_
20
3
.
Az olyan elrendezsek szma, ahol (legalbb) kt irattart resen marad:
_
4
2
_
20
2
.
Az olyan elrendezsek szma, ahol hrom irattart resen marad:
_
4
3
_
20.
gy az sszes helyes elrendezsek szma:
20
4

_
4
1
_
20
3
+
_
4
2
_
20
2

_
4
3
_
20.
3.10. Egy zskban 10 pr cip van. Hnyflekppen vehetnk ki belle 6 darabot gy, hogy ne
legyen kztk egy pr sem?
1. Megolds:
Logikai-szitaformulval :
_
20
6
_

_
10
1
_

_
18
4
_
+
_
10
2
_

_
16
2
_

_
10
3
_

_
14
0
_
= 13440.
2. Megolds:
Direkt szmolssal :
II.3.2. TOVBBI GYAKORL FELADATOK 117
A 6 cipt gy vlasztjuk, hogy elszr eldntjk, hogy melyik 6 prbl vlasztunk (
_
10
6
_
lehetsg), majd minden kivlasztot pr esetn eldntjk, hogy a jobb- vagy a ballbas cpre
van-e szksgnk. (2
6
lehetsg). gy a szorzsi szably alapjn sszesen
_
10
6
_
2
6
kivlaszts lehetsges.
II.3.2. Tovbbi gyakorl feladatok
3.11. Adott egy X alaphalmaz s annak A
1
, A
2
, . . . , A
n
rszhalmazai. Hny elem tartozik az A
i
-k
kzl pontosan 3-ba?
eredmny
3.12. Hnyflekppen ltethetnk le egy kerekasztal krl hrom angolt hrom francit s hrom
trkt gy, hogy hrom azonos nemzetisg ne ljn egyms mell?
megolds
3.13. Hny olyan rszhalmaza van az 1,2 . . . ,10 halmaznak, amely tartalmaz legalbb egy pratlan
szmot?
eredmny
3.14. Hny olyan rszhalmaza van az 1,2 . . . ,10 halmaznak, amely tartalmaz legalbb hrom
pratlan szmot?
eredmny
3.15. Hnyflekppen vlaszthat ki az augusztus hnap 5 napja gy, hogy ne legyenek kztk
egymsutni napok?
megolds
3.16. Az 1,2, . . . n szmoknak hny olyan permutcija van, hogy 1,2, . . . , k nem llnak egyms
utn
i) ebben a sorrendben,
ii) valamilyen sorrendben.
megolds
118 II.3. FEJEZET. SZITAKPLETEK
II.4. fejezet
sszeszmllsi feladatok
II.4.1. Kidolgozott pldk
4.1. Hnyflekppen juthatunk el az nn-es sakktbln a bal als sarokbl a jobb fels sarokba,
ha minden lpssel kzelednk az [[ [[
1
-es vektornorma rtelmben?
([[x[[
1
=
n

i=1
[x
i
[)
Megolds:
A megfogalmazs annyit takar, hogy minden lpsben egyet jobbra, vagy egyet fel lpnk.
Ezzel a problmakrrel mr foglalkoztunk korbban. Az albbi kt megkzeltssel elemi
sszeszmllsi problmra vezethetjk a feladatot.
1. Megolds: n1 darab jobbra illetve n1 darab fel jelet kell az sszes lehetsges mdon
sorbarendezni. (Az azonos irnyokhoz tartoz jelek pronknt megklnbztethetetlenek) gy
egy-egy tvonalhoz egy-egy ismtlses permutcit rendeltnk, mghozz bijektv mdon.
Ezek szma:
P
n1,n1
2n2
=
(2n2)!
(n1)! (n1)!
.
2. Megolds: 2n2 lpsben jutunk el a bal als sarokbl a jobb felsbe. Ezen lpsek
kzl vlasszuk ki azt az n1 darabot (a sorrendre val tekintet nlkl), amelyekben jobbra
lpnk. gy egy-egy tvonalhoz egy-egy ismtlsnlkli kombincit rendeltnk, mghozz
bijektv mdon. Ezek szma:
C
n1
2n2
=
(2n2)!
(n1)! (n1)!
.
4.2. 16 csapatot 4 darab 4-es csoportba osztottunk, a csoportokon bell egy-forduls krmrkzsek
dntik el a sorrendet. Minden csoport els helyezettje bejut a 4-es dntbe, ahol a 4 csapat
megint krmrkzst jtszik egymssal. Hnyfle lehet a 4-es dnt meznye, ha a csoportok
beosztst rgztettnek tekintjk?
Gondoljuk meg a problmt kt ts s kt hrmas csoport esetn is!
Megolds:
A feladat ltal vzolt problmt az ismtlses permutci ltalnostsnak tekinthetjk.
A lehetsgek szma 4 4 4 4. Az els csoportbl ugyanis 4-fle csapatot vlaszthatunk.
Brmelyik csapat jutott is tovbb, a msodik csoportbl val kivlasztssal a sort 4-flekppen
119
120 II.4. FEJEZET. SSZESZMLLSI FELADATOK
folytathatjuk, gy az els kt csapat 4
2
-flekppen vlaszthat ki. s gy tovbb. . . Ezzel a
szorzsi szably formuljhoz jutunk.
Ha kt ts s kt hrmas csoport sorn kell a tovbbjutkat kivlasztani, akkor ez a fenti
gondolatmenetet kvetve 5 3 5 3-flekppen tehet meg.
4.3. A knyvespolcon 12 knyv ll. Hnyflekppen lehet kzlk kivlasztani tt gy, hogy ezek
kztt ne legyen kt egyms meletti ?
Megolds:
Minden kivlasztsnak megfeleltethetnk egy 5 egyesbl s 7 nullbl ll 12 bites sorozatot.
Azok a j sorozatok, ahol nem ll egymsmellett 2 egyes jegy. rjuk le a nullkat gy, hogy
kztk, elttk illetve mgttk egy-egy helyet hagyunk az egyeseknek.
gy az 5 darab egyesnket 8 helyre rakhatjuk. A 8 hely kzl kell tehet 5-t kivlasztani ezt
_
8
5
_
-flekppen
tehetjk meg. Lsd mg elmleti rsz, I.4.1.9 Feladat.
4.4. Arthur kirly kerek asztala krl 12 lovag l. Minden lovag haragban van a szomszdaival
(s csak velk). A kirlynak egy ttag csapatot kell lovagjaibl sszelltania gy, hogy a
kivlasztott lovagok mindegyike bkben legyen a msik nggyel. Hnyflekppen teheti ezt
meg a kirly?
Megolds:
Asszerint, hogy egy kitntetett lovag, nevezzk mondjuk Sir Ny-nek, tagja-e a csapatnak,
kt diszjunkt osztlyra bonthatjuk a lehetsgeket.
a) Ha Sir Ny a csapat tagja, akkor szomszdai, 1-es s
11-es lovag biztosan nem kerlhetnek kivlasztsra.
Az brn Sir Ny-t ponttal jelltk. A maradk
kilenc lovag kzl (2-10) gy kell ngyet vlasztani,
hogy ne legyen kztk kt egyms melletti. Azzal,
hogy kivlasztottuk Sir Nyt, a kerek asztalbl
add problmkat elkerltk. Jelljk a kivlasztott
lovagokat 1-essel, a nem kivlasztottakat 0-val.
Ekkor a feladat gy rhat t, hogy ksztsnk 5
darab 0-bl s 4 darab 1-bl ll kilencjegy
bitsorozatokat gy, hogy kt 1 ne legyen egyms
mellett.
Rakjuk le a 0-kat gy, hogy kztk illetve elttk s mgttk egy-egy helyet kihagyunk.
II.4.1. KIDOLGOZOTT PLDK 121
Ekkor a ngy egyest a fent jellt hat hely brmelyikre eloszthatjuk. Lehetsgek:
_
6
4
_
.
b) Ha Sir Ny nincs a csapatban, akkor a tbbi 11 lovag kzl kell tt kivlasztani gy,
hogy ne legyenek kztk szomszdosak. Az elz meggondols alapjn 5 darab egyesbl
s 6 darab nullbl ll sorozatot kell ksztennk. Ezt
_
7
5
_
fle kppen tehetjk meg.
Mivel a felsorolt kt lehetsg egymst kizr eseteket tartalmaz, ezrt az sszes lehetsgek
szma az sszeadsi szably alapjn:
_
6
4
_
+
_
7
5
_
.
4.5. Hny mrkzst jtszanak egy n csapatos krmrkzses bajnoksgban? Legalbb hny fordulban
oldhat ez meg?
Megolds:
a) Annyi mrkzsre van szksg, ahnyflekppen kivlaszthatunk az n csapat kzl 2-t,
azaz
_
n
2
_
=
n (n1)
2
.
b) Ha n pros, akkor egy fordulban legfeljebb
n
2
mrkzst lehet lejtszani. gy legalbb
n1 fordulra van szksg.
Ha n pratlan, akkor minden fordulban legfeljebb
n1
2
mrkzs lehet. gy legalbb n
fordulra van szksg.
A grfelmlet tmakrben be fogjuk lt, hogy meg lehet oldani ennyi fordulban.
4.6. Hny mrkzst jtszanak egy kiesses ping-pong versenyen ahol n jtkos indult?
Megolds:
Minden mrkzsen pontosan 1 jtkos esik ki. Mire gyztest hirdetnek n1 jtkos kiesse
szksges, gy n1 mrkzsre van szksg.
4.7. Egy n hosszsg botot n darab egysgnyi hosszsg darabra szeretnnk szttrni. Mi a
lehetsges vgrehajtsok szma, ha minden lpsben eltrnk egy 1-nl hosszabb darabot?
Megolds:
n1 helyen lehet eltrni a plct. Minden vgrehajtsnak megfeleltethetnk egyet-egyet az
1, 2, . . . , n1 szmok permutciibl. Ilyen permutcibl
P
n1
= (n1)!
darab van.
4.8. Mennyi azoknak a hatjegy szmoknak az sszege, melyeket az 1; 1; 3; 5; 5; 5 szmjegyek
ismtlses permutciiknt kapunk?
Megolds:
Az ismtlses permutcik szma P
3,2,1
6
=
6!
3!2!
= 60.
122 II.4. FEJEZET. SSZESZMLLSI FELADATOK
rjuk ezt a 60 darab hatjegy szmot egyms al, mintha az sszeadst rsban szeretnnk
elvgezni :
1 1 3 5 5 5
1 1 5 3 5 5
1 1 5 5 3 5
.
.
.
+ 5 5 5 3 1 1
Vegyk szre, hogy minden helyirtken ugyanazon szmok sszege szerepel (csak ms sorrendben
vannak felrva). Ha az ismtlses permutcik vizsglatnal mr bevlt sznezses trkkt
alkalmazzuk, akkor knnyen lthat, hogy egy-egy helyirtken minden jel (a sznekre val
tekintettel) ugyanannyiszor szerepel. gy minden helyirtken a jegyek
1
6
-a 3-as,
2
6
-a 1-es s
3
6
-a 5-s. gy az azonos helyirtken szerepl szmjegyek sszege:
S = 10 3+20 1+30 5 = 200.
Az egyesek helyn lv jegyek sszegnek valdi rtke S, a tzesek helyn lvk 10S s gy
tovbb. gy a hatjegy szmok sszege:
S (1+10+100+1000+10000+100000) = 111111 S
4.9. Igazoljuk, hogy p(1)+p(2)+. . .+p(n)<p(2n) minden n1 szmra, ahol p(n) a partcifggvny.
Megolds: Legyen k = a
1
+a
2
+. . . +a
r
egy tetszleges partcija a k szmnak (1 k n),
ahol k a
1
a
2
. . . a
r
1.
Akkor
2n = (2nk)+a
1
+a
2
+. . . +a
r
,
ahol 2nk n k a
1
a
2
. . . a
r
1 s kvetkezik, hogy ez egy partcija 2n-nek.
gy egy olyan megfeleltetst kapunk, amely a k szmok (1 k n) partciihoz a 2n szm
partciit rendeli hozz. A k szmok (1 k n) klnbz partciinak a 2n szm klnbz
partcii felelnek meg. Valban, ha k
1
,=k
2
, akkor 2nk
1
,=2nk
2
, gy a 2n szm azon partcii,
amelyeket k
1
s k
2
partcihoz rendelnk biztosan klnbzk. Ha pedig k rgztett s k kt
klnbz partcijt tekintjk, akkor 2n megfelel partciiban az els tagok egyenlk, de
azt kveten legalbb egy tag klnbz.
Teht ez egy injektv megfeleltets. Ugyanakkor nem szrjektv, mert pl. a 2n = 2n egytag
partci nem szrmaztathat ilyen mdon. Ezzel igazoltuk az adott egyenltlensget, amely
szigor minden n 1-re. Vizsgljuk rszletesen az n = 3 s n = 4 eseteket!
II.4.2. Tovbbi gyakorl feladatok
4.10. Egy vilgbajnoki selejtezben Eurpbl 5 csapat indult, zsibl 10 rsztvev van, szak-
Amerikbl, Dl-Amerikbl s Afrikbl egyarnt 7-en neveztek. A vilgbajnoki dnt 5
csapatos meznye gy ll ssze, hogy minden kontinensrl pontosan egy csapat jut be. A
kontinenseken lejtszott selejtezk utn hnyfle lehet a dnt sszelltsa?
II.4.2. TOVBBI GYAKORL FELADATOK 123
eredmny
4.11. Melyik domin kszletben van tbb elem:
a) amelynek elemein 0-tl 8-ig vannak pontok s tartalmaz dupla dominkat, vagy
b) amelynek elemein 0-tl 9-ig vannak pontok, de nem tartalmaz dupla dominkat?
eredmny
4.12. t hzasprbl, hat hajadonbl s ht agglegnybl hnyflekppen tudunk 10 embert kivlasztani
gy, hogy hzaspr ne legyen kzttk, de legyen kztk kt asszony kt ns fr hrom
agglegny s hrom hajadon?
eredmny
4.13. Az EB-selejtezn 7 csoportban 50 csapat szerepel (egy 8 csapatos s hat 7 csapatos csoport
van). A csoportokban oda-visszavgs rendszerben krmrkzseket jtszanak. (Mindenki
jtszik mindenkivel egyszer idegenben s egyszer otthon.) Hny mrkzst jtszanak sszesen?
Hnyfle lehet a dnt 16-os meznye, ha minden csoportbl az els kt helyezett jut tovbb
(a kt hzigazda nem vesz rszt a selejtezn automatikusan tagjai a dntnek).
eredmny
4.14. 2n klnbz magassg ember hnyflekppen tud kt n-hosszsg sorballni gy, hogy az
els sorban mindenki alacsonyabb legyen, a hts sorban a megfelel helyen llnl ?
megolds
4.15. Mennyi azoknak a tzjegy szmoknak az sszege, melyek a 0, 1, 2, . . . , 9 szmjegyek
ismtlsnlkli permutciiknt kaphatk. (Figyelem 0-val nem kezddhet szm!)
megolds
4.16. Mennyi azoknak a hatjegy szmoknak az sszege, melyek a 0, 1, 1, 1, 2, 2 szmjegyek
permutlsval kpezhetk?
eredmny
4.17. Jellje p
k
(n) az n szm k rszre val partciinak a szmt. Mennyi p
1
(n), p
2
(n) s p
3
(n) az
n fggvnyben?
tmutats
4.18. Igazoljuk, hogy minden n 1 szmra
n p(n) =
n

m=1
m
[n/m]

k=1
p(nkm),
n p(n) =
n

r=1
(r)p(nr),
ahol p(n) az n partciinak a szma, (r) pedig az r pozitv osztinak az sszege.
tmutats
124 II.4. FEJEZET. SSZESZMLLSI FELADATOK
II.5. fejezet
Kombinatorika a geometriban
II.5.1. Kidolgozott pldk
5.1. Adott a skon 17 ltalnos helyzet pont (vagyis semelyik hrom nem esik egy egyenesbe).
Hny egyenest hatroznak meg ezek a pontok?
Megolds:
ltalnos helyzet pontok esetn pontosan annyi egyenes illeszthet a pontokra, ahnyflekppen
a pontok kzl kivlaszthat 2, a sorrendre val tekintet nlkl, hiszen 2 pontra egy s csak
egy egyenes illeszthet s mivel a pontok kzl semelyik 3 nem esik egy egyenesbe, ezrt az
elbb generlt egyenesek pronknt klnbzek. gy a pontok ltal meghatrozott egyenesek
szma:
C
2
17
=
_
17
2
_
.
Megjegyzs: Termszetesen klasszikus sszeszmllsi problmra val hivatkozs nlkl is
megoldhat a feladat. Egy egyenest egyrtelmen meghatroz kt pontja. Az els pont 17-
flekpppen vlaszthat, a msodik az elstl klnbz pont vlasztsra 16 lehetsg
van. A szorzsi szably alapjn az sszes lehetsgek szma 17 16. De ezzel minden egyenest
pontosan ktszer szmoltunk, hiszen pldul az A s B pontok ltal meghatrozott egyenes
megegyezik a B s az A pontok ltal meghatrozottal, amit mi klnbznek szmoltunk.
gy a klnbz egyenesek szma:
17 16
2
=
_
17
2
_
.
5.2. Adott 21 pont a skon ezek kzl 10 darab biztosan egy egyenesen van. Brmely kt pontot
egyenessel sszektve maximlisan hny klnbz egyenest kaphatunk?
Megolds:
Vlasszunk 2 pontot ahnyflekppen lehet, ezek rendre meghatroznak egy-egy egyenest.
(Termszetesen lesz olyan egyenes, amit tbbszr szmoltunk, de ezeket le fogjuk vonni.)
A 2 pont kivlasztsra
_
21
2
_
lehetsgnk van. Azt az egyenest, amelyre a 10 pontunk
illeszkedik legalbb annyiszor szmtottuk bele az elz sszestsnl, ahnyflekppen ki
tudunk vlasztani kettt a tz pont kzl (
_
10
2
_
), pedig elg lett volna egyszer szmolni.
125
126 II.5. FEJEZET. KOMBINATORIKA A GEOMETRIBAN
gy az egyenesek szma nem lehet tbb
K =
_
21
2
_

_
10
2
_
+1
darabnl. Knnyen lthat, hogy megadhatk gy a pontok, hogy a rjuk illeszked egyenesek
szma pontosan ennyi legyen, gy a fenti K fels korlt felvtetik, azaz az illeszthet egyenesek
szmnak ez a maximuma.
Megjegyzs: Jelljk pirossal a garantltan egy egyenesre illeszked 10 pontot s kkkel a
tbbi pontot. Maximlis szm egyenes illeszthet a pontrendszernkre, ha brmely hrom
pont, melyek kzl legfeljebb kett piros nem illeszkedik egy egyenesre.
5.3. Hny tlja van egy n oldal konvex sokszgnek (n 4)?
1. Megolds: Minden cscsbl n3 tl indul ki. (Nem indul tl sajt magba illetve
a 2 szomszdjba, de az sszes tbbi cscsba megy.) Ha minden cscs esetn sszeszmoljuk
az onnan kiindul tlk szmt (n(n3)), akkor minden tlt pontosan ktszer szmoltunk.
gy az tlk szma
n (n3)
2
2. Megolds: Minden tl pontosan kt cscshoz tartozik. Ha kivlasztjuk a kt cscsot,
azzal az tl teht adott lesz. Ez
C
2
n
=
_
n
2
_
=
n (n1)
2
lehetsget jelentene. De gy az oldalakat is beleszmoltuk, vagyis az tlk szma:
_
n
2
_
n =
n (n1)2n
2
=
n (n3)
2
.
5.4. Hny pontban metszik egymst egy n oldal konvex sokszg tli, ha tudjuk, hogy brmely
hrom tlnak nincs kzs pontja?
Megolds: Minden metszspontnak megfeleltethet az t megad tlk vgpontjai ltal
alkotott pontngyes. Knnyen lthat, hogy minden cscsngyeshez pontosan egy metszspont
tartozik. A feladat meghatrozni, hnyflekppenvlaszthatunk n pont kzl 4-et:
_
n
4
_
=
n (n1) (n2) (n3)
24
.
5.5. Tekintsk az brn lthat 5 7-es ngyzethlt. Hny olyan tglalap rajzolhat, melynek
cscsai rcspontok, lei pedig rcsvonalak?
II.5.2. TOVBBI GYAKORL FELADATOK 127
Megolds: Knnyen lthat, hogy minden tglalapot meghatrozhatunk az tljnak kt
vgpontjval. A vgpontok vlasztsra
_
48
2
_
lehetsg van.
Az gy kivlasztott pontprok esetn a kvetkezkre kell gyelni.
A vlasztott pontok ne essenek egy rcsegyenesre.
Ezt legknnyebben gy tudjuk biztostani, ha az sszes eset kzl elhagyjuk a felttelnek
nem megfelel pontokat:
Kzs vzszintes egyenesre es pontokat
_
6
1
_

_
8
2
_
-flekppen vlaszthatunk.
Kzs fggleges egyenesre es pontokat
_
8
1
_

_
6
2
_
-flekppen vlaszthatunk.
gy nem egy rcs-egyenesre es pontpr vlasztsra
_
48
2
_

_
6
1
_

_
8
2
_

_
8
1
_

_
6
2
_
lehetsgnk
van.
A fenti mdszerrel minden tglalapot pontosan ktszer szmoltunk, hiszen mindkt
tljnak vgpontjait kivlaszthattuk.
gy az sszes tglalapok szma:
_
48
2
_

_
6
1
_

_
8
2
_

_
8
1
_

_
6
2
_
2
= 420.
II.5.2. Tovbbi gyakorl feladatok
5.6. Adott 10 ltalnos helyzet pont a skon, vagyis olyan pontok, hogy semelyik hrom nincs
egy egyenesen s semelyik ngy nincs egy krn.
a) hny klnbz egyenest hatroznak meg a pontok?
b) hny klnbz krt hatroznak meg a pontok?
megolds
5.7. Adott a skon 10 pont gy, hogy tudjuk, hogy kzlk legalbb 5 egy krn van. Legfeljebb
hny klnbz krt hatroznak meg ezek a pontok?
eredmny
5.8. A skon 9 prhuzamos egyenes n darab ugyancsak prhuzamos egyenest metsz, s ezek
mindegyike merleges az els 9-re. Az egyenesek sszesen 756 darab tglalapot hatroznak
meg. Mennyi az n szm rtke?
megolds
5.9. Tekintsk az albbi hromszghlt!
a) Hny, az ABC hromszghz hasonl hromszg rajzolhat, amelynek cscsai rcspontok?
Azaz hny olyan hromszg rajzolhat, melynek cscsai rcspontok, lei pedig rcsvonalak?
b) Hny olyan hromszg rajzolhat, melynek cscsai rcspontok?
128 II.5. FEJEZET. KOMBINATORIKA A GEOMETRIBAN
megolds
II.6. fejezet
Fibonacci-szmok
II.6.1. Kidolgozott pldk
6.1. Deniljuk az (x
n
)
n0
sorozatot a kvetkezkppen: x
n+1
=ax
n
+bx
n1
(n1) s x
0
=c, x
1
=d,
ahol a, b, c, d adott vals szmok. Ezt msodrend lland egytthats lineris rekurzinak
nevezzk.
Hatrozzuk meg a sorozat n-edik tagjt!
Megolds
Keressk a sorozat n-edik tagjt r
n
alakban. Behelyettestve:
r
n+1
= ar
n
+br
n1
, r
n1
(r
2
ar b) = 0.
Ha r gyke az r
2
arb =0 egyenletnek, akkor az elz egyenlsg teljesl minden n 1-re.
1. Tegyk fel, hogy az r
2
arb =0 egyenletnek (ezt karakterisztikus egyenletnek nevezzk)
kt klnbz vals gyke van: r
1
s r
2
. Ennek felttele, hogy a
2
+4b > 0 legyen.
Akkor az elbbiek szerint x

n
= r
n
1
s x

n
= r
n
2
kielgtik az adott rekurzit minden n 1-re,
azaz
r
n+1
1
= ar
n
1
+br
n1
1
, r
n+1
2
=ar
n
2
+br
n1
2
(n 1).
Kvetkezik, hogy ezek tetszleges lineris kombincija, azaz x
n
= c
1
r
n
1
+c
2
r
n
2
is kielgti a
rekurzit minden n 1-re. Valban,
ax
n
+bx
n1
=a(c
1
r
n
1
+c
2
r
n
2
)+b(c
1
r
n1
1
+c
2
r
n1
2
) = c
1
(ar
n
1
+br
n1
1
)+c
2
(ar
n
2
+br
n1
2
) =
=c
1
r
n+1
1
+c
2
r
n+1
2
=x
n+1
.
Az x
0
=c s x
1
=d kezdeti rtkek illesztsvel meghatrozhatk a c
1
s c
2
szmok:
x
0
=c
1
+c
2
=c, x
1
=c
1
r
1
+c
2
r
2
=d,
ahonnan
c
1
=
dcr
2
r
1
r
2
, c
2
=
cr
1
d
r
1
r
2
, (1)
kihasznlva, hogy r
1
,=r
2
.
Fordtva, lssuk be, hogy minden megolds, teht minden, a feladat feltteleit kielgt sorozat
ilyen alak:
x
n
= c
1
r
n
1
+c
2
r
n
2
(n 0), (2)
129
130 II.6. FEJEZET. FIBONACCI-SZMOK
ahol r
1
s r
2
a karakterisztikus egyenlet gykei, c
1
s c
2
pedig alkalmasan vlasztott szmok!
Legyen (x
n
)
n0
egy tetszleges megolds s deniljuk a a c
1
s c
2
szmokat gy:
c
1
+c
2
=c, c
1
r
1
+c
2
r
2
=d. (3)
Innen c
1
-re s c
2
-re ppen a fenti (1) rtkeket kapjuk. Teljes indukcival igazoljuk, hogy (2)
teljesl minden n 0-ra!
Ha n = 0, akkor x
0
=c
1
+c
2
=c, x
1
=c
1
r
1
+c
2
r
2
=d a (3) egyenlsgek szerint. Most tegyk
fel, hogy (n1)-re s n-re
x
n1
=c
1
r
n1
1
+c
2
r
n1
2
, x
n
=c
1
r
n
1
+c
2
r
n
2
.
Akkor (n+1)-re
x
n+1
=ax
n
+bx
n1
=a(c
1
r
n
1
+c
2
r
n
2
)+b(c
1
r
n1
1
+c
2
r
n1
2
) =
=c
1
r
n1
1
(ar
1
+b)+c
2
r
n1
2
(ar
2
+b) = c
1
r
n1
1
r
2
1
+c
2
r
n1
2
r
2
2
=c
1
r
n+1
1
+c
2
r
n+1
2
.
2. Tegyk fel, hogy az r
2
ar b = 0 egyenletnek kt egyenl vals gyke van: r
1
= r
2
= r.
Ennek felttele, hogy a
2
+4b = 0 legyen. Igazoljuk, hogy ekkor
x
n
=c
1
r
n
+c
2
nr
n
= (c
1
+c
2
n)r
n
(n 0),
ahol c
1
s c
2
a kezdeti rtkek illesztsvel hatrozhatk meg.
3. Mit mondhatunk abban az esetben, ha a karakterisztikus egyenletnek komplex gykei
vannak?
6.2. Alkalmazzuk az elz feladat megoldsban szerepl mdszert a Fibonacci-sorozat s a Lucas-
sorozat n-edik tagjnak meghatrozsra (lsd elmleti rsz, 14. fejezet vge)!
Megolds
Ha az elz feladatban a =b =1, c =0, d =1 akkor visszakapjuk a Fibonacci-sorozatot. Itt a
karakterisztikus egyenlet r
2
r1 =0. Ennek gykei : r
1
=
1+

5
2
, r
2
=
1

5
2
. Kvetkezik, hogy
F
n
=c
1
r
n
1
+c
2
r
n
2
s a kezdeti rtkekre: F
0
=c
1
+c
2
=0, F
1
=c
1
r
1
+c
2
r
2
=1. Innen c
1
=1/

5,
c
2
=1/

5 s a Binet-kpletet kapjuk:
F
n
=
1

5
__
1+

5
2
_
n

_
1

5
2
_
n
_
(n 0).
Tekintsk az (L
n
)
n0
, L
n+1
=L
n
+L
n1
(n1), L
0
=2, L
1
=1 Lucas-sorozatot. Itt a =b =1,
c=2, d=1. A karakterisztikus egyenlet most is r
2
r1=0, ennek gykei : r
1
=
1+

5
2
, r
2
=
1

5
2
.
Kapjuk, hogy L
n
=c
1
r
n
1
+c
2
r
n
2
s itt a kezdeti rtkekre: L
0
=c
1
+c
2
=2, L
1
=c
1
r
1
+c
2
r
2
=1.
Innen c
1
=c
2
= 1 s kvetkezik, hogy:
L
n
=
_
1+

5
2
_
n
+
_
1

5
2
_
n
(n 0).
II.6.1. KIDOLGOZOTT PLDK 131
6.3. Tekintsk jra az
x
n+1
=ax
n
+bx
n1
n 1
x
0
=c, x
1
=d
msodrend llandegytthats lineris rekurzit, ahol a, b, c, d R s a
2
+4b > 0.
a) Igazoljuk, hogy ha k R a
k
2
akb = 0 (*)
egyenlet gyke, akkor
x
n+1
k x
n
= (ak)(x
n
k x
n1
).
b) Jellje k
1
s k
2
a (*) egyenlet gykeit, melyekre a felttel alapjn k
1
, k
2
R s k
1
,= k
2
.
Igazoljuk, hogy
x
n+1
k
1
x
n
=k
n
2
(dk
1
c), s x
n+1
k
2
x
n
=k
n
1
(dk
2
c).
c) Igazoljuk, hogy a fenti jellsekkel
x
n
=
k
n
1
(dk
2
c)k
n
2
(dk
1
c)
k
1
k
2
n 0.
Megolds
a)
Bizonyts. A (*) egyenlet trendezsbl : k (ka) = k
2
ak =b k =
b
ka
.
x
n+1
k x
n
=a x
n
+b x
n1
k x
n
= (ak)
_
x
n
+
b
ak
x
n1
_
=
= (ak)
_
_
_
_
x
n

b
ak
. .
=k
x
n1
_
_
_
_
= (ak) (x
n
k x
n1
) .
b)
1. Megolds
Bizonyts. Az elz pontban bizonytottak alapjn:
x
n+1
k
1
x
n
= (ak
1
)(x
n
k
1
x
n1
) = (ak
1
)
2
(x
n1
k
1
x
n2
) = =
= (ak
1
)
n
(x
1
k
1
x
0
) = (ak
1
)
n
(dk
1
c).
A Viete-formula alapjn igaz: k
1
+k
2
= a k
2
= ak
1
, amit az elz sszefggsbe
visszarva az llts megkaphat:
x
n+1
k
1
x
n
=k
n
2
(dk
1
c).
132 II.6. FEJEZET. FIBONACCI-SZMOK
2. Megolds
Bizonyts. n-szerinti teljes indukcival :
i) n = 0 esetn az llts igaz, hiszen
x
1
k
1
x
0
=dk
1
c =k
0
2
(dk
1
c) = dk
1
c.
ii) Tegyk fel, hogy valamely n N esetn az llts teljesl, azaz
x
n
k
1
x
n1
=k
n1
2
(dk
1
c).
iii) Igazoljuk n+1-re:
x
n+1
k
1
x
n
= a x
n
+b x
n1
k
1
x
n
= (ak
1
) x
n
+b x
n1
=
= (ak
1
)
_
_
_
_
x
n
+
b
ak
1
. .
k
1
x
n1
_
_
_
_
= (ak
1
) (x
n
k
1
x
n1
) =
= (ak
1
)
. .
k
2
(k
n1
2
(dk
1
c)) = k
n
2
(dk
1
c).
A msik llts igazolsa hasonlan vgezhet.
c)
1. Megolds
Bizonyts. Az elz pont kt lltsbl kifejezve x
n+1
-et:
x
n+1
= k
1
x
n
+k
n
2
(dk
1
c)
x
n+1
= k
2
x
n
+k
n
1
(dk
2
c)
majd a kt egyenltlensget kivonjuk egymsbl :
0 = (k
1
k
2
)x
n
+k
n
2
(dk
1
c)k
n
1
(dk
2
c),
ahonnan x
n
kifejezhet:
x
n
=
k
n
1
(dk
2
c)k
n
2
(dk
1
c)
k
1
k
2
.
2. Megolds
Bizonyts. n-szerinti teljes indukcival :
i) n = 0 s n = 1 esetn az llts igaz, hiszen
n = 0 :
x
0
= c =
k
0
1
(dk
2
c)k
0
2
(dk
1
c)
k
1
k
2
=
dk
2
cd+k
1
c
k
1
k
2
=c.
n = 1 :
x
1
=d =
k
1
(dk
2
c)k
2
(dk
1
c)
k
1
k
2
=
k
1
dk
1
k
2
ck
2
d+k
1
k
2
c
k
1
k
2
=d.
II.6.1. KIDOLGOZOTT PLDK 133
ii) Tegyk fel, hogy valamely n N esetn az llts teljesl, minden N n
0
n, azaz
x
n
=
k
n
1
(dk
2
c)k
n
2
(dk
1
c)
k
1
k
2
s
x
n1
=
k
n1
1
(dk
2
c)k
n1
2
(dk
1
c)
k
1
k
2
.
iii) Igazoljuk n+1-re:
x
n+1
= ax
n
+bx
n1
= a
k
n
1
(dk
2
c)k
n
2
(dk
1
c)
k
1
k
2
+b
k
n1
1
(dk
2
c)k
n1
2
(dk
1
c)
k
1
k
2
=
=
dk
2
c
k
1
k
2
(ak
n
1
+bk
n1
1
)
dk
1
c
k
1
k
2
(ak
n
2
+bk
n1
2
) =
=
dk
2
c
k
1
k
2
k
n1
1
(ak
1
+b)
. .
k
2
1

dk
1
c
k
1
k
2
k
n1
2
(ak
2
+b)
. .
k
2
2
=
=
k
n+1
1
(dk
2
c)k
n+1
2
(dk
1
c)
k
1
k
2
.
6.4. rjuk fel a fenti sszefggs alapjn a Fibonacci-szmokra vonatkoz Binet-kpletet!
Megolds: a =b =d = 1, c = 0 :
k
2
k1 = 0 k
1,2
=
1

1+4
2
F
n
=
_
1+

5
2
_
n

_
1
1

5
2
0
_

_
1

5
2
_
n

_
1
1+

5
2
0
_
1+

5
2

1

5
2
=
_
1+

5
2
_
n

_
1

5
2
_
n

5
=
1

n
_
,
ahol =
1+

5
2
s =
1

5
2
.
6.5. Az
L
n+1
=L
n
+L
n1
, L
0
= 2, L
1
= 1
rekurzi ltal meghatrozott sorozat elemeit Lucas-szmoknak nevezzk. rjunk fel a Lucas-
szmok kiszmtsra explicit formult!
Megolds: a =b =d = 1, c = 2 :
k
2
k1 = 0 k
1,2
=
1

1+4
2
L
n
=
_
1+

5
2
_
n

_
1
1

5
2
2
_

_
1

5
2
_
n

_
1
1+

5
2
2
_
1+

5
2

1

5
2
=
=
_
1+

5
2
_
n

5
_
1

5
2
_
n
(

5)

5
=
n
+
n
.
134 II.6. FEJEZET. FIBONACCI-SZMOK
6.6. Igazoljuk, hogy a Fibonacci-szmokra (F
n
) s a Lucas-szmokra (L
n
) igazak a kvetkezk:
a) F
n
L
n
=F
2n
, n 0
b) L
n
= F
n+1
+F
n1
, n 1
c) L
n+1
+L
n1
= 5 F
n
, n 1.
Megolds:
a) F
n
L
n
=F
2n
Bizonyts.
F
n
L
n
=
1

n
_

n
+
n
_
=
1

2n

2n
_
=F
2n
b) L
n
= F
n+1
+F
n1
Bizonyts. n-szerinti teljes indukcival :
i) n = 1 esetn az llts igaz, hiszen
n = 1 :
L
1
= 1 = F
1
+F
0
= 1+0 = 1
ii) Tegyk fel, hogy valamely n N

esetn az llts teljesl, minden N

n, azaz
L
n
=F
n
+F
n

1
iii) Igazoljuk n+1-re:
L
n+1
=L
n
+L
n1
=F
n+1
+F
n1
+F
n
+F
n2
=F
n+1
+F
n
. .
F
n+2
+(F
n1
+F
n2
. .
F
n
)=F
(n+1)+1
+F
(n+1)1
.
c) L
n+1
+L
n1
= 5 F
n
1. Megolds
Bizonyts.
L
n+1
+L
n1
b)
= (F
n+2
+F
n
)+(F
n
+F
n2
)
def
= (F
n+1
+F
n
+F
n
)+(F
n
+F
n
2) =
= (F
n
+F
n1
+F
n
+F
n
)+(F
n
+F
n
2) = 4F
n
+F
n1
+F
n2
. .
F
n
= 5F
n
.
2. Megolds
Bizonyts. n-szerinti teljes indukcival :
i) n = 1 esetn az llts igaz, hiszen
L
2
+L
0
= 2+3 = 5 F
1
= 5 1 = 5.
II.6.1. KIDOLGOZOTT PLDK 135
ii) Tegyk fel, hogy valamely n N

esetn az llts teljesl, minden N

n, azaz
L
n

+1
+L
n

1
= 5 F
n

iii) Igazoljuk n+1-re:


5F
n+1
= 5 (F
n
+F
n1
) = 5F
n
+5F
n1
=L
n+1
+L
n1
+L
n
+L
n2
=
= L
n+1
+L
n
. .
L
n+2
+L
n1
+L
n2
. .
L
n
=L
(n+1)+1
+L
(n+1)1
.
6.7. a) Van 10 forintunk. Minden nap pontosan egyet vsrolunk az albbi termkekbl
perec 1Ft
cukorka 2Ft
fagylalt 2Ft.
Hatrozzuk meg, hnyflekppen klthetjk el a pnznket!
b) Az elz feladat helyett vizsgljuk a problmt ltalnosan. Jellje M
n
azt, ahnyflekppen
elklthetnk n forintot. rjuk fel az M
n
kiszmtsra szolgl formult!
Megolds: A b) feladatot fogjuk megoldani, melynek specilis eseteknt vlaszolhatunk az
a)-ban feltett krdsre.
Jellje M
k
azt ahnyflekppen elkltetnk k forintot. Vizsgljuk a lehetsgeinket az els
napon:
perecet vesznk, a maradk pnzt M
n1
-flekppen lehet elklteni,
cukrot vesznk, a maradk pnzt M
n2
-flekppen lehet elklteni,
fagylaltot vesznk, a maradk pnzt M
n2
-flekppen tudjuk elklteni.
A fentiekbl kiderl, hogy
M
n
=M
n1
+2M
n2
,
azaz a keresett rtk egy msodrend rekurzival adott sorozat n-edik eleme. Ehhez kt
kezdelemre van szksg:
M
0
=M
1
= 1.
Ahonnan a sorozat elemei szmolhatk:
M
2
= 3, M
3
= 5, M
4
= 11, M
5
= 21, M
6
= 43, M
7
= 85, M
8
= 171, M
9
= 341, M
10
= 683.
rjuk fel az explicit formult:
a = 1, b = 2, M
0
=c = 1, M
1
= d = 1
k
2
akb = 0 k
2
k2 = 0
k
1,2
=
1

1+8
2
=
k
1
= 2
k
2
= 1
gy az n-edik elem kzvetlenl szmolhat:
M
n
=
k
n
1
(dk
2
c)k
n
2
(dk
1
c)
k
1
k
2
=
2
n
(1+1)(1)
n
(12)
2(1)
=
2
n+1
(1)
n+1
3
=
1
3
_
2
n+1
(1)
n+1
_
gy
M
10
=
1
3
_
2
11
(1)
11
_
=
1
3
(2048+1) =
2049
3
= 683.
136 II.6. FEJEZET. FIBONACCI-SZMOK
II.6.2. Tovbbi gyakorl feladatok
6.8. Adjuk meg az a
n+1
= 5 a
n
2 a
n1
, a
0
= 1, a
1
= 3 rekurzv sorozat explicit alakjt!
tmutats
6.9. Adjuk meg az a
n+1
= 2 a
n
a
n1
, a
0
=2, a
1
= 3 rekurzv sorozat explicit alakjt!
tmutats
6.10. Adjuk meg az a
n+1
= 2 a
n
3 a
n1
, a
0
= 0, a
1
= 1 rekurzv sorozat explicit alakjt!
tmutats
6.11. Hnyflekppen lehet felmenni n lpcsfokon, ha egyszerre egy vagy kt fokot lphetnk?
megolds
6.12. Hnyflekppen lehet 21-es dominkkal lefedni egy 215-s tblzatot? (A domink nem
lghatnak ki a tblrl s nem fedhetik egymst.)
tmutats
6.13. Piros s kk golykbl 10 goly hosszsg lncot ksztnk. Hnyflekppen tehetjk ezt
meg, ha nem szeretnnk, hogy kk golyk kerljenek egyms mell?
tmutats
6.14. Ismt lncot fznk. Most piros, kk s srga gyngykbl. Hnyflekppen kszthetnk n
hossz lncot, ha most sem szeretnnk, hogy kk golyk kerljenek egyms mell?
tmutats
6.15. Igazoljuk a Fibonacci-szmokra vonatkoz albbi lltsokat:
a) F
1
+F
2
+ +F
n
= F
n+2
1 n 1,
b) F
1
+F
3
+ +F
2n1
= F
2n
n 1,
c) F
2
+F
4
+ +F
2n
=F
2n+1
1 n 1,
d) F
1
F
2
+F
3
+(1)
n+1
F
n
= (1)
n+1
F
n1
+1 n 1,
e) F
n+m
=F
n1
F
m
+F
n
F
m+1
n, m1,
f) F
2
1
+F
2
2
+ +F
2
n
=F
n
F
n+1
1 n 1
g) F
n+1
F
n1
F
2
n
= (1)
n
(n 1, Cassini-kplet),
h) lim
n
F
n+1
F
n
=.
megoldsok, tmutatsok
6.16. Igazoljuk, hogy az F
n
Fibonacci-szmokra:
a) n [ m akkor s csak akkor teljesl, ha F
n
[ F
m
(n, m1),
b) (F
n
, F
m
) = F
(n,m)
(n, m1), ahol (u, v) az u s v szmok legnagyobb kzs osztja.
megolds
6.17. Igaz-e, hogy az F
n
Fibonacci-szmokra ha F
n
prm, akkor n is prm?
megolds
II.7. fejezet
Catalan-szmok
II.7.1. Kidolgozott pldk
7.1. Egy kr alak asztal krl 2n szemly l. Hnyflekppen alkothatnak ezek prokat gy, hogy
az egy prban levk kezet foghassanak anlkl, hogy egy msik pr keze alatt vagy felett t
kellene nylniuk. (Az asztal felett tnylhatnak s klnbznek tekintjk az elforgatsbl
szrmaz elrendezseket.)
Megolds: Jellje x
n
a lehetsgek szmt.
Vizsgljunk meg s rajzoljunk le nhny esetet:
n = 1 esetn 1 lehetsg van (x
1
= C
1
):
n = 2 esetn a kvetkez kt prosts lehetsges (x
2
=C
2
):
Ha n = 3, akkor 5 lehetsg van (x
3
= C
3
):
137
138 II.7. FEJEZET. CATALAN-SZMOK
n = 4 esetn 14 megfelel prosts van (x
4
=C
4
). Rajzoljuk fel ezeket!
ltalnos esetben egy megfelel prt
sszektve a 2n-szg megmaradt cscsait
kt osztlyra bonthatjuk, aszerint, hogy a
berajzolt hr melyik oldaln helyezkednek el
a krben. Csak azok a prostsok lesznek
megfelelek, melyeknl nem prostunk
klnbz osztlyba tartoz pontokat. gy a
Catalan szmokhoz hasonl rekurzi rhat
fel :
x
n
=
n1

k=0
x
k
x
nk1
.
Mivel a kezd elemek is megegyeznek, ezrt
x
n
=C
n
.
7.2. Hnyflekppen juthatunk el a koordintarendszerben a (0,0) pontbl a (2n,0) pontba gy,
hogy lpseink az f =(1,1) illetve az =(1, 1) vektorok lehetnek s ne menjnk az x-tengely
al.
1. Megolds:
Az ilyen utakat Dyck utaknak nevezzk. Legyen az (2i,0) az els pont, ahol az t visszatr
az x-tengelyre. Nyilvnval, hogy a (0,0) pontbl csak f lpssel indulhattunk, mg a (2i,0)
pontba egy lpssel rkezhettnk meg. Ebbl az is lthat, hogy az (1,1) s (2i1,1) pontok
kzti t is egy Dyck-t, mghozz ppen 2(i 1) hossz. A (2i,0) s (2n,0) pontok kztt
pedig egy 2(ni) hossz Dyck-t megy:
II.7.1. KIDOLGOZOTT PLDK 139
Jelljk y
n
-nel a 2n hossz Dyck-utak szmt. A fentiek alapjn a kvetkez rekurzi rhat
fel :
y
n
=
n

i=1
y
i1
y
ni
=
n1

i=0
y
i
y
n1i
.
Mivel 0, illetve 2 hosszsg Dyck-t egyarnt 1 darab van s a fenti rekurzi megegyezik a
Catalan-szmokra felrhat rekurzival, ezrt:
y
n
= C
n
.
2. Megolds: A megolds sorn a fent bevezetett y
n
kiszmtsra direkt formult fogunk
levezetetni, amely az elz megolds alapjn alkalmas a Catalan-szmok kzvetlen kiszmtsra
is.
A (0,0) pontbl a (2n,0) pontba 2n lpssel tudunk eljutni, melyek kzl n darab f- s n
darab -tpus. gy az sszesen
_
2n
n
_
fle t rajzolhat.
Szmoljuk ssze a rossz utakat! Ezek azok, amelyek tlpik az x-tengelyt, azaz van kzs
pontjuk az y =1 egyenessel. Az els ilyen metszspont legyen P. Az OP utat tkrzzk az
y =1 egyenesre.
Vegyk az O

PQ utat. Knnyen lthat, hogy az O

Q utak klcsnsen megfeleltethetk az


OQ rossz utaknak. Az O

pont koordinti (0, 2), gy az O

Q t sorn 2n lpst tesznk,


melyek kzl n+1 darab f- s n1 darab -tpus. gy
_
2n
n+1
_
rossz t van.
140 II.7. FEJEZET. CATALAN-SZMOK
A j utak szma s gy az n-edik Catalan-szm az albbi mdon szmolhat:
y
n
=C
n
=
_
2n
n
_

_
2n
n+1
_
=
_
2n
n
_

n
n+1
_
2n
n
_
=
1
n+1

_
2n
n
_
.
Ez utbbi formult eladson kzvetlenl igazoltk.
7.3. Igazoljuk, hogy a Catalan-szmokra
C
n
=
_
2n
n
_

_
2n
n+1
_
(n 0).
Megolds:
Az elz feladat kt megoldsa alapjn nyilvnval.
7.4. Igazoljuk, hogy a Cataln szmokra fennll az
(n+2)C
n+1
= (4n+2)C
n
(n 0)
rekurzi.
Megolds:
Fel fogjuk hasznlni, az elz feladatban kapott direkt kpletet, mely szerint:
C
n
=
1
n+1

_
2n
n
_
.
gy a bizonytand llts baloldala:
(n+2)C
n+1
=(n+2)
1
(n+1)+1

_
2(n+1)
n+1
_
=(n+2)
1
(n+1)+1

_
2n+2
n+1
_
=
(2n+2)!
(n+1)! (n+1)!
Mg a jobboldal az albbi alakban rhat:
(4n+2) C
n
= (4n+2)
1
n+1
_
2n
n
_
= 2 (2n+1)
1
n+1

(2n)!
n! n!
=
= 2
(2n+1)!
(n+1)! n!

n+1
n+1
=
(2n+2)!
(n+1)! (n+1)!
.
Ahonnan az llts kvetkezik.
7.5. Egy mozipnztrnl 2n gyerek ll sorba 10Ft-os jegyekrt. Kzlk n-nl 10Ft-os van, a
tbbieknek pedig 20-asa van. A kasszban a nyitskor nem volt vltpnz. Hny olyan
sorrendje van a gyerekeknek, amikor a sor nem akad el, azaz a pnztros mindig tud visszaadni,
ha kell ?
Megolds:
Ez a feladat is visszavezethet a Dyck-utak problmjra. Jelljk f lpssel, ha valaki 10Ft-
osssal zet s -lpssel, ha 20-assal. Ekkor a pontok y-koordinti megmutatjk, az aktulisan
a kasszban lv 10 forintosok szmt. Ez azt is jelenti, hogy az t nem mehet az x-tengely
al s amikor mindenki megvette a jegyt a pnztrban csak 20 forintosok maradnak, azaz
valban Dyck-trl van sz.
II.7.1. KIDOLGOZOTT PLDK 141
7.6. 15 s 12 lny hnyflekppen mehet be egy tncterembe, ha sosem lehet bent tbb lny
mint ?
Megolds:
Ismt az elz problmakrre visszavezethet feladatrl van sz. Jelljk f lpssel, ha lp
a terembe s -lpssel, ha lny. Ekkor a pontok y-koordinti megmutatja, hogy aktulisan
mennyivel van tbb a teremben, mint lny. Ez azt is jelenti, hogy gy szeretnnk a (0,0)
pontbl a (15+12, 1512)=(27, 3) pontba eljutni, hogy kzben az t nem mehet az x-tengely
al.
Az sszes utak szma:
_
27
12
_
.
Rossz utak szma (A tkrzses mdszer alapjn) ppen annyi, ahny t megy a (0, 2)
pontbl a (27, 3) pontba. Ezek szma:
_
27
11
_
.
gy a feladatnak megfelel utak szma:
_
27
12
_

_
27
11
_
.
7.7. ltalnostsuk a feladatot n lny s n+m esetre!
Megolds:
Most a (0, 0) pont s a (2n+m, m) pont kztt kell olyan utakat felrajzolni, melyek nem
lpik t az x-tengelyt. Ezek szma a fentiek alapjn:
_
2n+m
n+m
_

_
2n+m
n+m+1
_
7.8. n hangya mszik egy szk jratban, melybl kzpen egy zskutca gazik ki. A jrat s a
mellkg olyan szkek, hogy kt hangya mr nem fr el egyms mellett, azaz nem elzhetik
meg egymst. (Aki bemegy a mellkgba, azt akrhnyan meg tudjk elzni, de a mellkgban
is egyszerre legfeljebb egy hangya tartzkodhat.) Hnyfle sorrendben jhetnek ki a hangyk?
Megolds:
Jelljk X
n
-nel azt, hogy n darab hangya hnyfle sorrendben jhet ki a jratbl. Vizsgljunk
meg nhny konkrt esetet:
Ha n = 0, akkor nyilvnvalan csak egyetlen sorrend lehetsges, mint ahogy n = 1 esetn is.
Azaz X
0
=X
1
= 1.
Ha n = 2, akkor kt lehetsg van (X
2
= 2):
1 2
2 1
n = 3 esetn 4 lehetsg (X
3
= 4):
1 2 3
1 3 2
2 1 3
2 3 1
142 II.7. FEJEZET. CATALAN-SZMOK
n = 4 : 8 lehetsg (X
4
= 8):
1 2 3 4
1 2 4 3
1 3 2 4
1 3 4 2
2 1 3 4
2 1 4 3
2 3 1 4
2 3 4 1
A fentiek alapjn meggyelhet, hogy ha a k-adik hangya az els aki betr a zskutcba,
akkor az els k 1 hangya az eredeti sorrendben jn ki, majd a k +1-edik kvetkezik, azaz
ilyenkor a sorrend els k helyn meghatrozott hangyk llnak. A maradk nk darab hangya
X
nk
sorrendben jhet ki.
gy az sszes lehetsgek szmt megkapjuk, ha az elbbi eseteket sszegezzk, mg a k befutja
az sszes lehetsges rtket. Noha az utols hangynak nincs rtelme bemenni a zskutcba,
mgis rdemes az sszegbe belevenni, mert ez azt az esetet fogja takarni, amikor a hangyk
az eredeti sorrendben jttek ki az alagtbl. gy X
n
kiszmtsra az albbi rekurzi addik:
X
n
=
n

k=1
X
nk
=
n1

k=0
X
k
X
0
= 1.
A rekurzi egyszeren feloldhat:
X
n
=
n1

k=0
X
k
=X
n1
+
n2

k=0
X
k
. .
X
n1
= 2 X
n1
= = 2
n1
X
1
= 2
n1
n 1.
II.7.2. Tovbbi gyakorl feladatok
7.9. Az origbl indulva, mindig jobbra vagy felfel lpve egyet-egyet hnyflekppen juthatunk
el a (7,11) pontba gy, hogy sosem lpnk olyan helyre, ahol y = x3?
tmutats
7.10. Hny olyan sorozat kpezhet n darab +1 s n darab 1 felhasznlsval, melyben minden
rszletsszeg 0?
tmutats
II.7.2. TOVBBI GYAKORL FELADATOK 143
7.11. Igazoljuk, hogy 2n+1 elem 1 tagokbl ll sorozatok szma, melyben a teljes sszeg 1 s
minden rszletsszeg pozitv
1
2n+1
_
2n+1
n
_
.
Mikze ezeknek a Catalan-szmokhoz?
tmutats
144 II.7. FEJEZET. CATALAN-SZMOK
II.8. fejezet
Stirling-szmok
II.8.1. Kidolgozott pldk
8.1. Adjuk meg az A =1,2,3,4,5 halmaz 3 rszhalmazra val partciit.
Megolds:
A halmazt vagy kt egyelem s egy hromelem halmaz unijra, vagy egy egyelem s kt
ktelem halmaz unijra bontjuk. Az els fajta felbontsbl
_
5
1
_

_
4
1
_

_
3
3
_
2
= 10,
mg a msodik tpusbl
_
5
1
_

_
4
2
_

_
2
2
_
2
= 15,
felbonts llthat el:
1 2 3, 4, 5
1 3 2, 4, 5
1 4 2, 3, 5
1 5 2, 3, 4
2 3 1, 4, 5
2 4 1, 3, 5
2 5 1, 3, 4
3 4 1, 2, 5
3 5 1, 2, 4
4 5 1, 2, 3
1 2, 3 4 5
1 2, 4 3 5
1 2, 5 3 4
2 1, 3 4 5
2 1, 4 3 5
2 1, 5 3 4
3 2, 1 4 5
3 2, 4 1 5
3 2, 5 1 4
4 2, 3 1 5
4 2, 1 3 5
4 2, 5 3 1
5 2, 3 4 1
5 2, 4 3 1
5 2, 1 3 4
8.2. Ksztsk el a msodfaj Stirling-szmok tblzatt s olvassuk le, hogy az elz feladat
sorn hny partcit kellett ellltani :

5
3

.
Megolds:
145
146 II.8. FEJEZET. STIRLING-SZMOK
Egy n-elem halmaz k rszhalmazra val particionlsainak szmt msodfaj Stirling-szmnak
nevezzk s S(n, k) =
_
n
k
_
-val jelljk. Ezek kiszmtsra az albbi rekurzi rhat fel :
_
n
k
_
=k
_
n1
k
_
+
_
n1
k1
_
Bizonyts. Tekintsk a H
n
=1, 2, . . . , n halmaz k-rszhalmazra val particionlsait. Kt
esetet lehet megklnbztetni :
Van olyan egyelem rszhalmaz, amely az n-edik elemet tartalmazza,
nincs ilyen rszhalmaz.
Az els esetben az egyik rszhalmaz mr rgztett, teht a maradk n 1 elemet k 1
rszhalmazra kell bontanunk, ami S(n1, k1) =
_
n1
k1
_
-flekppen tehet meg.
A msodik esetben elszr osszuk az H
n1
=1, 2, . . . , n1 halmaz elemeit k osztlyba. Erre
S(n1, k) =
_
n1
k
_
lehetsgnk van. A flretett n-edik elemet brmelyik osztlyhoz
hozzvve az eredeti halmaz egy-egy a feltteleknek megfelel osztlyozst kapjuk. gy a
H
n1
halmaz minden osztlyozshoz hozzrendelhet a H
n
halmaz k klnbz osztlyozsa.
Mivel S(1,1) = 1, a fenti rekurzi alapjn S(n, k) szmolhat. A szmts megknnytsre a
kvetkez tblzat hasznlhat:
k
1 2 3 4 5
1 1
2 1 1
n 3 1 3 1
4 1 7 6 1
5 1 15 25 10 1
A tblzat n-edik sornak k-adik eleme ppen S(n, k). A fenti rekurzi a tblzatban a
kvetkezkppen valsthat meg:
A tblzat minden eleme gy szmolhat, hogy a fltte lev elemet megszorozzuk annak
oszlopindexvel, majd hozzadjuk a tle balra elhelyezked elemet.
8.3. Adjuk meg azokat a harmadfok permutcikat, melyek k=1, illetve k=2 ciklust tartalmaznak!
Megolds:
Ha egy darab ciklust tartalmaz a permutci, akkor a ciklus hossza hrom, azaz hrom
hosszsg ciklikus permutcik szmra vagyunk kivncsiak. (Lsd kerek asztal krli ltetsek.)
Ilyen permutcibl
3!
3
= 2 darab van.
(123) (132)
Ha kt darab ciklus van a permutciban, az csak gy lehet, hogy az egyik ciklus hossza 1,
a msik pedig 2, azaz a permutcinak pontosan egy xpontja van. Most ezt gy tudjuk
II.8.1. KIDOLGOZOTT PLDK 147
megtenni, ha kivlasztjuk a xpontot. Ez mr a 2 hossz ciklust egyrtelmen meghatrozza.
Teht 3 ilyen permutci rhat fel.
(1)(23) (2)(13) (3)(12)
8.4. Ksztsk el az elsfaj Stirling-szmok tblzatt.
Megolds:
Azoknak az n-elem permutciknak a szmt, amelyek pontosan k ciklust tartalmaznak
elsfaj Stirling-szmnak nevezzk s s(n, k)=
_
n
k
_
-val jelljk. Ezek kiszmtsra az albbi
rekurzi rhat fel :
_
n
k
_
= (n1)
_
n1
k
_
+
_
n1
k1
_
Bizonyts. Tekintsk az 1, 2, . . . , n elemek permutciit. Kt esetet lehet megklnbztetni :
Az n-edik elem xpont, azaz (n) egy egyelem ciklus,
az n-edik elem nem xpont.
Az els esetben az egyik ciklus mr rgztett, teht a maradk n1 elemet k1 ciklusra kell
bontanunk, ami s(n1, k1) =
_
n1
k1
_
-flekppen tehet meg.
A msodik esetben tekintsnk az 1, 2, . . . , n1 elemek egy k ciklust tartalmaz permutcijt.
Ilyen permutci ppen s(n1, k)=
_
n1
k
_
darab van. A flretett n-edik elemet brmelyik
elem utn a permutci ciklusokra val felbontsban, ezt n1 flekppen tehetjk meg. gy
egy olyan n-edfok permutcihoz jutunk, melynek n nem xpontja. Azaz minden pontosan
k ciklust tartalmaz permutcihoz n1 darab olyan n-edfok permutci tartozik, melyben
n nem xpont.
Mivel s(1,1) = 1, a fenti rekurzi alapjn s(n, k) szmolhat. A szmts megknnytsre a
kvetkez tblzat hasznlhat:
k
1 2 3 4 5
1 1
2 1 1
n 3 2 3 1
4 6 11 6 1
5 24 50 35 10 1
A tblzat n-edik sornak k-adik eleme ppen s(n, k). A fenti rekurzi a tblzatban a
kvetkezkppen valsthat meg:
A tblzat minden eleme gy szmolhat, hogy a fltte lev elemet megszorozzuk annak
sorindexvel, majd hozzadjuk a tle balra elhelyezked elemet.
148 II.8. FEJEZET. STIRLING-SZMOK
8.5. Adjuk meg azokat a negyedfok permutcikat, melyek k=2, illetve k=3 ciklust tartalmaznak!
Megolds:
Ha kt darab ciklus van a permutciban, az gy lehet, hogy az egyik ciklus hossza 1, a msik
pedig 3, vagy mindegyik 2 elem. Szmoljuk ki, hny ilyen permutci van az els esetben
a xpont 4-flekppen vlaszthat. A maradk hrom elembl mondjuk a harmadik feladat
meggondolsai alapjn 2-flekppen kszthetnk 3 hosszsg ciklust, gy az (a)(bcd) tpus
permutcik szma 4 2 = 8. A msodik esethez tartoz permutcikat meghatrozhatjuk
azltal, hogy kivlasztjuk az els ciklusban szerepl 2 elemet (a sorrendre val tekintet nlkl),
ami binom42=6-flekppen tehet meg. Ilyenkor viszont a kt ciklus sorrendje fontos lenne (pl
(ab)(cd) s (cd)(ab)eseteket klnbznek tekintennk), azaz minden permutcit pontosan
ktszer szmoltunk. gy a pontosan kt ciklust tartalmaz 4-edfok permutcik szma 8+
+3 = 11.
(1)(234) (1)(243) (2)(134) (2)(143) (3)(124) (3)(142) (4)(123) (4)(132)
illetve
(12)(34) (13)(24) (14)(23)
II.8.2. Tovbbi gyakorl feladatok
8.6. Igazoljuk a msodfaj Stirling-szmokra vonatkoz albbi rekrzit! Ha 1 k n, akkor
_
n
k
_
=
nk+1

j=1
_
n1
j 1
__
nj
k1
_
.
megolds
8.7. Igazoljuk a Bell-szmokra vonatkoz albbi rekrzit!
Ha n, m0, akkor a 0
0
= 1 konvenci hasznlatval
B(n+m) =
m

j=0
n

k=0
j
nk
_
m
j
__
n
k
_
B(k).
megolds
8.8. Igazoljuk, hogy a msodfaj Stirling-szmokra
_
n
k
_
=

a
1
+...+a
k
=n
a
1
,...,a
k
1
n!
k! a
1
! a
k
!
.
megolds
8.9. Igazoljuk, hogy minden n k 1 esetn

x
1
+...+x
k
=n
x
1
,...,x
k
1
1
x
1
x
k
=
k!
n!
_
n
k
_
,
ahol
_
n
k

az elsfaj Stirling-szmok. megolds


8.10. Igazoljuk, hogy a B(n) Bell-szmokra B(n) <n! minden n 3-ra. tmutats
II.9. fejezet
Grfelmleti fogalmak
II.9.1. Kidolgozott pldk
9.1. Egy grf cscsai reprezentljk a termszetes szmokat 1-tl 8-ig. Kt cscs kzt fusson l,
ha a nekik megfeleltetett termszetes szmok relatv prmek. Rajzoljuk meg a grfot!
Megolds:
9.2. Van-e olyan egyszer grf, amely cscsainak fokszma rendre
a) 1, 2, 2, 3, 3, 3?
b) 1, 1, 2, 2, 3, 4, 4?
c) 8, 8, 8, 5, 5, 5, 3, 2, 2?
Megolds:
a) Igen van, pldul a kvetkez:
149
150 II.9. FEJEZET. GRFELMLETI FOGALMAK
b) Mivel a fokszmok sszege pros kellene, hogy legyen, ezrt ilyen grf nem ltezik.
c) A grfnak 9 cscsa van. Ez azt jelenti, hogy a 8-adfok cscsok foka maximlis, azaz
minden msik csccsal ssze vannak ktve. A feladatban vzolt grfnak 3 ilyen cscsa
lenne, ami azt jelenti, hogy a grf cscsai kzl a legkisebb fokszm is legalbb 3 csccsal
ssze van ktve, azaz nem lehet 2-odfok cscs.
9.3. Egy trsasgban az ismeretsgek klcsnsek. Bizonytsuk be, hogy van kt ember, akinek
ugyanannyi ismerse van. Azaz vges egyszer grfban mindig van kt pont, amelyek fokszma
megegyezik.
Megolds:
Tegyk fel, hogy mindenkinek klnbz szm ismerse van, azaz a grf minden pontjnak
klnbz a fokszma. Egy n cscs egyszer grf minden cscsnak fokszma legalbb 0,
de legfeljebb n1,azaz
0 deg(P) n1 P G.
Ez pontosan n klnbz rtket takarna, de az elz feladatban lttuk, hogy egy grfban
nem lehet egyszerre maximlis fokszm s nulladfok cscs is. Hiszen a maximlis fokszm
cscs minden msik ponttal ssze van ktve, mg a nulladfok egyikkel sincs, gy ezek a
felttelek egyszerre nem teljesthetk.
Azaz a fokszmokra egyszer grf esetn n1 klnbz lehetsges rtknk van. A grf n
cscsa kzl teht a skatulya-elv alapjn biztosan lesz legalbb kt olyan, melyek foka azonos.
9.4. Mi a helyzet, ha az elz feladat felttelei mellett megengednk tbbszrs leket?
Megolds:
Ha n3, akkor mindig konstrulhat olyan sszefgg, huroklt nem tartalmaz grf, melynek
minden pontjnak klnbz a fokszma.
Az lltst gy bizonytjuk, hogy n = 3-ra mutatunk egy ilyen grfot, majd megmutatjuk,
hogy ha valamely n N esetn ismert egy a feltteleknek megfelel elrendezs, akkor abbl
szrmaztathat egy n+1 cscs, sszefgg, huroklt nem tartalmaz grf, melyben minden
cscs klnbz fokszm.
i) n = 3 esetn a cscsok mell a fokszmukat rtuk:
II.9.1. KIDOLGOZOTT PLDK 151
ii) Tegyk fel, hogy valamely n N esetn ltezik egy a feltteleknek megfelel elrendezs.
Az ilyen grf cscsainak fokszmai a felttelek alapjn olyan halmazt alkotnak, melynek
n darab klnbz eleme van s kzllk a legkisebb is legalbb 1.
iii) Igazoljuk n+1-re.
A grfhoz vegynk hozz egy j cscsot s ezzel egytt egy olyan krt, amely az j
pontbl indul s minden cscsot pontosan egyszer rint, majd visszatr az jonan felvett
pontba. (Hamilton kr, lsd ksbb.) Ekkor a cscsok fokaira a kvetkez rhat fel :
az j pont fokszma 2,
ha valamely P pont fokszma az n-edfok grfban k = deg P volt, akkor az n+1-
edfok grfban deg P +2.
Knnyen lthat, hogy a cscsok fokszmai most is kielgtik a feltteleket, azaz n+1
darab klnbz termszetes szmot kaptunk, melyek kztt nem szerepel a 0.
Az n = 4 eset szrmaztatsa:
9.5. Egy trsasgban 5 hzaspr van jelen. Azok akik nem ismerik egymst bemutatkozsul kezet
fognak egymssal. A r megkrdezi minden jelenlevtl, hogy hny emberrel fogott kezet
s csupa klnbz szmot kap vlaszul. Hny emberrel fogott kezet A r?
Megolds:
Ksztsnk egy grfot. A cscsok reprezentljk a trsasg 10 tagjt, mg kt cscs kztt
akkor fusson l, ha a nekik megfeleltetett szemlyek kezetfogtak. Ekkor a krdsre adott
vlaszok ppen a grf megfelel cscsnak fokszmai. A hzastrsval senki nem fog kezet,
ezrt a kilenc megkrdezett csak gy mondhat csupa klnbz szmot, ha a vlaszok rendre
0, 1, 2, . . . , 8. Aki nyolcat mond, az a hzastrsn kvl mindenkivel kezett fogott, gy
nullt csak a hzastrsa mondhatott. Ezt a hzasprt a trsasgbl elhagyva, azaz a kt
152 II.9. FEJEZET. GRFELMLETI FOGALMAK
cscsot s a bellk kiindul leket a grfbl kitpve egy az elzvel analg problmhoz
jutunk. Knny ltni, hogy minden a grfban marad cscs foka pontosan eggyel cskken.
Az elzhz hasonlan belthat, hogy az eredeti grf hetedfok s elsfok pontjai ltal
reprezentlt szemlyek egy hzasprt alkotnak. s gy tovbb. . . A kvetkez sszetartoz
prokat kaptuk: (2, 6); (3, 5). Az eljrst addig fojtatjuk, amg a grfban mr csak kt
pont marad. Ezek nyilvnvalan egy hzasprt alkotnak s egyikk maga A r. A kt pont
termszetesen nincs sszektve, hiszen a nekik megfeleltetett szemlyek eredetileg is ismertk
egymst. Az eljrs minden lpsben ezen pontok fokszma pontosan eggyel cskkent. Az
utols grfhoz ngy lpsben, ngy pr elhagysval jutottunk, gy az eredeti grfba mindkt
pont negyedfok volt. Azaz A r 4 emberrel fogott kezet, ppgy, mint a felesge.
9.6. Adjuk meg, hny 3 pont egyszer grf van! Rajzoljuk fel ket, szmoljuk meg a nemizomorf
eseteket!
Megolds: Hrompont grf esetn maximlisan 3 l hzhat be. (Ennyi le van a hrom-
cscs teljes grfnak). Minden l esetben kt lehetsg kzl vlaszthatunk (vagy behzzuk,
vagy nem). gy az sszes grfok szma 2
3
. Ezen grfok termszetesen csak akkor lesznek mind
klnbzek, ha cimkzet grfokrl beszlnk. Cimkzetlen grfok esetn ezek kztt vannak
izomorfak.
Azaz a 8 grf kzl 4 darab pronknt nem izomorf vlaszthat ki.
9.7. Mutassunk pldt olyan G= (V, E
G
) s H = (V, E
H
) s grfokra, melyekre
d
G
(x) = d
H
(x) x V esetn, de G s H nem izomorfak. Megolds:
A kt grf nyilvnvalan nem izomorf, hiszen az elsfok pont kpe csak az elsfok pont
lehetne, de mg az els grfban az egyik msodfok pont a szomszdja, addig a msik grfban
a harmadfok ponthoz kapcsoldik.
II.9.1. KIDOLGOZOTT PLDK 153
9.8. Legyen G egy tetszleges egyszer grf. Igazoljuk, hogy ekkor G vagy G sszefgg!
Megolds: Az llts igazolshoz elszr a kvetkez segdttelt igazoljuk:
Segdttel : Egy G grf akkor s csak akkor nem-sszefgg, ha pontjai kt nem res
osztlyba sorolhatk gy, hogy a klnbz osztlyban lv pontok kztt nem fut a grfban
l.
Bizonyts. (A segdttel:)
Ha Gnem sszefgg, akkor tbb sszefgg komponensre bomlik. Az egyik osztlyba tartozzanak
az egyik komponens pontjai, mg a msik osztlyba rakhatjuk az sszes tbbi pontot. Knnyen
lthat, hogy ez az osztlyozs megfelel a feltteleknek.
Ha G pontjai kt osztlyba sorolhatak gy, hogy a klnbz osztlyba tartoz pontok kzt
nem fut l, akkor a grf nyilvnvalan nem sszefgg.
Ha a G grf sszefgg, akkor ksz vagyunk. Ha G nem sszefgg, akkor pontjait kt
osztlyba tudjuk sorolni gy, hogy a klnbz osztlybeli cscsok kztt nem fut l G-
ben. Legyen a kt osztly A s B. A G grf G komplementerben A minden pontja ssze van
ktve B minden pontjval. Azaz A brmely pontjbl egy len t el lehet jutni B brmely
pontjba s viszont, gy G brmely pontjbl legfeljebb kt len haladva el lehet jutni a grf
brmely pontjba.
9.9. Igazoljuk, hogy ha egy n-pont egyszer grfnak tbb mint
_
n1
2
_
le van, akkor a grf
sszefgg!
Megolds: Tekintsnk egy n-pont nem sszefgg grfot. A grf legalbb kt komponenst
tartalmaz. Beltjuk, hogy nem-sszefgg grf esetn akkor maximlis az lek szma, ha a
grf kt komponensre bomlik.
Legyen K>2 a grf komponenseinek szma. A grfban a klnbz komponensekhez tartoz
pontok kztt biztosan nem fut l. Vonjunk ssze kt komponenst oly mdon, hogy kivlasztunk
mindkettbl egy egy pontot, majd a kt pontot sszekt lt behzzuk. Ezzel egy K1
komponensbl ll grfhoz jutunk, amely legalbb eggyel tbb lt tartalmaz, mint aza grf,
amelybl kiindultunk. A komponensek szma tetszlegesen nem cskkenthet, hiszen nem-
sszefgg grf legalbb kt komponensre bomlik.
Vizsgljuk most az olyan grfokat, amelyek pontosan kt komponensre bomlanak. Kiindulsknt
tekintsk azt az esetet, amikor a kt komponens egy n1 cscs s egy egycscs rszgrf.
(Azaz egy n1 pont sszefgg grfra s egy izollt pontra.) Ekkor a grfnak maximlisan
_
n1
2
_
le lehet, hiszen ezltal az sszefgg grf-rsz sszes lehetsges lt behztuk.
Be fogjuk ltni, hogy nem-sszefgg grf esetn ennl tbb lt nem tudunk behzni. Ha
egy n-pont grf kt komponensre, egy k-pont teljes grfra s egy nk-pont teljes grfra
bomlik (k<nk), akkor ha a nagyobb cscs-szm grfrszbl egy cscsot trakunk a msik
grfrszbe, azzal a lehetsges lek szmt nem nveljk, hiszen ekkor az els grfrszben az
lek szma nk 1-gyel cskken, mg a msik komponensben k-val nvekszik. gy az lek
szma a legkedvezbb esetben is csak vltozatlanul marad.
gy kt komponens esetn az lek maximlis szma
_
n1
2
_
.
154 II.9. FEJEZET. GRFELMLETI FOGALMAK
9.10. Igazoljuk, hogy ha egy 2n-pont grf minden pontjnak foka legalbb n, akkor a grf sszefgg!
Megolds: Tegyk fel, hogy a grf nem sszefgg. Ekkor a grf legalbb kt nem res
komponensre bomlik. Tekintsnk egy pontot, mondjuk A-t. Mivel A fokszma legalbb n,
ezrt a grf cscsai kzl legalbb n-nel egy komponensbe tartozik. gy a grf egyik komponense
legalbb n +1 pontot tartalmaz. A msik komponensben legfeljebb n 1 pont tartozhat.
Ezek egyike legyen B. Mivel B fokszma legalbb n, ezrt legalbb n tovbbi pontnak
kellene a komponensben lennie, ami ellentmonds. Az ellentmonds csak abbl a feltevsbl
szrmazhat, hogy a grf nem sszefgg.
9.11. Tekintsk n darab weboldalt, melyekre igaz, hogyha a oldalon hivatkozst tallunk b oldalra,
akkor b-n is van link a-ra. Mutassuk meg, hogy ha kzlk brmely kett kztt van linkekkel
trtn (kzvetett) kapcsolat, akkor a weboldalak kztt van (n1) kzvetlen link is.
Megolds: A feladat grfelmleti tfogalmazshoz tekintsk a kvetkez grfot! A grf
cscsai reprezentljk a weboldalakat. Kt cscs kztt fusson l, ha a nekik megfeleltetett
weboldalak kztt van kzvetlen kapcsolat. Ekkor a krds gy hangzik, hogy igazoljuk, hogy
ha a grf sszefgg, akkor van legalbb n1 le.
Az lltst n-szerinti teljes indukcival igazoljuk.
i) n = 1 esetn az llts trivilisan igaz.
ii) Tegyk fel, hogy valamely n N

esetn az llts igaz.


iii) Igazoljuk n+1 pont grfra.
Indirekt tegyk fel, hogy a grf sszefgg, de legfeljebb n+12 =n1 le van. Ekkor
a grfnak biztosan van elsfok cscsa, hiszen izollt pontja nem lehet, ha pedig minden
cscsa legalbb 2-odfok lenne, akkor az lek szma legalbb
(n+1)2
2
= n+1 lenne, ami
ellentmondana a felttelnek. Hagyjuk el az elsfok cscsot s a hozztartoz lt a grfbl,
gy a grf nyilvnvalan sszefgg marad. Ekkor a grf cscsainak szma n lesz, az lek
szma is eggyel cskken, gy az indirekt felttel alapjn legfeljebb n2 l marad. De
az indukcis felttel alapjn minden n-cscs sszefgg grfban legalbb n1 l van.
Ellentmondsra jutottunk, melynek oka csak az indirekt felttel lehet.
9.12. rjuk fel az albbi grfok szomszdsgi- s illeszkedsi-mtrixt!
a)
b)
II.9.1. KIDOLGOZOTT PLDK 155
Megolds:
a) Szomszdsgi mtrix:
_

_
0 1 0 1 0
1 0 1 0 1
0 1 0 1 1
1 0 1 0 0
0 1 1 0 0
_

_
Illeszkedsi mtrix:
_

_
1 0 0 1 0 0
1 1 0 0 1 0
0 1 1 0 0 1
0 0 1 1 0 0
0 0 0 0 1 1
_

_
b) Szomszdsgi mtrix:
_

_
0 1 0 0 0 0
1 0 0 0 1 0
0 0 0 1 0 1
0 0 1 0 0 0
0 1 0 0 0 1
0 0 1 0 1 0
_

_
Illeszkedsi mtrix:
_

_
1 0 0 0 0
1 1 0 0 0
0 0 1 1 0
0 0 1 0 0
0 1 0 0 1
0 0 0 1 1
_

_
A szomszdsgi mtrix felrsakor ebben az esetben kihasznlhatjuk, hogy a grf pros. A
mtrixbl csak azokat az elemeket rjuk le, amelyek nem szksgszeren 0-k:
_

_
1 3 5
2 0 0 1
4 1 1 0
6 0 1 1
_

_
9.13. Rajzoljuk meg a 9.14.a) grf komplementert s rjuk fel a szomszdsgi mtrixot!
Megolds:
_

_
0 0 1 0 1
0 0 0 1 0
1 0 0 0 0
0 1 0 0 1
1 0 0 1 0
_

_
Vegyk szre, hogy ha a grf szomszdsgi mtrixt sszeadjuk a komplementernek szomszdsgi
mtrixval, akkor a ftl kivtelvel minden elem helyn 1-est tallunk. (A ftlban 0-k
llnak.)
156 II.9. FEJEZET. GRFELMLETI FOGALMAK
9.14. Legyen az albbi grf szomszdsgi mtrixa A. Vgezzk el az A
2
= A A mtrix-szorzst,
prbljuk megmondani a mtrix elemeinek jelentst a grfra vonatkozan, indokoljuk is.
Megolds:
A grf szomszdsgi mtrixa s annak ngyzete:
A =
_

_
0 1 1 1 0 1
1 0 1 0 0 0
1 1 0 1 0 0
1 0 1 0 1 0
0 0 0 1 0 0
1 0 0 0 0 0
_

_
A
2
=
_

_
4 1 2 1 1 0
1 2 1 2 0 1
2 1 3 1 1 1
1 2 1 3 0 1
1 0 1 0 1 0
0 1 1 1 0 1
_

_
Az A
2
mtrix i-edik sornak j-edik elemt a mtrixszorzs szablya alapjn:
(A
2
)
ij
=
6

k=1
a
ik
a
kj
Az A
2
mtrix i-edik sornak j-edik elemnek jelentse teht az, hogy hny pontosan 2
hosszsg t vezet a mtrix i-edik pontjbl a j-edik pontba, hiszen az a
ik
elem jelentse,
hogy hnyflekppen juthatunk el az i-edik pontbl a k-adik pontba egyetlen len, s az a
kj
elem jelentse hasonl a k-adik s j-edik pont vonatkozsban.
Hasonlan lehet beltni, hogy az A
k
mtrix i-edik sornak j-edik eleme az i-edik s j-edik
pont kztti pontosan k hosszsg utak szma.
9.15. Egy klubesten a k s a lnyok feljegyeztk, hny partnerrel tncoltak. Igaz-e, hogy a k
ltal felrt szmok sszege megegyezik a lnyok ltal felrt szmokval ?
Megolds: A problmt egy pros grfon tudjuk szemlltetni. A grf cscsai reprezentljk
a tnccsoport tagjait. A pontok kztt akkor s csak akkor fusson l, ha a nekik megfeleltetett
szemlyek tncoltak egymssal. A tnccsoport tagjait kt osztlyba sorolhatjuk (lnyok illetve
k) gy, hogy az azonos osztlyba tartoz pontok kztt nem fut l. Tekintsk a kapott
grf egyszerstett szomszdsgi mtrixt. A sorokat rendeljk a lnyokhoz s az oszlopokat
a khoz. Azt hogy az i-edik lny hny val tncolt az i-edik sor elemeinek sszegeknt
kapjuk. Hasonlan azt hogy a j-edik tncpartnereinek szmt a j-edik oszlop elemeinek
sszegeknt kaphatjuk. Teht a lnyok ltal felrt szmok sszege a sorok sszegeinek sszege,
mg a k ltal felrt szmok az oszlopsszegek sszege. Mindkt sszeg a mtrix minden
elemt pontosan egyszer tartalmazza, gy nyilvnvalan egyenlk.
II.9.1. KIDOLGOZOTT PLDK 157
9.16. Ksztsk el egy hatcsapatos, egyforduls bajnoksg mrkzsrendjt! A problmval mr
foglalkoztunk a II.4 fejezet 5 feladatnak megoldsa sorn. Az ott igazolt llts szerint a
mrkzsek lebonyoltshoz legalbb 5 fordulra van szksg. Mutassuk meg, hogy ennyi
elg is!
Megolds:
Grfelmleti szempontbl egy hatcscs teljes grf 5 darab, pronknt kzs l nlkli 1-
faktornak megkeresse a feladat. (Egy grf 1-faktorn a grf sszes pontjt tartalmaz
regulris elsfok rszgrfjt rtjk. Az 1-faktorokat prostsoknak is nevezzk.) Knnyen
igazolhat, hogy ezek egyestse visszaadja a teljes grfot.
Egy szablyos tszg kzppontja s cscsai legyenek a grf szgpontjai. Az albbi brn egy
prostst ltunk.
A 0 pont krli 72

-os elforgats a fenti prosts leit egy msik prosts leibe viszi.
Knnyen lthat, hogy a kt 1-faktor lidegen. Ismtelt forgatsokkal az 5 darab elsfok
faktor mindegyike elllthat:
Az albbi brn szemlltetjk, hogy a prostsok valban lidegenek s az egyestsk
visszaadja a K
6
teljes grfot:
158 II.9. FEJEZET. GRFELMLETI FOGALMAK
9.17. Mikor bonthat K
n
teljes grf lidegen 1-faktorokra s hogyan tallhat meg egy ilyen
felbonts?
Megolds:
Ha n pros s n 2, akkor, s csak akkor ltezik a K
n
teljes grf lidegen 1-faktorokra val
felbontsa.
Legyen n 2 pros szm. Egy szablyos (n 1)-szg kzppontja s cscsai legyenek a
grf szgpontjai. A kzppontot 0-val, a cscsokat mondjuk pozitv krljrsi irnyban
jelljk az 1, 2, . . . , n1 szmokkal. A cscsok szmozst rdemes modulo n1 rteni.
A konkrt pldhoz hasonlan induljunk ki az albbi prostsbl (els fordul):
0, 1, 2, n1, 3, n2, , k, nk+1, ,
n
2
,
n
2
+1
A fenti prostsbl gy szrmaztathatunk tovbbi n2 prostst, hogy az leket
2
n1
szg
egsz tbbszrseivel elforgatjuk. Az i-edik ilyen prosts lei gy kaphatk, ha a 0, 1
l helyett a 0, i lt vesszk, mg a tbbi l esetn mindkt vgpont sorszmt i 1-gyel
megnveljk. Ahol i = 1,2, . . . , n1.
9.18. Ksztsk el a mrkzsrendet n = 5 csapat esetn. Az II.4 fejezet 5 feladatban igazoltak
szerint a lebonyoltshoz legalbb 5 fordulra van szksg. Mutassuk meg, hogy ennyi fordulban
ez meg is oldhat.
Megolds:
Vezessnk be egy ktv csapatot. Nevezzk mondjuk Pihens-nek. Ezltal a feladatot pros-
csapatos bajnoksg szervezsnek problmjra vezettk vissza. gy az n=5 csapatos bajnoksg
forduli az albbi rszgrfoknak feleltethetk meg:
Az egyes fordulkban a kvetkez mrkzseket kell lejtszani ?
1. fordul 2. fordul 3. fordul 4. fordul 5. fordul
2 5 1 3 1 5 1 2 1 4
3 4 4 5 2 4 3 5 2 3
9.19. Igazoljuk, hogy a K
n
teljes grf, ha n 2, pros,
n
2
1 pronknt lidegen Hamilton-krre s
egy ezekkel kzs l nlkli 1-faktorra bomlik.
Megolds:
A K
n
teljes grf kt szomszdos egy-faktornak egyestse a grf egy Hamilton-krt adja.
Az albbi brn n = 6 esetben az F
1
s F
2
prostsok egyestettjt szemlltettk:
II.9.1. KIDOLGOZOTT PLDK 159
Knnyen lthat, hogy ez mindig gy van. A grf lei egyetlen krt alkotnak, mely minden
cscson tmegy. Elegend beltnunk, hogy ez igaz az els s a msodik prosts egyestsre,
mert az sszes tbbi eset forgatssal visszavezethet ehhez az elrendezshez. F
1
lei legyenek
kkek, mg F
2
lei pirosak. Mivel a grf minden cscsnak fokszma pros (d
i
= 2), ezrt a
grf biztosan tartalmaz krt.
Induljunk el a 2 pontbl egy piros len t a 0 pontba, majd onnan egy kk len az 1-be.
A k-adik pontbl kk l fut az nk +1-edikbe s piros az nk +3-adikba. gy az utunk a
kvetkez pontokon vezet t:
2, 0, 1, n1+33 mod (n1), n3+1=n2, n(n2)+3=5, n5+1=n4, n(n4)+3=7, n6,...
Lthat, hogy a fenti sorozat k 4 pros index elemei egy d
1
= 2 klnbsg szmtani
sorozatot alkotnak, melynek kezdeleme 3, mg az 5 pratlan index elemek egy d
2
=
2 klnbsg szmtani sorozatot alkotnak, melynek kezdeleme n-2. Az elbbi rszsorozat
elemei mindaddig pratlanok, amg az elem nem nagyobb n1-nl, az utbbi sorozat elemei
mindaddig prosak, amg az elem nem negatv. gy az elemek kztt nem fordulhat el
ismtlds addig, amg az elz kt felttel teljesl. Vizsgljuk meg, melyik elem ismtldik
elszr!
A pros index elemek sorozata elszr a k=
n2
2
index esetn lesz n1-nl nagyobb, mghozz
n+1 amit modulo n1 reduklva az ismtld elem 2, a msik sorozat elemei kztt az
els ismtlds az =
n4
2
ekkor az elem ppen 2. Az utbbi sorozatban fordul el elszr
ismtlds. Az ismtld elem a teljes sorozat m=5+2 =5+n4 =n+1-edik elem, gy az
t n klnbz elemet tartalmaz, majd a legels eleme ismtldik, gy valjban egy olyan
krrl van sz, amely minden cscson tmegy.
gy az n1 darab 1-faktort mivel n pros, kettesvel
n
2
1 darab Hamilton-krr tudjuk
egyesteni. Mivel az 1-faktorok szma pratlan, gy a Hamilton-krk berajzolsa utn egy
1-faktor mg megmarad. Ezzel az lltst igazoltuk.
9.20. Igazoljuk, hogy ha n pozitv pratlan szm, akkor a K
n
teljes-grf
n1
2
darab pronknt
lidegen Hamilton-krre bomlik.
Megolds:
Elszr vizsgljuk a problmt n = 7 konkrt esetben! A K
7
grf egy Hamilton-kre lthat
az albbi brn:
160 II.9. FEJEZET. GRFELMLETI FOGALMAK
Knnyen lthat, hogy az leket =
2
7
szggel elforgatva az elzvel lidegen Hamilton-krhz
jutunk. Ismtelt elforgatsokkal tovbbi Hamilton-krket nyerhetnk, sszesen 3 darabot.
A fentihez hasonlan kszthetjk el ltalnos esetben a kiindulsi Hamilton-krt:
0, 1, 2, n1, 3, n2, 4, . . . ,
n+3
2
,
n+1
2
, 0
Az elforgats itt azt jelenti, hogy a 0-nl nagyobb szmokhoz rendre 1-et hozzadunk, majd
modulo n1 reduklunk.
9.21. Tekintsk az albbi pros grfokat! Melyiknek van s melyiknek nincs 1-faktora?
Megolds:
Az els grfnak van 1-faktora. A kr minden msodik lt kivlasztva egy ilyen 4-l rszgrfot
kapunk.
A msodik grfban biztosan nincs 1-faktor, mert a piros pontok szm 6, mg a feketk csak
5.
A harmadik grfban sem lehet prostst ltesteni, mert pldul a a jobb als sarok 4 piros
pontjnak sszesen csak 3 fekete szomszdja van.
Megjegyzs: A harmadik grfban minden pont foka legalbb kett, de vannak harmad s
negyedfok pontok is.
Kvetkezmny: Ha egy tncteremben minden pontosan kt lnyt ismer s minden lny
pontosan kt t ismer akkor a trsasg tagjai tudnak gy tncolni, hogy mindenki olyan
partnerrel tncoljon akit ismer, de ha minden legalbb kt lnyt ismer, de vannak olyan
k akik kettnl tbb lnyt ismernek s minden lny legalbb kt t ismer, de vannak
olyan lnyok is, akik tbb t ismernek, akkor a fenti prballtst nem lehet garantlni.
Hall-Rad-Knig-Ore ttel : Egy pros grfnak pontosan akkor van elsfok faktora, ha mindkt
pontosztlya ugyanannyi elemet tartalmaz s az egyik osztly brmely k darab pontjt
kivlasztva az ezekkel szomszdos pontok szma legalbb k. (Bizonyts: Hetyei Gbor:
Kombinatorika s grfelmlet 102.o.)
9.22. A II.4 fejezet 10 feladatban trgyalt a-tpus dominkszletnk kveivel jtszunk. A kszlet
pontosan egyszer tartalmaz minden olyan domint amelyek mindkt feln 0 s 8 kztti a
pontok szma. Az emltett feladatban belttuk, hogy a kszletben 45 darab domin van.
Krds, hogy krberakhatjuk-e gy a kveket, hogy egyms mellett azonos szmok legyenek.
Megolds:
Szemlltessk gral a problmt a kvetkez mdon: A grf cscsait feleltessk meg a 0
s 8 kztti termszetes szmoknak. Fusson l a s b kztt, ha nekik megfelel szmokkal
II.9.1. KIDOLGOZOTT PLDK 161
ltezik domin. A grf egy huroklekkel kiegsztett 9-cscs teljesgrf. Ekkor a domink
krberendezse a fenti grf egy Euler-krt adja.
Mivel minden pont fokszma pros d
i
=10 s a grf sszefgg, ezrt a grfban ltezik Euler-
kr.
9.23. Az 1, 2, 3, 4, 5, 6, 7, 8, 9 szmokat el lehet-e helyezni egy krn gy, hogy brmely kt egyms
melletti szm sszege ne legyen oszthat se hrommal, se ttel, se httel ?
Megolds:
Ksztsnk egy 9-cscs grfot. A cscsai feleljenek meg a termszetes szmoknak 1 s 9
kztt. Kt cscs kztt fusson l, ha a kt cscs szerepelhet egymsmellett a krn, azaz ha
az sszegk sem 3-mal, sem 5-tel, sem 7-tel nem oszthat. gy az albbi grfhoz jutunk. A
feladatnak megfelel elrendezsekhez a grf egy-egy Hamilton-kre.
A fenti grfban ha van Hamilton-kr, akkor a msodfok pontokhoztartoz lek mindegyike
benne kell, hogy legyen a krben. (Ezeket pirossal brzoltuk az als brn.) gy a 3, 1, 7, 4, 9, 2, 6
t rsze a Hamilton-krnek. (Mr ha van ilyen a grfban). Prbljuk folytatni a krt. A 6-os
162 II.9. FEJEZET. GRFELMLETI FOGALMAK
pontnak egyetlen olyan szomszdja van, amely mg nem szerepel a fenti tban. gy a kr csak
az 5-s ponttal folytathat. Hasonl meggondolsok miatt a kvetkez elem csak a 8-as lehet.
gy vgezetl a 3-assal bezrhatjuk a krt. A jobboldali brn a kapott elrendezs lthat.
9.24. Legyen k >1 egsz szm. Ha egy vges egyszer grfban minden pont foka legalbb k, akkor
a grfban van legalbb k+1 hossz kr s legalbb k hossz t (vagyis legalbb k+1 pontot
tartalmaz kr s t).
Bizonyts: Induljunk ki a grf egy tetszleges P
1
pontjbl s alkalmazzuk a kvetkez
eljrst: ha mr eljutottunk egy P
i
ponthoz, s ennek a pontnak van mg olyan szomszdja a
grfban, amely nem szerepel a kivlasztott pontok kztt, akkor legyen ez a (vagy az egyik
ilyen) szomszdja P
i+1
. Ekkor P
1
, P
2
, . . . , P
i+1
tovbbra is t a grfban. Ez az eljrs elbb-
utbb megakad, hiszen elfogynak a grf pontjai. Tegyk fel, hogy P
m
-nl akadunk el. Ekkor
P
m
-nek minden szomszdja szerepel mr a pontok kztt. Keressk meg a legkisebb indext
kzlk, legyen ez P
s
. Ekkor P
s
, P
s+1
, P
s+2
, . . . , P
m
kr s tartalmazza P
m
-et, valamint P
m
minden szomszdjt. Mivel P
m
-nek legalbb k szomszdja van a feladat felttele szerint, ezrt
ennek a krnek legalbb k+1 pontja van. Msrszt nyilvn mk+1, teht a tallt t hossza
legalbb k.
9.25. Rajzoljuk le az n pont fagrfokat. Mennyi a szmuk? Mennyi a nem-izomorfak szma?
(n = 1,2,3,4)
Megolds:
n = 1 illetve n = 2 esetn egy-egy fa rajzolhat:
n = 3 esetn 3 cimkzett fa grf van,a cimkzetlen fk izomorfak:
II.9.1. KIDOLGOZOTT PLDK 163
n = 4 esetn 16 cimkzett fa grf van, a cimkzetlen fk izomora alapjn kt osztlyba
sorolhatk:
164 II.9. FEJEZET. GRFELMLETI FOGALMAK
9.26. Hny n-pont cimkzett csillag ltezik? (Lsd elmleti rsz, I.9.3.8.)
Megolds:
Knnyen lthat, hogy kt cimkzett csillag pontosan akkor izomorf, ha grfok (n1)-edfok
pontjaira azonos cimke kerlt. gy ppen annyi cimkzett csillag kszthet, ahnyflekppen
a kzppont kivlaszthat, ez pedig n-flekppen tehet meg.
9.27. Hny n-pont cimkzett t van? (Lsd elmleti rsz, I.9.3.8.)
Megolds: Az t els eleme n-flekppen vlaszthat, a msodik (n 1)-flekpppen s
gy tovbb, az t utols pontja mr egyrtelmen meghatrozott. gy n!-fle elrendezst
ksztettnk. Ezek izomora alapjn prba rendezhetjk, hiszen minden t izomorf a megfordtsval
s csak ezzel az ttal lesz izomorf. gy a klnbz cimkzett utak szma:
n!
2
.
9.28. Bizonytsuk be, hogy egy legalbb 2-pont fagrfban legalbb kettvel nagyobb az elsfok
pontok szma, mint a legalbb harmadfokak!
Bizonyts. jellje a fagrf pontjainak szmt n, az elsfok pontokt e s a legalbb harmadfok
pontokt h. Ekkor a msodfok pontok szma: neh.
Ekkor a grf pontjainak fokszmsszegre igaz az albbi becsls:
n

i
d
i
1 e+2 (neh)+3 h = 2ne+h.
Msrszt mivel minden l pontosan kt cscshoz tartozik, ezrt igaz, hogy a cscsok fokszmainak
sszege ppen az lek szmnak ktszerese. Tovbb, mivel fagrfrl van sz az lek szma
pontosan n1. Ezt a fenti relciba visszahelyettestve ppen az lltst kapjuk:
2 (n1) 2ne+h 2+h e.
9.29. Adjuk meg az albbi fa Prfer-kdjt!
Megolds:
A grf legkisebb elsfok pontja az 1-es, aki a 4-es cscshoz kapcsoldik. gy a kd els eleme a
4-es. Az 1-es szm elsfok cscsot (a hozztartoz llel egytt) eltvoltva a grfbl szintn
fhoz jutunk:
II.9.1. KIDOLGOZOTT PLDK 165
Az gy kapott grfban elsfokak a 3-as, 7-es, 9-es s 10 -es pontok. Most a 3-ast kell
vlasztanunk, amely aa 2-es cscshoz kapcsoldik. gy a kd kvetkez eleme a 2-es. (Az
eddigi kd: 4, 2). A vizsglt elsfok pontot kitpve a grfbl az albbi fhoz jutunk:
Most a kvetkez pontok elsfokak: 2, 7, 9, 10. A 2-es pont szomszdja a grfban az 5-s,
gy a kdba az 5-s kerl be, mg az elsfok pontot a grfbl eltvoltva az albbi ft kapjuk:
Elsfok: 7, 9, 10. A hetest tpjk ki a grfbl, a szomszdja (6) bekerl a kdba. (4, 2, 5, 6).
Az j fa:
Most kt elsfok cscs van (9, 10). A kilencest tvoltjuk el, mg a szomszdja (8) bekerl
a kdba: 4, 2, 5, 6, 8.
A tovbbiakban az aktulis fagrfok megrajzolstl eltekintnk, csak azt rjuk fel, mikor
melyik cscsot tvoltottuk el a grfbl s azt, melyik elemmel bvlt a Prfer-kd:
A 8-as cscs eltvoltsa utn a szomszdjt (4) rjuk a kdba: (4, 2, 5, 6, 8, 4).
A 4-es cscs eltvoltsa utn a szomszdjt (5) rjuk a kdba: (4, 2, 5, 6, 8, 4, 5).
166 II.9. FEJEZET. GRFELMLETI FOGALMAK
Az 5-s cscs eltvoltsa utn a szomszdjt (6) rjuk a kdba: (4, 2, 5, 6, 8, 4, 6).
Ezek utn a grf kt elsfok pontbl ll fa (slyz), teht a Prfer-kd elkszlt.
9.30. lltsuk el azt a ft, amelynek a Prfer-kdja:
1, 1, 7, 7, 8, 8.
Megolds:
1. Mivel a kd 6 elem, a grfunknak 6+2 = 8 szgpontja van.
2. Felsoroljuk egyms al a grf pontjait, a kdot s az elsfok pontokat.
1 2 3 4 5 6 7 8
1 1 7 7 8 8
2 3 4 5 6
Vegyk szre, hogy a harmadik sorba pontosan azok az elemek kerlnek, akik szerepelnek
az els sorban, de nem szerepelnek a msodikban.
3. A fenti tblzatbl kiolvashat, hogy az els elem akit letptnk a 2. sorszm pont volt
( a legkisebb index elsfok) s a szomszdja az 1. sorszm volt (Ez a kd els eleme.)
Ezt a kt pontot sszekthetjk.
Hzzuk ki a kdbl s a pontok halmazbl a felhasznlt elemeket:
1 2/ 3 4 5 6 7 8
1/ 1 7 7 8 8
3 4 5 6
4. Az j tblzatbl lthat, hogy az kvetkez elem akit letptnk a 3. sorszm pont volt
( a legkisebb index elsfok) s a szomszdja az 1. sorszm volt (Ez a megmaradt kd
els eleme.)
Ezt a kt pontot sszekthetjk.
II.9.1. KIDOLGOZOTT PLDK 167
Hzzuk ki a kdbl s a pontok halmazbl a felhasznlt elemeket:
1 2/ 3/ 4 5 6 7 8
1/ 1/ 7 7 8 8
1 4 5 6
5. A kitpett elem az 1. sorszm pont volt ( a legkisebb index elsfok) s a szomszdja
az 7. sorszm volt (Ez a megmaradt kd els eleme.)
Ezt a kt pontot sszekthetjk.
Hzzuk ki a kdbl s a pontok halmazbl a felhasznlt elemeket:
1/ 2/ 3/ 4 5 6 7 8
1/ 1/ 7 / 7 8 8
4 5 6
6. A kitpett elem az 4. sorszm pont volt ( a legkisebb index elsfok) s a szomszdja
az 7. sorszm volt (Ez a megmaradt kd els eleme.)
Ezt a kt pontot sszekthetjk.
Hzzuk ki a kdbl s a pontok halmazbl a felhasznlt elemeket:
1/ 2/ 3/ 4/ 5 6 7 8
1/ 1/ 7 / 7 / 8 8
5 6 7
7. A kitpett elem az 5. sorszm pont volt ( a legkisebb index elsfok) s a szomszdja
az 8. sorszm volt (Ez a megmaradt kd els eleme.) Ezt a kt pontot sszekthetjk.
168 II.9. FEJEZET. GRFELMLETI FOGALMAK
Hzzuk ki a kdbl s a pontok halmazbl a felhasznlt elemeket:
1/ 2/ 3/ 4/ 5 / 6 7 8
1/ 1/ 7 / 7 / 8/ 8
6 7
8. A kitpett elem az 6. sorszm pont volt ( a legkisebb index elsfok) s a szomszdja
az 8. sorszm volt (Ez a megmaradt kd els eleme.) Ezt a kt pontot sszekthetjk.
Hzzuk ki a kdbl s a pontok halmazbl a felhasznlt elemeket:
1/ 2/ 3/ 4/ 5 / 6/ 7 8
1/ 1/ 7 / 7 / 8/ 8/
7 8
9. Mr csak kt els-fok pontunk maradt (k alkotjk a slyzt) kssk ssze ket. Ezzel
ksz a fnk. Ellenrzsknt el lehet kszteni a Prfer-kdot.
9.31. Oldjuk meg az elz feladatot a msik, kzi brzolshoz alkalmasabb mdszerrel is.
Megolds:
Mivel a Prfer-kd 6 elem, ezrt a fagrfnak 8 cscsa van. Melyek a t
1
, t
2
, . . . , t
6
pontok
melyek trlsvel ehhez a kdhoz jutottunk? Ehhez a megoldsi mdhoz jelljk a fa gykert
0-val (az a pont, amelyet az elbb 8-cal jelltnk.)
T t
1
t
2
t
3
t
4
t
5
t
6
K 1 1 7 7 0 0
Vizsgljuk a kd els elemt. Mivel 2 a legkisebb termszetes szm, aki nem szerepel az als
sorban a vizsglt pozcitl jobbra s a fels sorban a vizsglt pozcitl balra, ezrt t
1
= 2.
Hasonl meggondolsok alapjn a fenti tblzat els sora kitlthet:
T 2 3 1 4 5 6
K 1 1 7 7 0 0
II.9.1. KIDOLGOZOTT PLDK 169
A tblzat els sorbl egyetlen szmtsbajhet pozitv rtk hinyzik, mghozz a hetes.
Ez azt jelenti, hogy a Prfer-kd elksztse sorn a grfrl nem tptk le a 7-es cscsot,
vagyis az eljrs vgn a 7 0 slyz maradt. Ezzel az llel indtjuk a rajzolst, majd a
fenti tblzat oszlopai a grf egy-egy lt adjk meg. Jobbrl balra haladva a tblzatban
mindg olyan lt kell behznunk, melynek egyik cscsa korbban mr szerepelt a grfban. Az
brzols teht n1 = 7 lpsben megoldhat:
9.32. t falu elhatrozza, hogy regionlis vzmvet hoznak ltre. Az egyik faluban kutat frnak,
majd a falvak kztt vizvezetkcsveket fektetnek le. Az brn lthat az elkszlt terv.
Az egyes vezetkek ptsi kltsgt is felrva kiderlt, hogy nincs elg pnz a teljes hlzat
megptsre. Els krben egy olyan vezetkrendszert szeretnnek pteni, amellyel mindenkihez
eljut a vz, de a kltsg minimlis. Melyik vezetkeket ptsk meg?
A feladatot Kruskal-algoritmusa alapjn oldjuk meg. A clunk a fenti grf egy fesztfjt
meghatrozni. Az adott grf minden lnek kltsge klnbz, gy a minimlis kltsg
lt vlasztjuk elszr. Ezutn a ki nem vlasztott lek kzl mindig a legolcsbb olyan lt
vlasztjuk, mely nem hoz ltre krt a grfban:
170 II.9. FEJEZET. GRFELMLETI FOGALMAK
Teht a minimlis kltsg fesztfa a pirosan jellt lekbl ll. Kltsge: 15.
9.33. Adjuk meg az albbi grf minimlis kltsg feszt-grfjt!
Az elz feladathoz hasonlan kiindulsknt vlasszuk a minimlis kltsg lt.
Ha a harmadik lpsnl a zlddel jellt lt vlasztannk, akkor a grfban kr alakulna ki,
ami nem megengedett. gy az eljrst a kvetkez legkisebb kltsg l vlasztsval tudjuk
folytatni :
II.9.1. KIDOLGOZOTT PLDK 171
Teht a minimlis kltsg fesztgrf kltsge: 42.
9.34. Adjuk meg az albbi grf minimlis kltsg feszt-grfjt!
Mivel a grf lei kztt vannak azonos kltsgek, ezrt a feladat megoldsa eltt a grf
leinek kltsgt ideiglenesen megvltoztatjuk. Mghozz oly mdon, hogy ha a k kltsg r
darabb len szerepel, akkor a vizsglt r darab lhez rendre az albbi kltsgeket rendeljk:
k, k+, k+2, . . . , k+(r 1)
ahol > 0 konstanst olyannak vlasztjuk, hogy az utols ilyen lre rt k +(r 1) rtk se
legyen nagyobb a legkisebb k-nl nagyobb lkltsgnl :
172 II.9. FEJEZET. GRFELMLETI FOGALMAK
Most mr pronknt klnbz lkltsgekkel van dolgunk:
A kltsg szmtsnl termszetesen az lek eredeti kltsgeit kell gyelembe venni, gy a
fenti fesztfa kltsge: 20
II.9.2. TOVBBI GYAKORL FELADATOK 173
II.9.2. Tovbbi gyakorl feladatok
9.35. Egy grf cscsai reprezentljk a termszetes szmokat 1-tl 8-ig. A cscsbl B cscsba fusson
irnytott l, ha az A-nak megfeleltetett termszetes szm osztja a B-nek megfeleltetett
termszetes szmnak. Rajzoljuk meg a grfot!
megolds
9.36. Adjuk meg, hny 4 pont egyszer grf van! Rajzoljuk fel a nemizomorf eseteket. (Ezek szma
11, lsd elmleti rsz, I.9.2.5.)
megolds
9.37. Rajzoljuk meg az albbi szomszdsgi mtrixhoz tartoz grfot!
_

_
0 1 1 0 0
1 0 0 1 1
1 0 0 0 0
0 1 0 0 1
0 1 0 1 0
_

_
megolds
9.38. Rajzoljuk meg az albbi illeszkedsi mtrixhoz tartoz grfot!
_

_
1 1 0 0 0 1
0 0 1 0 0 0
0 0 0 0 1 1
0 0 0 1 1 0
0 1 1 1 0 0
1 0 0 0 0 0
_

_
megolds
9.39. Hny kr van a teljes ngyszgben? Hny t hossz kr van a teljes tsben, hny hat hossz
kr van a teljes hatosban? Hny n hossz kr van a teljes n-esben? (Az utols krds gy is
fogalmazhat: hny Hamilton-kre van a teljes n-esnek?)
megolds
174 II.9. FEJEZET. GRFELMLETI FOGALMAK
Harmadik rsz
Megoldsok, tmutatsok, eredmnyek
175
III.1. fejezet
Permutcik, varicik, kombincik
1.31. Egy fut versenyen nyolc fut kerlt a dntbe. A dntben hny klnbz befutsi sorrend
lehetsges, ha felttelezzk, hogy nem volt holtverseny?
Megolds:
8!
vissza a feladathoz
1.32. Hny olyan tjegy szm kpezhet az 1, 2, 3, 4, 5 szmjegyekbl, amelyben az 1-e s a 3-as
jegy egyms mellett helyezkedik el s minden szmjegyet pontosan egyszer hasznlhatunk
fel ? s ha a 2-es szmjegyet 0-ra cserljk?
Megolds:
2 4! illetve 2 4! 2 3!
vissza a feladathoz
1.33. Hny tjegy pros szm kpezhet a 0, 1, 2, 3, 4 szmjegyekbl, ha minden szmjegy csak
egyszer szerepelhet?
Megolds:
4! +2 3 3!
vissza a feladathoz
1.34. Hnyflekppen lhet le 9 ember egy kilencszemlyes hossz padra?
Megolds:
9!
vissza a feladathoz
1.35. Hnyflekppen lhet le 4 n s 5 fr egy kilencszemlyes hossz padra, gy hogy azonos
nemek ne kerljenek egyms mell?
Megolds:
5! 4!
vissza a feladathoz
177
178 III.1. FEJEZET. PERMUTCIK, VARICIK, KOMBINCIK
1.36. Hnyflekppen lhet le 4 n s 5 fr egy kilencszemlyes hossz padra, gy hogy azonos
nemek ne kerljenek egyms mell, ha csak az szmt, frrl vagy nrl van-e sz?
Megolds:
1
vissza a feladathoz
1.37. Hnyflekppen lhet le 9 ember egy kerekasztal krl, ahol kilenc szk van?
Megolds:
8!
vissza a feladathoz
1.38. Hnyflekppen lhet le 9 ember egy kerekasztal krl, ahol kilenc szk van, gy hogy kzlk
kt kitntetett (A s B) egyms mell kerljn?
Megolds:
2 7!
vissza a feladathoz
1.39. Hnyflekppen lhet le 4 n s 5 fr egy kerekasztal krl, ahol kilenc szk van gy hogy
azonos nemek ne kerljenek egyms mell?
Megolds:
0
vissza a feladathoz
1.40. Hnyfle kppen olvashat ki a kvetkez brbl KOMBINATORIKA sz, ha a tblzat bal
fels sarkbl indulunk s minden lpsben csak jobbra vagy lefel haladhatunk:
K O M B I N A
O M B I N A T
M B I N A T O
B I N A T O R
I N A T O R I
N A T O R I K
A T O R I K A
Megolds:
12!
6! 6!
vissza a feladathoz
1.41. Hnyflekppen lehet eljutni az origbl a (4, 7, 3) pontba, gy, hogy csak egysgnyi hossz
jobbra, fel s elre lpsek lehetsgesek?
Megolds:
14!
4! 7! 3!
vissza a feladathoz
179
1.42. Adott n klnbz elem. Ha az elemekhez hozzvesznk kt, az utolsval megegyez elemet,
akkor az elemek sszes klnbz sorbarendezseinek szma 22-szerese lesz, mint az eredeti
n elem sorbarendezseinek a szma.
Megolds:
A feladat lltsa alapjn:
22 P
n
= P
3
n+2
22 n! =
(n+2)!
3!
132 n! = (n+2) (n+1) n!
132 = n
2
+3n+2
0 = (n10) (n+13)
n
1
= 10
n
2
=13 hamis gyk ( / N)
vissza a feladathoz
1.43. Hny permutcija van az ASZTALLAP, s a KUTYAFUTTAT szavaknak?
Megolds:
asztallap: kutyafuttat:
9!
3! 2!
12!
4! 2! 2!
vissza a feladathoz
1.44. A 0, 1, 1, 1, 2, 2, 5 szmjegyek permutlsval hny
a) pros,
b) pratlan,
c) nggyel oszthat,
d) ttel oszthat,
e) 12-vel oszthat
szm kpezhet, ha a szmok nem kezddhetnek 0-val ?
Megolds:
a) Ha a szm pros, akkor az utols jegy vagy 0, vagy 2. Ezt a kt lehetsget kezeljk is
kln.
i) 0-ra vgzd szm P
3,2
6
darab van, hiszen, ha az utols helyre rgztem a 0-t, a
maradk 6 helyre az 1, 1, 1, 2, 2, 5 jegyek brmely sorbarendezse j s ezekbl
pont ennyi van.
ii) Ha 2-esre vgzdik a szm, akkor feladat ekvivalens azzal a krdssel, hogy hny
hatjegy szmot kpezhetnk a 0, 1, 1, 1, 2, 5 jegyekbl, ha az els helyen nem llhat
0. Ha az 1-eseket megklnbztethetnek tekintenm (mondjuk beszneznnk ket),
akkor a megfelel elrendezsek szma 5 5! lenne. Ha most a szinezstl eltekintnk,
akkor azokat az eseteket amelyek az 1-esek cserjvel kaphatk, nem tudjuk egymstl
megklnbztetni. Ez azt jelenti, hogy minden esetet 3!-szor szmoltunk ssze. (Ennyi
klnbz sorrendben rhat a hrom klnbz 1-es.) gy a lehetsgek szma
55!
3!
180 III.1. FEJEZET. PERMUTCIK, VARICIK, KOMBINCIK
Ez kt egymst kizr eset, gy az sszes lehetsg az ezekhez tartoz lehetsgek sszege:
6!
3! 2!
+
5 5!
3!
=
8 5!
3!
.
b) pratlan, A pratlan szmok szma megkaphat gy, hogy az sszes htjegy szm szmbl
kivonjuk a prosak szmt. (Lsd ksbb logikai szita-formula), vagy kzvetlenl az albbi
okfejts alapjn.
A fenti szmjegyek felhasznlsval kpzett szm pontosan akkor pratlan, ha az utols
jegye 1 vagy 5.
Ha az utols jegy 1, a feladatot visszavezettk az eredetivel analg problmra, nevezetesen
arra, hogy a 0, 1, 1, 2, 2, 5 szmjegyek permutlsval hny hatjegy szm kpezhet,
ha a 0 nem llhat az els helyen.
5 5!
2! 2!
,
Ha az utols jegy 5, az alapfeladat, amire a feladatot visszavezettk, A 0, 1, 1, 1, 2, 2
szmjegyek permutlsval hny hatjegy szm kpezhet, ha a 0 nem llhat az els
helyen.
5 5!
3! 2!
,
Az sszes eset szma ezen, egymst kizr esetekhez tartoz darabszmok sszege.
c) nggyel oszthat, A fenti szmjegyek felhasznlsval kpzett szm pontosan akkor oszthat
4-gyel, ha az utols kt jegybl alkotott szm 20 vagy 12 vagy 52. Ez hrom egymst
pronknt kizr eset, alkalmazhat az sszeadsi szably. A tovbbi rszletes magyarzattl
eltekintennk s csak a vgeredmnyt kzljk:
5!
3!
..
20-ra vgzd
+
4 4!
2!
..
12-re vgzd
+
4 4!
2!
..
52-re vgzd
.
d) ttel oszthat, Az 5-tel oszthat szmok az albbi kt egymst kizr esetbe sorolhatk:
i) 5-re vgzd (
55!
3!2!
darab, mint a b) rszben.)
ii) illetve 0-ra vgzd (P
3,2
6
mint az a) rszben)
Alkalmazhat az sszeadsi szably.
e) 12-vel oszthat Egy szm pontosan akkor oszthat 12-vel, ha 3-mal s 4-gyel is oszthat.
A fenti mdon kpzett szmok jegyeinek sszege minden esetben:
0+1+1+1+2+2+5 = 12,
vagyis az sszes elllthat szm oszthat hrommal, gy minden 4-gyel oszthat oszthat
12-vel is, ezeket pedig mr a c) rszben sszeszmoltuk.
vissza a feladathoz
1.45. Hnyflekppen oszthatunk szt 25 klnbz trgyat 5 szemly kztt egyenl arnyban?
Megolds:
25!
5! 5! 5! 5! 5!
vissza a feladathoz
181
1.46. Tizenkt tanul kztt hnyflekppen lehet kiosztani t klnbz trgyat, ha egy tanul
a) legfeljebb egy trgyat kaphat?
b) tbb trgyat is kaphat?
Megolds:
Feleltessnk meg az egyes trgyaknak egy-egy "helyirtket" egy tjegy szm felrsban.
A tanulkat jelkpezzk rendre a 0,1,2,3,4,5,6,7,8,9, A, B szmjegyekkel. Ekkor minden
kiosztsnak megfeleltethet egy 12-es szmrendszerbeli tjegy szm, a megfeleltets klcsnsen
egyrtelm. (A feladat megoldsa sorn nem csak a valdi tjegy szmokat tekintjk, hanem
megengedjk, hogy a szmok 0-val, a b feladat esetben akr tbb 0-val is kezddjenek.)
a) Minden szmjegy legfeljebb egyszer hasznlhat. V
5
12
= 12 11 10 9 8
b) A szmjegyek tbbszris felhasznlhatk. V
5
12
= 12
5
vissza a feladathoz
1.47. Az 1; 2; 3; 4; 5; 6; 7; 8; 9 szmjegyekkel hny pros, tjegy szmot kpezhetnk, ha
brmely kt szomszdos jegy klnbz parits s
a) minden jegyet legfeljebb egyszer hasznlunk?
b) Ugyanaz a jegy tbbszr is felhasznlhat?
Megolds:
a) 4 5 3 4 2
b) 4
3
5
2
vissza a feladathoz
1.48. Tizes-szmrendszerbeli hatjegy szmokat kpeznk.
a) Hny ilyen szm van?
b) Hny pros illetve hny pratlan szm van?
c) Hny 4-gyel oszthat szm van?
d) Hny 5-tel oszthat szm van?
e) Hny 3-mal oszthat szm van?
Megolds:
a) 9 10
5
b) 45 10
4
pros s ugyanennyi pratlan szm van.
c)
910
5
4
mivel minden 4. szm oszthat 4-gyel, vagy ms megkzeltssel : az utols kt
szmjegy az albbi 25 ktjegy szm kzl vlaszthat
00 04 08
12 16
20 24 28
32 36
40 44 48
52 56
60 64 68
72 76
80 84 88
92 96,
az els hrom jegybl ll szm pedig az a) feladathoz hasonlan generlhat. gy sszesen
9 10
2
20 szmot tudunk ellltani.
182 III.1. FEJEZET. PERMUTCIK, VARICIK, KOMBINCIK
d)
910
5
5
mivel minden 5. szm oszthat 5-tel, vagy msknt: az utols szmjegy ktfle lehet
(0, 5), az els t jegybl ll szm pedig az a) feladathoz hasonlan generlhat. gy
sszesen 9 10
4
2 szmot tudunk ellltani.
e)
910
5
3
mivel minden 3. szm oszthat 3-mal. A msik megkzelts lnyegesen problmsabb.
Csak az alaptletet vzoljuk. Egy szm pontosan akkor oszthat hrommal, ha a szmjegyeinek
sszege is oszthat. gy ezt a tulajdonsgot csak az hatrozza meg, mely jegyek alkotjk a
szmot. rdemes ez alapjn osztlyozni a szmainkat, majd az egyes osztlyokba tartoz
szmokat kln-kln sszeszmllni.
vissza a feladathoz
1.49. Kettes-szmrendszerbeli hatjegy szmokat kpeznk.
a) Hny ilyen szm van?
b) Hny pros illetve hny pratlan szm van?
c) Hny 4-gyel oszthat szm van?
d) Hny 8-cal oszthat szm van?
Megolds:
a) 2
5
b) 2
4
pros s ugyanennyi pratlan szm. (A pros szmok mind 0-ra, a pratlanok 1-re
vgzdnek.)
c) 2
3
4-gyel oszthat szm van, hiszen az ilyen szmok utols kt jegye 0.
d) 2
2
8-cal oszthat szm van, hiszen az ilyen szmok utols hrom jegye 0.
vissza a feladathoz
1.50. Tizenhatos-szmrendszerbeli hatjegy szmokat kpeznk.
(A szmjegyek: 0; 1; 2; 3; 4; 5; 6; 7; 8; 9; A; B; C; D; E; F)
a) Hny ilyen szm van?
b) Hny pros illetve hny pratlan szm van?
c) Hny 4-gyel oszthat szm van?
d) Hny 32-vel oszthat szm van?
Megolds:
a) 15 16
5
b) 15 16
4
8 pros s ugyanennyi pratlan szm. (A pros szmok 0-ra, 2-re, 4-re, 6-ra, 8-ra,
A-ra, C-re, vagy E-re vgzdnek. Hasonlan 8 klnbz utols jegye lehet a pratlan
szmoknak is. Termszetesen mkdne az a megkzelts is, hogy az a) feladatban generl
szmok fele pros s fele pratlan.)
c) 15 16
4
4 4-gyel oszthat szm van, hiszen az ilyen szmok 0-ra, 4-re, 8-ra, vagy C-re
vgzdnek.
d) 1516
3
8 32-vel oszthat szm van, hiszen az ilyen szmok utols jegye 0, az els 5 jegybl
ll szm pedig egy pros 5-jegy szm..
vissza a feladathoz
183
1.51. Tekintsk az albbi brn lthat hinyos mtrixokot. (Tridiagonlis mtrix rtkes elemeket
tartalmaz mezi. Ha gy tetszik, akkor egy szttrtt sakktbla.) Hnyflekppen juthatunk
el a bal fels mezbl a jobb als mezbe, gy hogy minden lpsnl egy mezt jobbra, vagy
egy mezt lefel lpnk s a kijellt terletet nem hagyhatjuk el ?
ltalnostsuk a problmt s 88-as mtrix helyett vizsgljunk nn-es mtrixot!
Megolds:
Brmilyen bejrst is tekintnk, a mtrix ftljnak mezire r kell lpnnk. Kt szomszdos
diagonlis mez kztti kt lps hossz t megttelre mindig pontosan kt lehetsgnk van.
(Vagy az tl felett haladunk, jobbra-le lpsekkkel, vagy az tl alatt megynk, le-jobbra
sorrenddel.) Ezen lehetsgeket jellhetjk mondjuk F(ent) s L(ent) szimblumokkal. Ekkor
minden bejrsnak megfeleltethet (klcsnsen egyrtelm mdon) egy htjegy (ltalnos
esetben n1-jegy) karaktersorozat, melynek jegyei az F, L halmazbl valk. gy
88-as mtrix esetn 2
7
klnbz t vlaszthat,
nn-es mtrix esetn 2
n1
.
vissza a feladathoz
1.52. Az 1; 2; 3; 4; 5; 6; 7; 8; 9 elemek hny negyedosztly ismtlsnlkli kombincija
tartalmazza az 1; 2 elemeket?
Megolds:
_
7
2
_
vissza a feladathoz
1.53. Hny olyan tjegy szm van, melynek jegyei
a) nvekv sorrendben
b) nem cskken sorrendben
kvetkeznek egyms utn?
Megolds:
a)
_
9
5
_
b)
_
9+51
5
_
=
_
13
5
_
vissza a feladathoz
184 III.1. FEJEZET. PERMUTCIK, VARICIK, KOMBINCIK
1.54. Az ts, vagy a hatos lottn lehet kevesebb szelvnnyel biztosan telitallatot elrni. (Az ts
lott esetn 90 szm kzl kell tt eltallni, a hatoson 45-bl 6-ot.)
Megolds:
ts lott esetn
_
90
5
_
szelvnyre, hatos lottnl pedig
_
45
6
_
szelvnyre van szksg. Vizsgljuk
a
_
90
5
_

_
45
6
_
klnbsg eljelt:
_
90
5
_

_
45
6
_
=
90 89 88 87 86
5!

45 44 43 42 41 40
6!
=
=
1
6!
(6 90 89 88 87 8645 44 43 42 41 40) =
=
1
6!
45 44 43 (6 2 89 2 87 242 41 40) =
=
1
6!
45 44 43
. .
>0
(48 89 8742 41 40)
. .
>0
>0
Azaz az ts lott esetn tbb szelvnyt kell kitlteni.
vissza a feladathoz
1.55. Hnyfle lehet a lott-hzs eredmnye, ha tudjuk, hogy csupa egymsutni szmot hztak
ki. (A lotthzs sorn 90 szm kzl hznak ki 5-t gy, hogy a kihzott elemek sorrendje
rdektelen.)
Megolds:
Az ilyen hzsokat klcsnsen egyrtelm mdon azonosthatjuk a legkisebb kihzott szmmal.
Ez a szm tetszlegesen vlaszthat az 1,2, . . . ,86 szmok kzl. gy 86 klnbz hzs
lehetsges.
vissza a feladathoz
1.56. Hnyfle lehet a lott-hzs eredmnye, ha tudjuk, hogy kihztak kt pr egymsutni
szmot, de nem hztak hrom egyms utnit?
Megolds:
Jellje a kihzott szmokat: n
1
<n
2
<n
3
<n
4
<n
5
. Aszerint, hogy a kihzott szmok kzl
melyik a klnll hrom esetet lehet megklnbztetni (n
1
, n
3
, n
5
). Knnyen lthat, hogy
az els illetve az utols eset szimetrikus.
Ha n
1
a klnll, akkor n
1
, n
2
1 s n
4
3 hrom klnbz szmot jell az 1, 2, . . . , 86
halmazbl. Ha a
1
< a
2
< a
3
hrom szm 1, 2, . . . , 86-bl, akkor a feltteleknek megfelel
szmok az albbi mdon generlhatk:
n
1
=a
1
, n
2
= a
2
+1, n
3
= a
2
+2, n
4
=a
3
+3, n
5
=a
3
+4.
gy az ilyen hzsoknak klcsnsen egyrtelmen megfeleltethetek a C
3
86
ismtls nlkli
kombincik, azaz
_
86
3
_
a feltteleknek megfelel lotthzs lehetsges. (Ugyanennyi esetet
jelent, ha n
5
a klnll szm.)
Ha n
3
a klnll, akkor a
1
= n
1
, a
2
= n
2
2 s n
4
3 jell hrom klnbz szmot az
1, 2, . . . , 86 halmazbl. gy a felttelnek megfelel hzsok s a 86 elem halmaz 3 elem
rszhalmazai kztt most is klcsnsen egyrtelm megfeleltets ltesthet.
Az sszes lehetsgek szma teht 3
_
86
3
_
.
vissza a feladathoz
185
1.57. Egy vfolyam 50 lny s 30 hallgatja ttag kldttsget vlaszt, spedig 3 lnyt s 2
t. Hnyflekppen teheteik ezt meg? Hnyflekppen vlaszthatnak kldttsget akkor, ha
Ancsa s Berci ppen haragban vannak, ezrt nem akarnak egytt bekerlni.
Megolds:
sszesen
_
50
3
_

_
30
2
_
-fle bizottsg llthat ssze.
Ha gyelnk arra, hogy A s B ne kerljn egyszerre a bizottsgba:
_
50
3
_

_
30
2
_

_
49
2
_

_
29
1
_
vagy ms megkzeltsben
_
49
3
_

_
29
2
_
. .
egyik sem
+
_
49
2
_

_
29
2
_
. .
csak A
+
_
49
3
_

_
29
1
_
. .
csak B
vissza a feladathoz
1.58. Egy postahivatalban 10-fle kpeslapot rulnak. Hnyflekppen vsrolhatunk
a) 8 klnbz kpeslapot?
b) 8 kpeslapot?
c) 12 kpeslapot?
Megolds:
a)
_
10
8
_
b)
_
17
8
_
c)
_
21
12
_
vissza a feladathoz
1.59. Hny sztrt kell kiadnunk, hogy kzvetlenl tudjunk fordtani 10 klnbz nyelv kzl
brmelyikrl brmelyik msikra?
Megolds:
_
10
2
_
vissza a feladathoz
186 III.1. FEJEZET. PERMUTCIK, VARICIK, KOMBINCIK
III.2. fejezet
A binomilis s a polinomilis ttel
2.9. Igazoljuk, hogy
_
n
1
_
+
_
n
3
_
+
_
n
5
_
= 2
n1
.
tlet : Gondoljuk meg, hogy hnyflekppen vlaszthat ki n trgy kzl pratlan szm.
vissza a feladathoz
2.10. Igazoljuk, hogy ha 0 r n, akkor
_
n
0
__
n
r
_
+
_
n
1
__
n
r 1
_
+
_
n
2
__
n
r 2
_
+... +
_
n
r
__
n
0
_
=
_
2n
r
_
.
tlet : A Vandermonde-azonossgban legyen m=n.
vissza a feladathoz
2.11. Igazoljuk, hogy minden n 2-re
n

k=2
k(k1)
_
n
k
_
=n(n1)2
n2
.
Adjunk kombinatorikus bizonytst is.
tlet : Kombinatorikusan: gondoljuk meg, hogy hnyflekppen lehet n szemly kzl kivlasztani
egy legalbb 2 fbl ll bizottsgot, majd ennek kijellni egy elnkt s egy alelnkt.
vissza a feladathoz
2.12. Igazoljuk, hogy minden n 2-re
_
n
2
_
+2
_
n
3
_
+3
_
n
4
_
+. . . +(n1)
_
n
n
_
= (n2) 2
n1
+1.
tlet : Az igazoland llts mindkt oldalhoz adjunk 2
n
1-et! A baloldalon ezt gy tegyk
meg, hogy hozz a kvetkez trst hasznljuk:
2
n
1 =
_
n
1
_
+
_
n
2
_
+ +
_
n
n
_
.
vissza a feladathoz
187
188 III.2. FEJEZET. A BINOMILIS S A POLINOMILIS TTEL
2.13. Szmtsuk ki az
n

i=k
_
n
i
__
i
k
_
(1)
ik
sszeget, ahol 1 k n.
tlet : Jellje S az adott sszeget. Ha k = n, akkor S =
_
n
n
__
n
n
_
= 1. Legyen k < n, akkor a
trinomilis alakot hasznlva
S =
n

i=k
_
n
k
__
nk
i k
_
(1)
ik
=
_
n
k
_
n

i=k
_
nk
i k
_
(1)
ik
=
nk

j=0
_
nk
j
_
(1)
j
= 0.
Mskpp, x
n
= (x1+1)
n
=
n

i=0
_
n
i
_
(x1)
i
=
n

i=0
_
n
i
_
i

k=0
_
i
k
_
(1)
ik
x
k
, teht
x
n
=
n

k=0
_
n

i=k
_
n
i
__
i
k
_
(1)
ik
_
x
k
, s hasonltsuk ssze x egytthatit.
vissza a feladathoz
2.14. Legyen
a
n
=
1
_
n
0
_ +
1
_
n
1
_ + +
1
_
n
n
_, n 0.
Igazoljuk, hogy a
n+1
=
n+2
2(n+1)
a
n
+1, n 0, s a
n
2 mikor n .
Megolds. Itt a
0
=1/1=1, a
1
=1/1+1/1=2, a
2
=1/1+1/2+1/1=5/2=2,5, a
3
=1/1+1/3+
+1/3+1/1=8/3=2,66..., a
4
=1/1+1/4+1/6+1/4+1/1=8/3=2,66..., a
5
=1/1+1/5+1/10+
+1/10+1/5+1/1 = 13/5 = 2,6, a
6
= 151/60 = 2,51... . Minden n 0-ra a
n
=
n

k=0
k! (nk)!
n!
,
ami nem hozhat egyszerbb alakra, de
a
n+1
=
n+1

k=0
k! (n+1k)!
(n+1)!
=
1
n+1
n

k=0
k! (nk)!
n!
(n+1k)+1=a
n

1
n+1
n

k=0
k k! (nk)!
n!
+1=
=a
n

1
n+1
n

k=0
(k+11)k! (nk)!
n!
+1=a
n

1
n+1
n

k=0
(k+1)! (nk)!
n!
+
1
n+1
n

k=0
k! (nk)!
n!
+1=
= a
n
(a
n+1
1)+
1
n+1
a
n
+1 =
n+2
(n+1)
a
n
a
n+1
+2,
ahonnan megvan a rekurzi. Tovbb a
n
>2, ha n3, st a
n
>2+
2
n
, ha n4 (az
_
n
1
_
=
_
n
n1
_
=n
binomilis egytthatk gyelembevtelvel). Innen
a
n+1
a
n
=
n+2
2(n+1)
+
1
a
n
<
n+2
2(n+1)
+
n
2(n+1)
= 1, n 4,
teht az (a
n
) sorozat szigoran cskken, ha n4. Kvetkezik, hogy (a
n
) konvergens, legyen
a = lim
n
a
n
. A rekurzibl azonnali, hogy a = 2.
vissza a feladathoz
189
2.15. Szmtsuk ki a
n

i=k
_
n
i
__
i
k
_
(1)
ik
sszeget, ahol 1 k n.
Megolds:
Felhasznljuk az
_
n
k
_

_
k
m
_
=
_
n
m
_

_
nm
km
_
sszefggst, amelyet kombinatorikus meggondolsok alapjn igazolhatunk. Egy n-tag trsasgbl
egy k-tag bizottsgot s annak egy m-tag albizottsgt szeretnnk megvlasztani. A baloldali
kifejezs azon vlasztsokat rja le, melyek sorn kivlasztjuk a k-tag bizottsgot, majd a
bizottsg tagjai kzl vlasztjuk ki azokat, akik az albizottsgnak is tagjai. A msik mdszer,
hogy elszr az albizottsg m tagjt vlasztjuk, majd a maradk n m szemly kzl
kivlasztjuk azokat akik a bizottsgnak ugyan tagjai, de az albizottsgban nincsenek benne,
k km-en vannak. gy
n

i=k
_
n
i
__
i
k
_
(1)
ik
=
n

i=k
_
n
k
__
nk
i k
_
(1)
ik
=
_
n
k
_

i=k
_
nk
i k
_
(1)
ik
=
_
n
k
_

nk

j=0
_
nk
j
_
(1)
j
. .
=0
=0
vissza a feladathoz
2.16. Igazoljuk, hogy az
_
n
k
_
=
_
n+k1
k
_
ismtlses kombincikra vonatkozan:
_
n
k
_
=
_
n1
k
_
+
_
n
k1
_
, (n, k 1).
Megolds:
Algebrai megolds:
_
n1
k
_
+
_
n
k1
_
=
_
(n1)+k1
k
_
+
_
n+(k1)1
k1
_
=
_
n+k2
k
_
+
_
n+k2
k1
_
=
_
n+k1
k
_
=
_
n
k
_
Kombinatorikai megolds:
n elem kzl vlasztunk visszatevssel, a sorrendre val tekintet nlkl k darabot. Ez az
ismtlses kombinci alapfeladata, gy

n
k
_
-flekppen tehet meg. Most osztlyozzuk ezeket
a kivlasztsokat aszerint, hogy a kivasztott elemek kztt szerepelt-e egy kitntetett elem.
Ha szerepel, akkor a maradk k1 helyre

n
k1
_
-flekppen vlaszthatunk elemeket. Ha nem
szerepel a kivlasztottak kzt, akkor

n1
k
_
kivlaszts lehetsges.
vissza a feladathoz
190 III.2. FEJEZET. A BINOMILIS S A POLINOMILIS TTEL
III.3. fejezet
Szitakpletek
3.11. Adott egy X alaphalmaz s annak A
1
, A
2
, . . . , A
n
rszhalmazai. Hny elem tartozik az A
i
-k
kzl pontosan 3-ba?
Megolds:

|A
i
A
j
A
k
|
(
4
3
)

|A
i
A
j
A
k
A

|+
(
5
3
)

|A
i
A
j
A
k
A

A
m
|+(1)
n+1
(
n
3
)
|A
1
A
2
...A
n
|
vissza a feladathoz
3.12. Hnyflekppen ltethetnk le egy kerekasztal krl hrom angolt hrom francit s hrom
trkt gy, hogy hrom azonos nemzetisg ne ljn egyms mell?
Megolds:
Logikai-szitaformulval :
sszes ltetsi lehetsgek: 8!
Ha (legalbb) egy np blokkban l :
_
3
1
_
6! 3!
Ha (legalbb) kt np blokkban l :
_
3
2
_
4! 3! 3!
Ha mind a 3 csapat blokkban l : 2! 3! 3! 3!
gy a helyes ltetsek szma:
8!
_
3
1
_
6! 3! +
_
3
2
_
4! 3! 3! 2! 3! 3! 3!
vissza a feladathoz
3.13. Hny olyan rszhalmaza van az 1,2 . . . ,10 halmaznak, amely tartalmaz legalbb egy pratlan
szmot?
2
10
2
5
.
vissza a feladathoz
3.14. Hny olyan rszhalmaza van az 1,2 . . . ,10 halmaznak, amely tartalmaz legalbb hrom
pratlan szmot?
2
10
2
5
2
5

_
5
1
_
2
5

_
5
2
_
.
vissza a feladathoz
191
192 III.3. FEJEZET. SZITAKPLETEK
3.15. Hnyflekppen vlaszthat ki az augusztus hnap 5 napja gy, hogy ne legyenek kztk
egymsutni napok?
Megolds. Ha az adott hnap t tetszleges napjt kellene kivlasztani, akkor a lehetsgek
szma
_
31
5
_
. Jellje most a
1
, a
2
, a
3
, a
4
, a
5
a kivlasztand napok sorszmt, ahol a felttelnek
megfelelen 1 a
1
<a
2
1 <a
3
2 <a
4
3 <a
5
4 27. A vlasz:
_
27
5
_
.
vissza a feladathoz
3.16. Az 1,2, . . . n szmoknak hny olyan permutcija van, hogy 1,2, . . . , k nem llnak egyms
utn
i) ebben a sorrendben,
ii) valamilyen sorrendben.
Megolds.
i) Ha 1,2, . . . , k egyms utn llnak, akkor 1,2, . . . , k egy blokknak tekintend, (nk+1)!
ilyen permutci van, s a vlasz: n! (nk+1)!.
ii) Ha 1,2, . . . k egyms utn llnak valamilyen sorrendben, akkor k! (nk+1)! ilyen permutci
van, s a vlasz: n! k! (nk+1)!.
vissza a feladathoz
III.4. fejezet
sszeszmllsi feladatok
4.10. Egy vilgbajnoki selejtezben Eurpbl 5 csapat indult, zsibl 10 rsztvev van, szak-
Amerikbl, Dl-Amerikbl s Afrikbl egyarnt 7-en neveztek. A vilgbajnoki dnt 5
csapatos meznye gy ll ssze, hogy minden kontinensrl pontosan egy csapat jut be. A
kontinenseken lejtszott selejtezk utn hnyfle lehet a dnt sszelltsa?
Megolds:
5 10 7
3
.
vissza a feladathoz
4.11. Melyik domin kszletben van tbb elem:
a) amelynek elemein 0-tl 8-ig vannak pontok s tartalmaz dupla dominkat, vagy
b) amelynek elemein 0-tl 9-ig vannak pontok, de nem tartalmaz dupla dominkat?
Megolds:
Az a-tpus kszletben
_
9
2
_
+9 =
98
2
+9 =45 domin van, a b-tpusban
_
10
2
_
=
109
2
=45. Azaz
a kt kszlet ugyanannyi domint tartalmaz.
vissza a feladathoz
4.12. t hzasprbl, hat hajadonbl s ht agglegnybl hnyflekppen tudunk 10 embert kivlasztani
gy, hogy hzaspr ne legyen kzttk, de legyen kztk kt asszony kt ns fr hrom
agglegny s hrom hajadon?
Megolds:
_
5
2
_

_
3
2
_
73 63
vissza a feladathoz
4.13. Az EB-selejtezn 7 csoportban 50 csapat szerepel (egy 8 csapatos s hat 7 csapatos csoport
van). A csoportokban oda-visszavgs rendszerben krmrkzseket jtszanak. (Mindenki
jtszik mindenkivel egyszer idegenben s egyszer otthon.) Hny mrkzst jtszanak sszesen?
Hnyfle lehet a dnt 16-os meznye, ha minden csoportbl az els kt helyezett jut tovbb
(a kt hzigazda nem vesz rszt a selejtezn automatikusan tagjai a dntnek).
Megolds:
Mrkzsek szma: 2
_
8
2
_
+6 2
_
7
2
_
A dnt meznye
_
8
2
_

_
7
2
_
6
-flekppen llhat ssze.
vissza a feladathoz
193
194 III.4. FEJEZET. SSZESZMLLSI FELADATOK
4.14. 2n klnbz magassg ember hnyflekppen tud kt n-hosszsg sorballni gy, hogy az
els sorban mindenki alacsonyabb legyen, a hts sorban a megfelel helyen llnl ?
Megolds:
1. Megolds: Minden helyes sorbarendezs klcsnsen egyrtelmen megfeleltethet a
2n szemly egy prostsnak, hiszen ha a prokat kialaktjuk, akkor azzal egyrtelmen
meghatroztuk azt is, hogy egy-egy prbl ki ll az els s ki a hts sorba.
A prostsok szma:
_
2n
2
_

_
2n2
2
_
. . .
_
2
2
_
=
2n (2n1)
2

(2n2) (2n3)
2
. . .
2 1
2
=
(2n)!
2
n
2. Megolds: A 2n szemly a rendelkezsre ll 2n helyen (2n)!-flekppen helyezkedhet el,
ha nem gyelnk a megktsre. Minden, a feltteleknek megfelel elrendezs szrmaztathat
gy, hogy a fent sszeszmolt elhelyezkedsek kzl egy tetszlegest kivlasztunk s abban
a rossz sorrendben ll prokat (ahol a magasabb szemly ll ell) felcserljk. Az n-pr
mindegyikben egy j elrendezshez tartozik egy rossz elrendezs. gy minden j sorrendhez
2
n
elrendezst szmoltunk. gy a helyes elrendezsek szma:
(2n)!
2
n
.
vissza a feladathoz
4.15. Mennyi azoknak a tzjegy szmoknak az sszege, melyek a 0, 1, 2, . . . , 9 szmjegyek
ismtlsnlkli permutciiknt kaphatk. (Figyelem 0-val nem kezddhet szm!)
Megolds:
Elszr vegyk az sszes ismtlsnlkli permutci sszegt. (Ne foglalkozzunk azzal a
felttellel, hogy az els helyen nem llhat 0.) Ekkor a kvetkez meggyelseket tehetjk:
1) A permutcik szma (belertve a 0-val kezddket is): 10!
2) Az egyes helyirtkeknl kpezve az sszeget ugyanazokat a szmokat kell sszeadni,
csak ms sorrendben. Kvetkezskppen Az egyesek a tizesek stb alakirtknek sszege
megegyezik.
3) Az gy alkotott tzjegy szmokat egymsfl rva az egyesek helyn minden szmjegy
ugyanannyiszor szerepel. (Termszetesen hasonl megllapts tehet az sszes helyirtkre.)
Vagyis minden szmjegy minden helyirtken
10!
10
= 9! alkalommal fordul el.
A fentiek alapjn az egyesek sszege:
S = 9! (0+1+2+3+4+5+6+7+8+9) = 9!
9 10
2
= 45 9!
Mivel minden helyirtken az alakirtkek sszege S az rai indoklssal analg mdon az
sszeg:
= S (1+10+100+ +1000000000) = 1111111111 9! 45.
Ebben az sszegben azonban bennevannak azok a nemkvnt szmok is amelyek 0-val kezddnek.
Ezek sszegt az elzekhez hasonlan szmolhatjuk ki. Mivel a legnagyobb helyirtken 0
ll, ezek valjban az 1, 2, . . . , 9 szmjegyekbl alkotott kilencjegy szmok.
Az sszegk a fentiekhez hasonlan
= (8! 45) 111111111
195
A krdses sszeg pedig
= 1111111111 9! 45(8! 45) 111111111 = 8! 45 (9999999999111111111) =
= 8! 45 9888888888.
vissza a feladathoz
4.16. Mennyi azoknak a hatjegy szmoknak az sszege, melyek a 0, 1, 1, 1, 2, 2 szmjegyek
permutlsval kpezhetk?
Megolds:
70 11111114 11111.
vissza a feladathoz
4.17. Jellje p
k
(n) az n szm k rszre val partciinak a szmt. Mennyi p
1
(n), p
2
(n) s p
3
(n) az
n fggvnyben?
tmutats: A denci alapjn azonnali, hogy p
1
(n) = 1 minden n 1 estn.
Legyen most n =a+b, ahol a b 1. Hnyflekppen vlaszthat meg a s b? Vizsgljuk az
n pros s n pratlan eseteket! Vlasz: p
2
(n) = [n/2] minden n 1-re.
p
3
(n)-re mr nehezebb kpletet adni. Hasznlva a p
k
(n) =p
k
(nk)+p
k1
(n1) (n k 2)
rekurzit, lsd 6.2. Ttel, igazoljuk, hogy
p
3
(n) = p
3
(n3)+p
2
(n3)+1 (n 3).
Vizsgljuk az n=6q+r, 0r 5 eseteket. Mutassuk meg, hogy ha n=6q alak (q 1), akkor
p
3
(n) =n
2
/12, ha n =6q+1 alak, akkor p
3
(n) =(n
2
1)/12. Mennyi p
3
(n) a tbbi esetben?
vissza a feladathoz
4.18. Igazoljuk, hogy minden n 1 szmra
n p(n) =
n

m=1
m
[n/m]

k=1
p(nkm),
n p(n) =
n

r=1
(r)p(nr),
ahol p(n) az n partciinak a szma, (r) pedig az r pozitv osztinak az sszege.
tmutats: Tekintsk az n sszes n = a
1
+a
2
+. . . +a
s
partcijt s adjuk ssze ezeket az
egyenlsgeket. Akkor a bal oldal ppen np(n). A jobb oldalon csoportostsunk a fellp m
(1 mn) tagokok szerint. Ha m f(m)-szer fordul el, akkor kapjuk, hogy
n p(n) =
n

m=1
mf(m).
Mennyi az f(m)?
A msodik kplet levezetshez rendezzk t az els kplet jobb oldalnak tagjait a km = r
rtkek szerint csoportostva.
vissza a feladathoz
196 III.4. FEJEZET. SSZESZMLLSI FELADATOK
III.5. fejezet
Kombinatorika a geometriban
5.6. Adott 10 ltalnos helyzet pont a skon, vagyis olyan pontok, hogy semelyik hrom nincs
egy egyenesen s semelyik ngy nincs egy krn.
a) hny klnbz egyenest hatroznak meg a pontok?
b) hny klnbz krt hatroznak meg a pontok?
Megolds:
a) Minden egyenest egyrtelmen meghatroz kt pontja, mivel semelyik hrom pont nem
esik egy egyenesre, ezrt pontosan annyi egyenest hatroznak meg a pontok, ahnyflekppen
kivlaszthatunk kzlk kettt a sorrendre val tekintet nlkl :
_
10
2
_
b) Minden krt egyrtelmen meghatroz hrom pontja, mivel semelyik ngy pont nincs egy
krn, ezrt pontosan annyi krt hatroznak meg a pontok, ahnyflekppen kivlaszthatunk
kzlk hrmat a sorrendre val tekintet nlkl :
_
10
3
_
vissza a feladathoz
5.7. Adott a skon 10 pont gy, hogy tudjuk, hogy kzlk legalbb 5 egy krn van. Legfeljebb
hny klnbz krt hatroznak meg ezek a pontok?
Megolds:
_
10
3
_

_
5
3
_
+1.
vissza a feladathoz
5.8. A skon 9 prhuzamos egyenes n darab ugyancsak prhuzamos egyenest metsz, s ezek
mindegyike merleges az els 9-re. Az egyenesek sszesen 756 darab tglalapot hatroznak
meg. Mennyi az n szm rtke?
Megolds:
Nevezzk a megadott 9 egyenesnket fgglegeseknek s a rjuk merleges n darab egyenest
vzszintesnek. (Fggetlenl a valsgos irnyuktl.) Minden tglalapot klcsnsen egyrtelmen
meghatroz a ngy oldala. Ezek kzl kettt a 9 fggleges egyenes kzl kell vlasztanunk,
mghozz a sorrendre val tekintet nlkl, a msik kettt pedig az n darab vzszintes egyenes
kzl vlasztunk. gy sszesen
_
9
2
_

_
n
2
_
tglalap generlhat. A felttellel sszevetve az albbi
egyenlethez jutunk:
756 =
_
9
2
_

_
n
2
_
0 = n
2
n42.
Ahonnan n = 7 vagy n =6. Ez utbbi nyilvnvalan hamisgyk.
vissza a feladathoz
197
198 III.5. FEJEZET. KOMBINATORIKA A GEOMETRIBAN
5.9. Tekintsk az albbi hromszghlt!
a) Hny, az ABC hromszghz hasonl hromszg rajzolhat, amelynek cscsai rcspontok?
Azaz hny olyan hromszg rajzolhat, melynek cscsai rcspontok, lei pedig rcsvonalak?
b) Hny olyan hromszg rajzolhat, melynek cscsai rcspontok?
Megolds:
a) Minden, a feltteleknek megfelel hromszgnek van egy vzszintes le. Az alapjn, hogy
a harmadik cscs a vzszintes l alatt vagy fltt helyezkedik el, klnbztessnk meg fel-
s le-tpus hromszgeket. Ezeket szmoljuk kln.
fel-tpus hromszgek:
Vgezznk tovbbi eset-sztvlasztst a hromszgek oldalhossza alapjn:
Egysg-l hromszgek:
A hromszget egyrtelmen meghatrozza a harmadik (a vzszintes l fltti)
cscs vlasztsa. Ez a pirossal jellt cscsok kzl tetszs szerint vlaszthat,
azaz 15 ilyen hromszg rajzolhat.
kt egysg lhosszsg hromszgek:
A hromszget most is egyrtelmen meghatrozza a harmadik cscs vlasztsa.
Ez a pirossal jellt cscsok kzl tetszs szerint vlaszthat, azaz 10 ilyen hromszg
rajzolhat.
199
hrom egysg lhosszsg hromszgbl a fentiekhez hasonl megondolsok alapjn
6 darab van.
ngy egysg lhosszsg hromszgbl 3 darab van.
t egysg lhosszsg hromszgbl egyetlen darab rajzolhat.
le-tpus hromszgek:
Itt is az lhossz alapjn osztlyozunk:
Egysg-l hromszgek:
A hromszget most is egyrtelmen meghatrozza a harmadik (a vzszintes l
alatti) cscs vlasztsa. Ez a pirossal jellt cscsok kzl tetszs szerint vlaszthat,
azaz 6 ilyen hromszg rajzolhat.
kt egysg lhosszsg hromszgek:
A hromszget harmadik cscsa a pirossal jellt cscsok kzl vlaszthat, azaz
3 ilyen hromszg van.
nagyobb lhosszsg hromszg nem fr el a rcson.
Azaz sszesen 35 fel-tpus s 13 le-tpus hromszg rajzolhat. Ez 48 az ABC-
hez hasonl hromszget jelent.
b) A hromszget egyrtelmen meghatrozza a 3 cscspont kivlasztsa, amely
_
21
3
_
lehetsget
jelentene. Vigyznunk kell azonban, hogy a cscsok ne essenek egy egyenesre.
200 III.5. FEJEZET. KOMBINATORIKA A GEOMETRIBAN
Szmoljuk ssze teht a hibs vlasztsokat. Ehhez tekintsk az albbi brt:
Mindhrom pont a piros egyenesre esik
_
6
3
_
esetben, a zldre
_
5
3
_
, a feketre
_
4
3
_
esetben. A
srga a szrke s a pink egyenesek esetn egy-egy kivlaszts van. Vegyk szre, hogy az
brt 120
o
-kal a hromszg slypontja krl akr pozitv, akr negatv irnyba forgatva, a
rcspontok fedsbe kerlnek, de az elbb felrajtolt problms egyenesek jabb egyenesekbe
mennek t.
gy a hibs kivlasztsok szma 3
__
6
3
_
+
_
5
3
_
+
_
4
3
_
+3
_
A valdi hromszgek szma teht
_
21
3
_
3
__
6
3
_
+
_
5
3
_
+
_
4
3
_
+3
_
= 1219.
vissza a feladathoz
III.6. fejezet
Fibonacci-szmok
6.8. Alkalmazzuk a 6.1-es feladatban levezetett ltalnos kpletet!
vissza a feladathoz
6.9. Alkalmazzuk a 6.1-es feladatban levezetett ltalnos kpletet!
vissza a feladathoz
6.10. Alkalmazzuk a 6.1-es feladatban levezetett ltalnos kpletet!
vissza a feladathoz
6.11. Jellje S
n
a lehetsgek szmt n lpcsfok vgigjrsra. Aszerint, hogy az els lpsben
1 vagy kt lpcsfokot lpnk a tovbbi lehetsgek szma S
n1
illetve S
n2
. gy az albbi
msodfaj rekrzi rhat fel :
S
n
=S
n1
+S
n2
.
A kezdelemek egyszer leszmllssal meghatrozhatk: S
0
= 1 s S
1
= 1. gy (S
n
, n N)
pp a Fibonacci szmok sorozata.
vissza a feladathoz
6.12. tlet: tekintsk az albbi tblt:
Az els domin elhelyezsre kt lehetsgnk van.
Ha az els domint lltva helyezzk el, a tovbbiakban egy 2x14-es tblt kell lefednnk:
201
202 III.6. FEJEZET. FIBONACCI-SZMOK
A msik lehetsg, hogy az els domint fektetve helyezzk el. A msodikat ekkor csak
prhuzamosan fl helyezhetjk, s ezutn a maradk 2x13-as tblt kell lefednnk.
gy a Fibonacci-szmokhoz hasonl msodrend rekrzi rhat.
vissza a feladathoz
6.13. Az elzekhez hasonlan rjunk fel msodrend rekrzit!
vissza a feladathoz
6.14. Az elzekhez hasonlan rjunk fel msodrend rekrzit!
vissza a feladathoz
6.15. Igazoljuk a Fibonacci-szmokra vonatkoz albbi lltsokat:
a) F
1
+F
2
+ +F
n
= F
n+2
1 n 1,
b) F
1
+F
3
+ +F
2n1
= F
2n
n 1,
c) F
2
+F
4
+ +F
2n
=F
2n+1
1 n 1,
d) F
1
F
2
+F
3
+(1)
n+1
F
n
= (1)
n+1
F
n1
+1 n 1,
e) F
n+m
=F
n1
F
m
+F
n
F
m+1
n, m1,
f) F
2
1
+F
2
2
+ +F
2
n
=F
n
F
n+1
1 n 1
g) F
n+1
F
n1
F
2
n
= (1)
n
(n 1, Cassini-kplet),
h) lim
n
F
n+1
F
n
=.
Megolds:
a) Az llts igazolhat n-szerinti teljes indukcival, vagy gy:
F
2
=F
1
+F
0
F
3
=F
2
+F
1
F
4
=F
3
+F
2
...........
F
n+2
=F
n+1
+F
n
sszeadva a fenti egyenlsgeket kapjuk, hogy F
n+2
= 1+F
1
+F
2
+... +F
n
.
b) Az llts n-szerinti teljes indukcival igazolhat.
c) Az llts n-szerinti teljes indukcival igazolhat.
203
Bizonyts.
Az llts igazolhat n-szerinit teljes indukcival, vagy a b) s c) llts felhasznlsval
eset sztvlasztssal :
Ha n pros (n = 2m):
n

i=1
F
i
(1)
i+1
=
m

i=1
F
2i1
. .
F
2m
=F
n

i=1
F
2i
. .
F
2m+1
1=F
n+1
1
= F
n
F
n+1
+1 =
= F
n
(F
n
F
n1
)+1 =F
n1
+1 = (1)
n+1
F
n1
+1
Ha n pratlan (n = 2m+1):
n

i=1
F
i
(1)
i+1
=
m

i=1
F
2i1
. .
F
2m
=F
n1

i=1
F
2i
. .
F
2m+1
1=F
n
1
+F
n
=
= F
n1
F
n
+1+F
n
=F
n1
+1 = F
n1
+1 = (1)
n+1
F
n1
+1.
d) Bizonyts. m-szerinti teljes indukcival :
i) m= 1 esetn az llts igaz, hiszen
F
n+1
=F
n1
F
1
+F
n
F
2
= F
n1
+F
n
.
ii) Tegyk fel, hogy valamely m N

esetn az llts teljesl, azaz


F
n+m
=F
n1
F
m
+F
n
F
m+1
n N
iii) Igazoljuk m+1-re:
F
n+(m+1)
= F
n+1+m
= F
n+11
F
m
+F
n+1
F
m+1
= F
n
F
m
+(F
n
+F
n1
) F
m+1
=
= F
n
(F
m
+F
m+1
)+F
n1
F
m+1
=F
n
F
m+2
+F
n1
F
m+1
e) Bizonyts. n-szerinti teljes indukcival :
i) n = 1 esetn az llts teljesl, hiszen
1 = F
2
1
=F
1
F
2
= 1 1
ii) Tegyk fel, hogy valamely n N

esetn az llts teljesl, azaz


F
2
1
+F
2
2
+ +F
2
n
=F
n
F
n+1
1
item Igazoljuk n+1-re:
F
2
1
+F
2
2
+... +F
2
n
+F
2
n+1
=F
n
F
n+1
+F
2
n+1
=F
n+1
(F
n+1
+F
n
) = F
n+1
F
n+2
.
f) Bizonyts. n-szerinti teljes indukcival :
204 III.6. FEJEZET. FIBONACCI-SZMOK
i) n = 1 esetn az llts teljesl, hiszen
F
2
F
0
F
2
1
= 0 11
2
=1.
ii) Tegyk fel, hogy valamely n N

esetn az llts teljesl, azaz


F
n+1
F
n1
F
2
n
= (1)
n
iii) Igazoljuk n+1-re:
F
n+2
F
n
F
2
n+1
= (F
n+1
+F
n
) F
n
F
2
n+1
=F
n+1
F
n
+F
2
n
F
2
n+1
=F
2
n
F
n+1
(F
n+1
F
n
) =
=F
2
n
F
n+1
F
n1
=(1)
n
= (1)
n+1
.
g)
lim
n
F
n+1
F
n
= lim
n

F
n+1

n+1

5
F
n
= lim
n

n+1

n+1
_

n+1

5
1

n
_ =
= lim
n

_
1
_

_
n+1
_

_
1
_

_
n
_
=.
Mivel =
1+

5
2
s =
1

5
2
, ezrt 0 <

=
1

5
1+

5
< 1. gy lim
n
_

_
n+1
= lim
n
_

_
n
= 0.
vissza a feladathoz
6.16. Igazoljuk, hogy az F
n
Fibonacci-szmokra:
a) n [ m akkor s csak akkor teljesl, ha F
n
[ F
m
(n, m1),
b) (F
n
, F
m
) = F
(n,m)
(n, m1), ahol (u, v) az u s v szmok legnagyobb kzs osztja.
Bizonyts.
a) Tegyk fel, hogy n[ m. Ekkor legyen m=kn, ahol k 1. Igazoljuk az lltst k-szerinti
teljes indukcival !
i) k = 1 esetn F
n
=F
m
[ F
m
nyilvnvalan teljesl.
ii) Tegyk fel, hogy a tulajdonsg igaz valamely k N

esetn, azaz F
kn
= F
n
.
iii) Igazoljuk az lltst k+1-re:
F
(k+1)n
=F
kn+n
7.13.e
= F
kn1
F
n
+F
kn
F
n+1
ii)
=F
kn1
F
n
+F
n
F
n+1
=F
n
(F
kn1
+F
n+1
),
azaz F
n
valban osztja F
(k+1)n
-nek.
Airny igazolstl itt eltekintnk, mivel az az (F
n
, F
m
)=F
(n,m)
egyenlsgbl kvetkezik.
Legyen F
n
[ F
m
, ekkor F
(n,m)
= (F
n
, F
m
) = F
n
, ahonnan (n, m) = n, azaz n [ m.
b) Az lltst hrom lpsben fogjuk igazolni :
205
i) (F
n
, F
n+1
) = 1.
Valban, a legnagyobb kzs oszt tulajdonsgai alapjn
(F
n
, F
n+1
) = (F
n
, F
n
+F
n1
) = (F
n
, F
n1
) = = (F
1
, F
2
) = 1.
ii) Legyen a = b q +r, ekkor (F
a
, F
b
) = (F
b
, F
r
):
(F
a
, F
b
) = (F
bq+r
, F
b
)
7.13.e
= (F
bq1
F
r
+F
bq
F
r+1
, F
b
) = (F
bq1
F
r
, F
b
) = (F
r
, F
b
).
Az elz levezets sorn kihasznltuk, hogy F
b
[ F
bq
s hogy (F
bq
, F
bq1
) = 1.
iii) Alkalmazzuk az euklideszi algoritmust az n, m szmokra:
n =mq
1
+r
1
, 0 <r
1
<m,
m=r
1
q
2
+r
2
, 0 <r
2
<r
1
,
r
1
=r
2
q
3
+r
3
, 0 <r
3
<r
2
,
.
.
.
r
s2
=r
s1
q
s
+r
s
, 0 <r
s
<r
s1
,
r
s1
=r
s
q
s+1
, r
s+1
= 0,
ahol (n, m) = r
s
. Innen ii) szerint
(F
n
, F
m
) = (F
m
, F
r
1
),
(F
m
, F
r
1
) = (F
r
1
, F
r
2
),
(F
r
1
, F
r
2
) = (F
r
2
, F
r
3
),
.
.
.
(F
r
s2
, F
r
s1
) = (F
r
s1
, F
r
s
),
ahol r
s
[ r
s1
miatt F
r
s
[ F
r
s1
s kapjuk, hogy (F
n
, F
m
) = (F
r
s1
, F
r
s
) = F
r
s
=F
(n,m)
.
vissza a feladathoz
6.17. Igaz-e, hogy az F
n
Fibonacci-szmokra ha F
n
prm, akkor n is prm?
Megolds. F
4
= 3 prm, de 4 nem prm, teht az llts nem igaz. De ha n ,= 4 s F
n
prm,
akkor n prm. Valban, tegyk fel, hogy F
n
prm s n>4 sszetett. Akkor ltezik d[n, 2<d<n
s kapjuk, hogy F
d
[ F
n
, ahol 2 F
d
<F
n
, teht F
n
sszetett, ami ellentmonds .
Megjegyzs. Fordtva nem igaz. Ha n prm, akkor nem biztos, hogy F
n
is prm. A legkisebb
ellenplda: F
19
= 4181 = 113 37. Nem tudjuk, hogy van-e vgtelen sok Fibonacci-prm.
vissza a feladathoz
206 III.6. FEJEZET. FIBONACCI-SZMOK
III.7. fejezet
Catalan-szmok
7.9. Az origbl indulva, mindig jobbra vagy felfel lpve egyet-egyet hnyflekppen juthatunk
el a (7,11) pontba gy, hogy sosem lpnk olyan helyre, ahol y =x3?
tlet: A I.7.7-es feladatban ltottakhoz hasonlan jrhatunk el, csak most a tkrzst az
y = x3 egyenesre vgezzk. Figyeljnk arra is, hogy most a j utak haladnak az egyenes
felett s a rossz utak alatta.
vissza a feladathoz
7.10. Hny olyan sorozat kpezhet n darab +1 s n darab 1 felhasznlsval, melyben minden
rszletsszeg 0?
tlet: Induljunk az origbl. brzoljuk f-tpus lpssel, ha a sorozat aktulis eleme +1
s -tpus lpssel, ha 1. Ekkor egy Dyck-utat keresnk az orig s a (2n; 0) pont kztt.
vissza a feladathoz
7.11. Igazoljuk, hogy 2n+1 elem 1 tagokbl ll sorozatok szma, melyben a teljes sszeg 1 s
minden rszletsszeg pozitv
1
2n+1
_
2n+1
n
_
.
Mikze ezeknek a Catalan-szmokhoz?
tlet: Egsztsk ki a sorozatot egy n+1-edik 1-es taggal. Legyen ez a sorozat utols
eleme. Az ilyen sorozatok ellltsval foglalkoztunk eddig, teht a (0; 0) s a (2n+2; 0)
pontok kztti Dyck utak szmra vagyunk kvncsiak.
vissza a feladathoz
207
208 III.7. FEJEZET. CATALAN-SZMOK
III.8. fejezet
Stirling-szmok
8.6. Igazoljuk a msodfaj Stirling-szmokra vonatkoz albbi rekrzit! Ha 1 k n, akkor
_
n
k
_
=
nk+1

j=1
_
n1
j 1
__
nj
k1
_
.
Megolds.
Tekintsk az 1,2, ..., n halmaz k rszre val partciit. Ezek szma
n
k
. Msrszt csoportostsuk
ezeket aszerint, hogy hny elembl ll az n-et tartalmaz rszhalmaz. Ha j elembl ll, ahol
1 j nk+1 (mert j +k1 n kell legyen), akkor a tbbi j 1 elemet
_
n1
j1
_
-flekppen
lehet megvlasztani, a fennmarad k1 rszhalmazt pedig
nj
k1
-flekppen. Ezek szorzatait
kell sszegezni s kszen is vagyunk.
vissza a feladathoz
8.7. Igazoljuk a Bell-szmokra vonatkoz albbi rekrzit! (Lsd [16])
Ha n, m0, akkor a 0
0
= 1 konvenci hasznlatval
B(n+m) =
m

j=0
n

k=0
j
nk
_
m
j
__
n
k
_
B(k).
Megolds.
Ha n = 0, illetve ha m= 1, akkor innen a kvetkez ismert kpleteket kapjuk vissza:
() B(m) =
m

j=0
_
m
j
_
, () B(n+1) =
n

k=0
_
n
k
_
B(k).
Ha n=0, akkor a fenti (*) kpletet kapjuk. Ha m=0, akkor B(n)=B(n) addik, mert
0
0
=1.
Legyenek A s B olyan diszjunkt halmazok, amelyekre [A[ =m1, [B[ =n 1. Akkor [A
B[ = m+n, s nzzk, hogy hnyflekppen lehet particionlni az AB halmazt. Ez a
szm egyrszt B(n+m). Msrszt, az A halmazt osszuk j rszhalmazra. Ez
m
j
-flekppen
lehetsges. Tovbb, B-nek vlasszuk ki k elemt, ezt
_
n
k
_
-flekppen lehet, s ezt a k elemet

n
k
-flekppen lehet particionlni. A B halmaz tbbi nk elemt vegyk hozz az elbbi j
rszhalmazhoz, erre j
nk
lehetsg van.
209
210 III.8. FEJEZET. STIRLING-SZMOK
Teht j
nk

m
j

_
n
k
_
B(k) olyan partci van, hogy az A halmazt j rszhalmazra particionltuk
s B-nek k eleme alkot j rszhalmazokat. sszegezve j s k szerint megkpjuk az AB halmaz
partciinak a szmt.
vissza a feladathoz
8.8. Igazoljuk, hogy a msodfaj Stirling-szmokra
_
n
k
_
=

a
1
+...+a
k
=n
a
1
,...,a
k
1
n!
k! a
1
! a
k
!
.
Megolds.
_
n
k
_
megadja, hogy hnyflekppen lehet particionlni egy n elem halmazt k
rszhalmazra.
Ha a
1
, . . . , a
k
1 adottak gy, hogy a
1
+. . . +a
k
=n, akkor hny olyan partci van, hogy az
els halmazba (dobozba) a
1
elem kerl,..., a k-adik halmazba (dobozba) a
k
elem kerl ? Erre
a vlasz
n!
a
1
!a
k
!
, lsd 4.1. Feladat, de most a hamazok (dobozok) sorrendje nem szmt, ezrt
az ilyen partcik szma
n!
k!a
1
!a
k
!
. sszegezve ksz.
vissza a feladathoz
8.9. Igazoljuk, hogy minden n k 1 esetn

x
1
+...+x
k
=n
x
1
,...,x
k
1
1
x
1
x
k
=
k!
n!
_
n
k
_
,
ahol
_
n
k

az elsfaj Stirling-szmok.
Megolds. Legyenek x
1
, . . . , x
k
1 rgztettek gy, hogy x
1
+. . . +x
k
= n. Hny olyan n-
edfok permutci van, amelyben a ciklusok szma k s ezeknek a hossza rendre x
1
,...,x
k
gy, hogy a ciklusok sorrendjre nem vagyunk tekintettel ? Ez a szm
n!
k! x
1
x
k
, mert egy
x
i
hosszsg ciklus x
i
-flekppen rhat, a ciklusok pedig k!-flekppen permutlhatk.
sszegezve megkapjuk az sszes k ciklus alkotta permutcik szmt, ami msrszt pontosan
_
n
k

.
vissza a feladathoz
8.10. Igazoljuk, hogy a B(n) Bell-szmokra B(n) <n! minden n 3-ra.
tmutats: Vgezznk n szerinti indukcit hasznlva a B(n+1) =

n
k=0
_
n
k
_
B(k) rekurzit.
Mskpp: Az 1,2, . . . , n halmaz minden partcijhoz rendeljk hozz az 1,2, . . . , n szmoknak
azt a permutcijt, amelynek egyes ciklusait az ugyanahhoz a rszhalmazhoz tartoz szmok
alkotjk. Vizsgljuk ezt a megfeleltetst!
vissza a feladathoz
III.9. fejezet
Grfelmleti fogalmak
9.35. Egy grf cscsai reprezentljk a termszetes szmokat 1-tl 8-ig. A cscsbl B cscsba fusson
irnytott l, ha az A-nak megfeleltetett termszetes szm osztja a B-nek megfeleltetett
termszetes szmnak. Rajzoljuk meg a grfot!
Megolds:
vissza a feladathoz
9.36. Adjuk meg, hny 4 pont egyszer grf van! Rajzoljuk fel a nemizomorf eseteket. (Ezek szma
11, lsd TL: Kombinatorika 37.o.)
Megolds:
Az sszes lt behzva egy teljesgrfhoz jutunk, melynek 6 le van. gy 2
6
= 64 klnbz
szmozott grf rajzolhat. A nemizomorfak a kvetkezk:
0-l grf (resgrf) egyetlen egy darab van:
1-l grf hat darab van, de ezek mind izomorfak egymssal :
211
212 III.9. FEJEZET. GRFELMLETI FOGALMAK
2-l grf
_
6
2
_
= 15 darab van, de ezek az albbi kt grf valamelyikvel izomorfak:
3-l grf
_
6
3
_
= 20 darab van, de ezek az albbi hrom grf valamelyikvel izomorfak:
4-l grf
_
6
4
_
= 15 darab van, de ezek az albbi kt grf valamelyikvel izomorfak:
5-l grf
_
6
5
_
= 6 darab van, de ezek mind izomorfak:
6-l grf egyetlen egy darab van:
vissza a feladathoz
213
9.37. Rajzoljuk meg az albbi szomszdsgi mtrixhoz tartoz grfot!
_

_
0 1 1 0 0
1 0 0 1 1
1 0 0 0 0
0 1 0 0 1
0 1 0 1 0
_

_
Megolds:
vissza a feladathoz
9.38. Rajzoljuk meg az albbi illeszkedsi mtrixhoz tartoz grfot!
_

_
1 1 0 0 0 1
0 0 1 0 0 0
0 0 0 0 1 1
0 0 0 1 1 0
0 1 1 1 0 0
1 0 0 0 0 0
_

_
Megolds:
vissza a feladathoz
9.39. Hny kr van a teljes ngyszgben? Hny t hossz kr van a teljes tsben, hny hat hossz
kr van a teljes hatosban? Hny n hossz kr van a teljes n-esben? (Az utols krds gy is
fogalmazhat: hny Hamilton-kre van a teljes n-esnek?)
Megolds:
A teljes ngyesben ngy hromszg van s hrom ngyszg. Azrt hrom, mert minden
ngyszg komplementere egy-egy prhuzamos lpr, s ilyen hrom van.
214 III.9. FEJEZET. GRFELMLETI FOGALMAK
A teljes tsben 12 tszg van. Ezt az albbi mdon bizonythatjuk: Kivlasztunk egy pontot,
ennek a kt szomszdja
_
4
2
_
= 6-flekppen vlaszthat ki, a maradk kt pont pedig ktfle
sorrendben illeszthet be a krbe. Ez 6 2 = 12 kr.
A fenti meggondols n pont esetre is tvihet, gy a teljes n-szg Hamilton-kreinek szma
_
n1
2
_
(n3)! =
(n1)!
2
.
vissza a feladathoz
Irodalomjegyzk
[1] Andrsfalvi Bla: Grfelmlet, Akadmiai Kiad, Budapest, 1989.
[2] T. Davis: Catalan Numbers, 2006,
http://mathcircle.berkeley.edu/BMC6/ps/catalan.pdf
[3] Elekes Gyrgy: Kombinatorika feladatok, ELTE jegyzet, 2000.
[4] R. L. Graham, D. E. Knuth, O. Patashnik: Konkrt matematika, Mszaki Knyvkiad,
Budapest, 1998.
[5] Hajnal Pter: Elemi kombinatorikai feladatok, Polygon, Szeged, 1997.
[6] Hajnal Pter: sszeszmllsi problmk, Polygon, Szeged, 1997.
[7] Hajnal Pter: Grfelmlet, Polygon, Szeged, 1997.
[8] T. Harju: Lecture Notes in Combinatorial Enumeration, A short course, University of Turku,
2004, http://users.utu.fi/harju/combinenum/MainEn.pdf
[9] Hetyei Gbor: Kombinatorika s grfelmlet, Janus Pannonius Tudomnyegyetem Pcs, 1996.
[10] Hetyei Gbor, Kamars Lajos: Elemi matematika III., feladatgyjtemny, Pcsi
Tudomnyegyetem, 2000.
[11] Kalmrn N. Mrta - Katonn H. Eszter - Kmn Tams: Diszkrt matematikai feladatok,
Polygon, Szeged, 2005.
[12] Lovsz Lszl: Kombinatorikai problmk s feladatok, Typotex, Budapest, 1999.
[13] Lovsz Lszl, Pelikn Jzsef, Vesztergombi Katalin: Diszkrt matematika, Typotex,
Budapest, 2006.
[14] A. de Mier: Lecture notes for Enumerative Combinatorics, University of Oxford, 2004,
http://www.math.dartmouth.edu/archive/m68f05/public_html/lectec.pdf
[15] Orosz Gyula: Rekurzv sorozatok
[16] M. Z. Spivey, A generalized recurrence for Bell numbers, Journal of Integer Sequences, Vol.
11 (2008), Article 08.2.5.
[17] Surnyi Lszl: Grfelmlet,
http://home.fazekas.hu/~lsuranyi/Grafok/Grafelmelet.htm
[18] Szendrei gnes: Diszkrt matematika - logika, algebra, kombinatorika, Polygon, Szeged, 2000.
[19] N. J. Vilenkin: Kombinatorika, Mszaki Knyvkiad, Budapest, 1987.
215

Вам также может понравиться